SlideShare a Scribd company logo
1 of 120
Download to read offline
ĐẠI HỌC THÁI NGUYÊN
           TRƯỜNG ĐẠI HỌC KHOA HỌC




                 Hoàng Ngọc Quang




MỘT SỐ BẤT ĐẲNG THỨC HÌNH HỌC


    Chuyên Nghành: PHƯƠNG PHÁP TOÁN SƠ CẤP
                   MÃ SỐ: 60.46.40




        LUẬN VĂN THẠC SĨ TOÁN HỌC




  Người hướng dẫn khoa học: TS. Nguyễn Văn Ngọc




                 Thái Nguyên - 2011


   www.MATHVN.com - HOANG NGOC QUANG, Yen Bai
Công trình được hoàn thành tại
       Trường Đại học Khoa học - Đại học Thái Nguyên




      Người hướng dẫn khoa học: TS. Nguyễn Văn Ngọc




Phản biện 1: . . . . . . . . . . . . . . . . . . . . . . . . . . . . . . . . . . . . . . . . . . . . . . . . . . . . . . .
....................................................................




Phản biện 2: . . . . . . . . . . . . . . . . . . . . . . . . . . . . . . . . . . . . . . . . . . . . . . . . . . . . . . .
....................................................................




 Luận văn sẽ được bảo vệ trước hội đồng chấm luận văn họp tại:
    Trường Đại học Khoa học - Đại học Thái Nguyên
                 Ngày .... tháng .... năm 2011




                               Có thể tìm hiểu tại
                          Thư viện Đại học Thái Nguyên


         www.MATHVN.com - HOANG NGOC QUANG, Yen Bai
1


Mục lục



   Mục lục . . . . . . . . . . . . . . . . . . . . . . . . . . . . .              1
   Mở đầu . . . . . . . . . . . . . . . . . . . . . . . . . . . . . .             3

Chương 1. Các bất đẳng thức trong tam giác và tứ giác                             6
  1.1. Các bất đẳng thức đại số cơ bản . . . . . . . . . . . . . .                6
  1.2. Các đẳng thức và bất đẳng thức cơ bản trong tam giác .                     8
       1.2.1. Các đẳng thức cơ bản trong tam giác . . . . . . .                   8
       1.2.2. Các bất đẳng thức cơ bản trong tam giác . . . . .                  10
  1.3. Bất đẳng thức trong tam giác . . . . . . . . . . . . . . .                11
       1.3.1. Bất đẳng thức về độ dài các cạnh . . . . . . . . .                 11
       1.3.2. Bất đẳng thức về các đại lượng đặc biệt . . . . .                  14
  1.4. Các bất đẳng thức sinh ra từ các công thức hình học . .                   17
  1.5. Bất đẳng thức trong các tam giác đặc biệt . . . . . . . .                 23
       1.5.1. Các bất đẳng thức trong tam giác đều . . . . . .                   23
       1.5.2. Các bất đẳng thức trong tam giác vuông và tam
              giác cân . . . . . . . . . . . . . . . . . . . . . . .             27
  1.6. Các bất đẳng thức khác trong tam giác . . . . . . . . . .                 29
  1.7. Các bất đẳng thức trong tứ giác . . . . . . . . . . . . . .               40
       1.7.1. Các bất đẳng thức cơ bản trong tứ giác . . . . . .                 41
       1.7.2. Các bất đẳng thức khác trong tứ giác . . . . . . .                 45

Chương 2. Bất đẳng thức Ptolemy            và các mở rộng                        48
  2.1. Định lí Ptolemy . . . . . . . . .   . . . . . . . . . .   .   .   .   .   48
  2.2. Bất đẳng thức Ptolemy . . . . .     . . . . . . . . . .   .   .   .   .   53
  2.3. Định lí Bretschneider . . . . . .   . . . . . . . . . .   .   .   .   .   63
  2.4. Định lí Casey . . . . . . . . . .   . . . . . . . . . .   .   .   .   .   63
  2.5. Mở rộng bất đẳng thức Ptolemy       trong không gian      .   .   .   .   68




        www.MATHVN.com - HOANG NGOC QUANG, Yen Bai
2

Chương 3. Bất đẳng thức Erdos-Mordell và các mở rộng                   70
  3.1. Bất đẳng thức Erdos-Mordell trong tam giác . . . . . . .        70
  3.2. Bất đẳng thức Erdos-Mordell trong tam giác mở rộng . .          79
  3.3. Mở rộng bất đẳng thức Erdos-Mordell trong tứ giác . . .         85
  3.4. Mở rộng bất đẳng thức Erdos-Mordell trong đa giác . . .         87
  3.5. Mở rộng bất đẳng thức Erdos-Mordell trong tứ diện . . .         90

Chương 4. Các bất đẳng thức có trọng                                    92
  4.1. Bất đẳng thức dạng Hayashi và các hệ quả . . . . . .        . . 92
       4.1.1. Bất đẳng thức Hayashi . . . . . . . . . . . . .      . . 92
       4.1.2. Các hệ quả của bất đẳng thức hyashi . . . . .        . . 94
       4.1.3. Bài toán áp dụng . . . . . . . . . . . . . . . .     . . 94
  4.2. Bất đẳng thức Weizenbock suy rộng và các hệ quả .           . . 96
       4.2.1. Bất đẳng thức Weizenbock suy rộng . . . . .          . . 96
       4.2.2. Các hệ quả của bất đẳng thức Weizenbock suy          rộng101
  4.3. Bất đẳng thức Klamkin và các hệ quả . . . . . . . .         . . 105
       4.3.1. Bất đẳng thức Klamkin . . . . . . . . . . . .        . . 105
       4.3.2. Các hệ quả của bất đẳng thức Klamkin . . . .         . . 106
  4.4. Bất đẳng thức Jian Liu và các hệ quả . . . . . . . .        . . 108
       4.4.1. Bất đẳng thức Jian Liu . . . . . . . . . . . . .     . . 108
       4.4.2. Các hệ quả của bất đẳng thức Jian Liu . . . .        . . 110
  Kết luận . . . . . . . . . . . . . . . . . . . . . . . . . . .   . . 116
  Tài liệu tham khảo . . . . . . . . . . . . . . . . . . . . .     . . 117




        www.MATHVN.com - HOANG NGOC QUANG, Yen Bai
3


Mở đầu



   Các bài toán về bất đẳng thức và cực trị hình học thuộc loại những
bài toán khó, làm cho học sinh phổ thông, nhất là phổ thông cơ sở kể cả
học sinh giỏi lúng túng khi gặp các bài toán loại này. Thực sự nó là một
phần rất quan trọng của hình học và những kiến thức về bất đẳng thức
trong hình học cũng làm phong phú hơn phạm vi ứng dụng của toán
học. So với các bất đẳng thức đại số, các bất đẳng thức hình học chưa
được quan tâm nhiều. Một trong những nguyên nhân gây khó giải quyết
vấn đề này là vì phương pháp tiếp cận không phải là các phương pháp
thông thường hay được áp dụng trong hình học và càng không phải là
phương pháp đại số thuần túy. Để giải một bài toán về bất đẳng thức
hình học cần thiết phải biết vận dụng các kiến thức hình học và đại số
một cách thích hợp và nhạy bén.
   Luận văn này giới thiệu một số bất đẳng thức hình học từ cơ bản
đến nâng cao và mở rộng. Các bài toán về bất đẳng thức hình học được
trình bày trong luận văn này có thể tạm phân thành các nhóm sau:
   I. Nhóm các bài toán mà trong lời giải đòi hỏi nhất thiết phải có
hình vẽ. Phương pháp giải các bài toán nhóm này chủ yếu là "phương
pháp hình học", như vẽ thêm đường phụ, sử dụng tính chất giữa đường
vuông góc và đường xiên, giữa đường thẳng và đường gấp khúc, quan
hệ giữa các cạnh, giữa cạnh và góc trong một tam giác, hay tứ giác v.v..
Bất đẳng thức và cực trị trong hình học phẳng thuộc nhóm này là nội
dung thường gặp trong các kì thi chọn học sinh giỏi toán hay thi vào các
trường chuyên.
  II. Nhóm thứ hai gồm các bài toán mà khi giải chúng cần phải sử
dụng các hệ thức lượng đã biết, như các hệ thức lượng giác, hệ thức
đường trung tuyến, đường phân giác, công thức các bán kính, công thức


        www.MATHVN.com - HOANG NGOC QUANG, Yen Bai
4

diện tích của tam giác v.v.. Các bài toán này đã được quan tâm nhiều
và chúng được trình bày khá phong phú trong các tài liệu [4,7], vì thế
luận văn này sẽ không đề cập nhiều đến các bất đẳng thức trong tam
giác có trong các tài liệu trên mặc dù chúng rất hay mà chỉ nêu ra một
số bất đẳng thức cơ bản nhất để tiện sử dụng sau này.
   III. Nhóm thứ ba gồm các bài toán liên quan đến các bất đẳng thức
hình học nổi tiếng, đặc biệt là bất đẳng thức Ptolemy và bất đẳng thức
Erdos-Mordell và các bất đẳng thức có trọng như bất đẳng thức Hayshi,
bất đẳng thức Weizenbock, bất đẳng thức Klamkin v.v.. Các bất đẳng
thức này còn ít được giới thiệu bằng Tiếng Việt và thường gặp trong các
đề thi Olympic Quốc tế.
  Bản luận văn "Một số bất đẳng thức hình học" gồm có mở đầu,
bốn chương nội dung, kết luận và tài liệu tham khảo.
  Chương 1. Các bất đẳng thức trong tam giác và tứ giác.
Chương này trình bày một số bất đẳng thức thuộc nhóm I và nhóm II.
   Chương 2. Bất đẳng thức Ptolemy và các mở rộng.
Chương này trình bày đẳng thức Ptolemy, bất đẳng thức Ptolemy và
các bài toán áp dụng. Các bài toán này chủ yếu được trích ra từ các đề
thi vô địch các nước, đề thi vô địch khu vực và đề thi IMO, một số là do
tác giả sáng tác. Ngoài ra, còn trình bày một số mở rộng bất đẳng thức
Ptolemy trong tứ giác và trong tứ diện.
  Chương 3. Bất đẳng thức Erdos - Mordell và các mở rộng.
Chương này trình bày bất đẳng thức Edos-Mordell và các bài toán liên
quan. Ngoài ra, còn trình bày một số mở rộng bất đẳng thức này trong
tam giác, trong tứ giác và trong đa giác [11-13].
   Chương 4. Các bất đẳng thức có trọng.
Chương này trình bày một số bất đẳng thức liên quan đến tổng khoảng
cách từ một hay nhiều điểm của mặt phẳng đến các đỉnh hoặc các cạnh
của tam giác với các tham số dương tùy ý được gọi là trọng số hay gọi
tắt là trọng. Đó là các bất đẳng thức Hyashi, Weizenbock, Klamkin, Jian



        www.MATHVN.com - HOANG NGOC QUANG, Yen Bai
5

Liu, v.v.. Các bất đẳng thức này còn ít được giới thiệu bằng Tiếng Việt,
một số là kết quả nghiên cứu của các chuyên gia Quốc tế trong lĩnh vực
bất đẳng thức hình học [9,13-14].
   Luận văn này được hoàn thành tại trường Đại học Khoa học - Đại
học Thái Nguyên với sự hướng dẫn của TS. Nguyễn Văn Ngọc. Tác giả
xin được bày tỏ lòng biết ơn sâu sắc đối với sự quan tâm hướng dẫn của
Thầy, tới các thầy cô trong Ban Giám hiệu, Phòng Đào tạo và Khoa
Toán-Tin Trường Đại học Khoa học. Đồng thời tác giả xin cảm ơn tới
Sở GD - ĐT tỉnh Yên Bái, Ban Giám đốc, các đồng nghiệp Trung tâm
GDTX - HNDN Hồ Tùng Mậu huyện Lục Yên đã tạo điều kiện cho tác
giả học tập và hoàn thành kế hoạch học tập.


                              Thái Nguyên, ngày 20 tháng 6 năm 2011.
                                             Tác giả




                                       Hoàng Ngọc Quang




        www.MATHVN.com - HOANG NGOC QUANG, Yen Bai
6

Chương 1
Các bất đẳng thức trong tam giác
và tứ giác

   Chương này trình bày các bất đẳng thức trong tam giác và tứ giác
từ cơ bản đến nâng cao. Nội dung chủ yếu được hình thành từ các tài
liệu [1-7], [10], [12] và [15].

   Kí hiệu ∆ABC là tam giác ABC với các đỉnh là A, B, C. Để thuận
tiện, độ lớn của các góc ứng với các đỉnh A, B, C cũng được kí hiệu tương
ứng là A, B, C.
   Độ dài các cạnh của tam giác: BC = a, CA = b, AB = c.
                                   a+b+c
   Nửa chu vi của tam giác: p =                 .
                                         2
   Đường cao với các cạnh: ha , hb , hc .
   Đường trung tuyến với các cạnh: ma , mb , mc .
   Đường phân giác với các cạnh: la , lb , lc .
   Bán kính đường tròn ngoại tiếp và đường tròn nội tiếp: R và r.
   Bán kính đường tròn bàng tiếp các cạnh: ra , rb , rc .
   Diện tích tam giác ABC: S, SABC hay [ABC].

   Để giải được các bài toán bất đẳng thức hình học, trước hết ta cần
trang bị những kiến thức cơ sở đó là các bất đẳng thức đại số cơ bản và
các đẳng thức, bất đẳng thức đơn giản trong tam giác.

1.1.   Các bất đẳng thức đại số cơ bản

Định lý 1.1. (Bất đẳng thức AM-GM) Giả sử a1 , a2 , · · · , an là các số
thực không âm. Khi đó
                    a1 + a2 + · · · + an   √
                                         ≥ n a1 a2 ...an .          (1.1)
                            n
Đẳng thức xảy ra khi và chỉ khi a1 = a2 = · · · = an .


        www.MATHVN.com - HOANG NGOC QUANG, Yen Bai
7

Hệ quả 1.1. Với mọi bộ số dương a1 , a2 , · · · , an ta có
                 √                        n
                 n
                   a1 a2 ...an ≥ 1                     .                                         (1.2)
                                 a1 + a2 + · · · + a1n
                                      1


Đẳng thức xảy ra khi và chỉ khi a1 = a2 = · · · = an .
Hệ quả 1.2. Với mọi bộ số dương a1 , a2 , · · · , an ta có
                   1   1         1            n2
                     +   + ··· +    ≥                      .                                     (1.3)
                   a1 a2         an   a1 + a2 + · · · + an
Đẳng thức xảy ra khi và chỉ khi a1 = a2 = ... = an .
Hệ quả 1.3. Với mọi bộ số không âm a1 , a2 , · · · , an và m = 1, 2, · · · ta
có
                                                                                m
              am + am + · · · + am
               1    2            n                     a1 + a2 + · · · + an
                                   ≥                                                .            (1.4)
                      n                                        n
Đẳng thức xảy ra khi và chỉ khi a1 = a2 = · · · = an .
Định lý 1.2. (Bất đẳng thức Cauchy - Schwarz) Cho hai dãy số thực
a1 , a2 , · · · , an và b1 , b2 , · · · , bn . Khi đó
         (a1 b1 + a2 b2 + · · · + an bn )2 ≤ a2 + a2 + · · · + a2
                                              1    2            n   b2 + b2 + · · · + b2 .
                                                                     1    2            n          (1.5)

                                              a1        a2             an
Đẳng thức xảy ra khi và chỉ khi               b1   =    b2   = ··· =   bn .

Định lý 1.3. (Bất đẳng thức Jensen) Cho f (x) là hàm số liên tục và có
đạo hàm cấp hai trên I (a, b) và n điểm x1 , x2 , · · · , xn tùy ý trên đoạn
I (a, b). Khi đó
   i, Nếu f (x) > 0 với mọi x ∈ I (a, b) thì
                                                              x1 + x2 + · · · + xn
        f (x1 ) + f (x2 ) + · · · + f (xn ) ≥ nf                                             .
                                                                      n
   ii, Nếu f (x) < 0 với mọi x ∈ I (a, b) thì
                                                              x1 + x2 + · · · + xn
        f (x1 ) + f (x2 ) + · · · + f (xn ) ≤ nf                                             .
                                                                      n

   Ở đây I (a, b) nhằm ngầm định là một trong bốn tập hợp (a, b) , [a, b) ,
(a, b] , [a, b].


        www.MATHVN.com - HOANG NGOC QUANG, Yen Bai
8

Định lý 1.4. (Bất đẳng thức Chebyshev) Cho hai dãy số thực đơn điệu
cùng chiều a1 , a2 , · · · , an và b1 , b2 , · · · , bn . Khi đó ta có
                                           1
           a1 b1 + a2 b2 · · · + an bn ≥     (a1 + a2 + · · · + an ) (b1 + b2 + · · · + bn ) .     (1.6)
                                           n

 Nếu hai dãy số thực a1 , a2 , · · · , an và b1 , b2 , · · · , bn đơn điệu ngược chiều
thì bất đẳng thức trên đổi chiều.
Định lý 1.5. (Bất đẳng thức Nesbitt) Cho a, b, c là các số thực dương.
Bất đẳng thức sau luôn đúng
                               a   b   c  3
                                 +   +   ≥ .                                                      (1.7)
                              b+c c+a a+b 2
Đẳng thức xảy ra khi và chỉ khi a = b = c.

1.2.     Các đẳng thức và bất đẳng thức cơ bản trong tam giác

1.2.1.    Các đẳng thức cơ bản trong tam giác
Định lý 1.6. (Định lý hàm số sin) Trong tam giác ABC ta có
                               a     b       c
                                  =      =       = 2R.
                             sin A sin B   sin C
Định lý 1.7. (Định lý hàm số cosin) Trong tam giác ABC ta có

a2 = b2 + c2 − 2bc cos A, b2 = c2 + a2 − 2ca cos B, c2 = a2 + b2 − 2ab cos C.

Định lý 1.8. (Các công thức về diện tích) Diện tích tam giác ABC
được tính theo một trong các công thức sau
                         1       1       1
                     S = aha = bhb = chc                                                          (1.8)
                         2       2       2
                         1           1            1
                       = bc sin A = ca sin B = ab sin C                                           (1.9)
                         2           2            2
                       = pr                                                                      (1.10)
                         abc
                       =                                                                         (1.11)
                         4R
                       = (p − a)ra = (p − b)rb = (p − c)rc                                       (1.12)
                         =      p (p − a) (p − b) (p − c).                                       (1.13)

Công thức (1.13) được gọi là công thức Hê-rông.


          www.MATHVN.com - HOANG NGOC QUANG, Yen Bai
9

Định lý 1.9. (Định lý đường phân giác) Trong một tam giác, đường
phân giác của một góc chia cạnh đối diện thành hai đoạn thẳng tỉ lệ với
hai cạnh kề hai đoạn ấy .

Định lý 1.10. (Công thức đường phân giác) Trong tam giác ABC ta
có
             2bc    A        2ca    B          2ab    C
       la =      cos , lb =      cos , lc =        cos .
            b+c     2       c+a     2         a+b      2
Định lý 1.11. (Định lý đường trung tuyến) Trong một tam giác, ba
đường trung tuyến gặp nhau tại một điểm được gọi là trọng tâm của
tam giác. Trên mỗi đường trung tuyến, khoảng cách từ trọng tâm đến
đỉnh bằng hai lần khoảng cách trọng tâm đến chân đường trung tuyến.

Định lý 1.12. (Công thức đường trung tuyến) Trong tam giác ABC ta
có
           b2 + c2 a2               c2 + a2 b2                 a2 + b2 c2
    m2
     a   =        − ,        m2
                              b   =        − ,          m2
                                                         c   =        − .
              2    4                   2    4                     2    4
Định lý 1.13. (Công thức bán kính đường tròn nội tiếp) Trong tam
giác ABC ta có
                             A              B             C
           r = (p − a) tan     = (p − b) tan = (p − c) tan .
                             2              2             2
Định lý 1.14. (Công thức bán kính đường tròn bàng tiếp) Trong tam
giác ABC ta có
                           A                   B                  C
              ra = p tan     ,    rb = p tan     ,   rc = p tan     .
                           2                   2                  2
Định lý 1.15. (Các hệ thức lượng giác cơ bản) Với mọi tam giác ABC
ta luôn có các hệ thức sau
                                   A      B      C
      sin A + sin B + sin C = 4 cos   cos cos ,                         (1.14)
                                    2     2      2
      sin 2A + sin 2B + sin 2C = 4 sin A sin B sin C,                   (1.15)
                                        A     B      C
      cos A + cos B + cos C = 1 + 4 sin sin sin ,                       (1.16)
                                        2      2      2
      cos 2A + cos 2B + cos 2C = −1 − 4 cos A cos B cos C,              (1.17)


         www.MATHVN.com - HOANG NGOC QUANG, Yen Bai
10

         sin2 A + sin2 B + sin2 C = 2 (1 + sin A sin B sin C) ,       (1.18)
         cos2 A + cos2 B + cos2 C = 1 − 2 cos A cos B cos C,          (1.19)
         tan A + tan B + tan C = tan A tan B tan C,                   (1.20)
             A        B        C       A     B     C
         cot + cot + cot = cot cot cot ,                              (1.21)
             2        2        2       2      2    2
             A      B       B      C        C     A
         tan tan + tan tan + tan tan = 1,                             (1.22)
             2      2        2     2        2     2
         cot A cot B + cot B cot C + cot C cot A = 1.                 (1.23)

Riêng với hệ thức (1.20) thì tam giác ABC cần giả thiết không vuông.

1.2.2.    Các bất đẳng thức cơ bản trong tam giác
Định lý 1.16. (Bất đẳng thức tam giác) Trong tam giác ABC ta có

    |b − c| < a < b + c,     |c − a| < b < c + a,   |a − b| < c < a + b.

Định lý 1.17. (Các bất đẳng thức lượng giác cơ bản) Với mọi tam giác
ABC ta luôn có các bất đẳng thức sau
                                             √
                                            3 3
                   sin A + sin B + sin C ≤        ,           (1.24)
                                              2
                                             3
                   cos A + cos B + cos C ≤ ,                  (1.25)
                                             2 √
                       A        B        C     3 3
                   cos + cos + cos ≤                ,         (1.26)
                        2       2        2       2
                       A       B        C    3
                   sin + sin + sin ≤ ,                        (1.27)
                       2        2       2    2
                       A     B     C    1
                   sin sin sin ≤ ,                            (1.28)
                       2     2     2    8
                                        1
                   cos A cos B cos C ≤ ,                      (1.29)
                                        8
                                                9
                   sin2 A + sin2 B + sin2 C ≤ ,               (1.30)
                                                4
                        A        B        C √
                   tan + tan + tan ≥ 3,                       (1.31)
                        2        2        2    √
                   tan A + tan B + tan C ≥ 3 3,               (1.32)
                                            √
                   cot A + cot B + cot C ≥ 3.                 (1.33)



          www.MATHVN.com - HOANG NGOC QUANG, Yen Bai
11

  Riêng với bất đẳng thức (1.32) thì tam giác ABC cần giả thiết không
vuông. Đẳng thức xảy ra trong các bất đẳng thức trên khi và chỉ khi
ABC là tam giác đều.

1.3.     Bất đẳng thức trong tam giác

   Tam giác là hình đơn giản nhất trong các đa giác, mỗi đa giác bất kì
đều có thể chia thành các tam giác và sử dụng tính chất của nó. Vì vậy,
nghiên cứu các bất đẳng thức trong tam giác sẽ hữu ích trong việc giải
quyết các bất đẳng thức trong đa giác. Trước hết, chúng ta nghiên cứu
các bất đẳng thức cơ bản sau đây:

1.3.1.    Bất đẳng thức về độ dài các cạnh
Định lý 1.18. Cho hai đường tròn có bán kính lần lượt là R và R
(R ≥ R ), khoảng cách giữa tâm của chúng bằng d. Điều kiện cần và đủ
để hai đường tròn đó cắt nhau là R − R ≤ d ≤ R + R .

Chứng minh.




                  Hình 1.1 Hai đường tròn không cắt nhau.

   Rõ ràng nếu hai đường tròn ở ngoài nhau (hình 1.1 A) thì ta có
R + R < d. Nếu hai đường tròn chứa nhau (hình 1.1 B) thì ta cũng có
ngay d < R − R . Nếu hai đường tròn cắt nhau tại một điểm M thì theo
bất đẳng thức về ba cạnh của tam giác OO M , với O và O lần lượt là
tâm của đường tròn bán kính R và R , ta có R − R ≤ d ≤ R + R .
   Đảo lại, nếu R − R ≤ d ≤ R + R thì hai đường tròn đã cho không
thể ngoài nhau hoặc chứa nhau được (nếu không phải có R + R < d
hoặc d < R − R ). Do đó chúng chỉ có thể cắt nhau.


          www.MATHVN.com - HOANG NGOC QUANG, Yen Bai
12

Định lý 1.19. Các số dương a, b, c là độ dài 3 cạnh của một tam giác
khi và chỉ khi a + b > c, b + c > a, c + a > b.
Chứng minh. Nếu a, b, c là độ dài 3 cạnh của tam giác thì theo bất đẳng
thức về 3 cạnh của tam giác ta có a + b > c, b + c > a, c + a > b.
    Ngược lại, nếu có a, b và c là 3 số thực dương thỏa mãn a+b > c, b+c >
a, c + a > b, thì ta có thể chọn hai điểm A và B trên mặt phẳng cách
nhau một khoảng c. Lấy A và B làm tâm dựng hai đường tròn bán kính
tương ứng là a và b. Từ các bất đẳng thức a + b > c, b + c > a, c + a > b
ta có |a − b| < c < a + b. Theo định lý 1.18 thì hai đường tròn tâm A
và B phải cắt nhau tại một điểm C. Vậy a, b, c là độ dài các cạnh của
tam giác ABC theo cách dựng trên.
Định lý 1.20. Cho trước tam giác ABC và một điểm M ở trong tam
giác. Khi đó ta có M B + M C < AB + AC.
Chứng minh.
   Kéo dài BM về phía M cắt
cạnh AC tại điểm N . Theo định
lý 1.19 ta có
 MB + MC < MB + MN + NC
=BN + N C < AB + AN + N C
=AB + AC.
                                                  Hình 1.2

Bài toán 1.1. Cho M là một điểm nằm trong tam giác ABC. Chứng
minh rằng
                      p < M A + M B + M C < 2p.
Trong đó p là nửa chu vi của tam giác ABC.
Giải. Áp dụng định lý 1.19 cho các tam giác M AB, M BC và M CA ta có
AB < M A+M B, BC < M B +M C, CA < M C +M A. Cộng theo vế ba
bất đẳng thức trên rồi chia cả hai vế cho 2 ta được p < M A+M B +M C.
   Mặt khác, theo định lý 1.20 ta có M A+M B < CA+CB, M B+M C <
AB + AC, M C + M A < BC + BA. Cộng theo vế ba bất đẳng thức trên
và đem chia cả hai vế cho 2 ta được AM + BM + CM < 2p.


        www.MATHVN.com - HOANG NGOC QUANG, Yen Bai
13

Định lý 1.21. Trong một tam giác ứng với góc lớn hơn là cạnh dài hơn
và ngược lại.

Chứng minh. Xét tam giác ABC. Ta chứng minh nếu ABC > ACB thì
AC > AB và ngược lại.
                               Thật vậy, trong góc ABC ta kẻ tia Bx
                            tạo với cạnh BC góc bằng góc ACB. Do
                            ABC > ACB, nên Bx cắt cạnh AC tại
                            điểm D và tạo thành tam giác cân DBC,
                            do đó DB = DC. Mặt khác, trong tam
                            giác ABD ta có AD + DB > AB. Do đó
          Hình 1.3          AC = AD + DC = AD + DB > AB.
   Phần ngược lại của định lý là hiển nhiên. Vì nếu ABC < ACB thì
ta phải có AC < AB là điều vô lí.

Bài toán 1.2. Chứng minh rằng đường vuông góc AH hạ từ điểm A
xuống đường thẳng d cho trước luôn nhỏ hơn đường xiên AB.

Giải. Tam giác AHB là tam giác vuông tại H, do đó AHB = 900 >
ABH. Theo định lý trên, ta có AB > AH.
   Sự tương ứng giữa độ lớn cạnh và góc còn đúng cho cạnh của hai tam
giác khác nhau. Dùng định lý 1.21 ta dễ dàng chứng minh kết quả sau.

Định lý 1.22. Cho trước hai tam giác ABC và A B C có hai cặp cạnh
bằng nhau AB = A B và AC = A C . Ta có bất đẳng thức BAC >
B A C khi và chỉ khi BC > B C .

Chứng minh.
   Trước hết, giả sử rằng BAC >
B A C , ta sẽ chứng minh BC > B C .
Không mất tính tổng quát giả sử
AB ≥ AC. Ta đem hình tam giác
ABC đặt chồng lên hình tam giác
A B C sao cho A ≡ A , C ≡ C
và đỉnh B, B nằm cùng phía so với                Hình 1.4

đường thẳng đi qua AC.


       www.MATHVN.com - HOANG NGOC QUANG, Yen Bai
14

   Do AB = A B , nên ta có ABB = AB B. Vì CBB < ABB và
CB B > AB B, nên ta có CBB < CB B. Theo định lý 1.21, ta có
CB > CB , hay là CB > C B .
   Nếu như BAC = B A C thì ta cũng dễ dàng thấy rằng BC = B C ,
do ∆ABC và ∆A B C (c.g.c). Vậy ta có BAC > B A C khi và chỉ khi
BC > B C .
Bài toán 1.3. Cho tam giác ABC và AM là trung tuyến. Chứng minh
                                   1
rằng BAC ≥ 900 khi và chỉ khi AM ≤ 2 BC.
Giải. Gọi A là điểm đối xứng với A qua trung điểm M của cạnh BC.
Tứ giác ABA C là tứ giác có hai đường chéo cắt nhau tại trung điểm
của mỗi đường nên ABA C là hình bình hành.
   Xét hai tam giác ABA và ABC có
cạnh AB là cạnh chung và có cặp cạnh
A B và AC bằng nhau. Theo định lý
1.22, ta có BC ≥ AA khi và chỉ khi
BAC ≥ ABA . Do BAC + ABA =
1800 , cho nên BAC ≥ ABA khi và
chỉ khi BAC ≥ 900 . Tóm lại, AM =
1        1
2 AA ≤ 2 BC khi và chỉ khi BAC ≥
                                                Hình 1.5
900 .
Định lý 1.23. Trong những đường xiên nối một điểm M cho trước với
điểm N trên một đường thẳng d cho trước, đường xiên nào có hình chiếu
dài hơn thì dài hơn.

1.3.2.   Bất đẳng thức về các đại lượng đặc biệt
   Trong một tam giác, mối quan hệ giữa các cạnh dẫn đến mối quan hệ
với các đại lượng đặc biệt. Với đường cao ta dễ thấy là đường cao tương
ứng với cạnh lớn hơn thì ngắn hơn. Đối với đường trung tuyến và đường
phân giác ta cũng sẽ chứng minh rằng ứng với cạnh dài hơn là đường
trung tuyến và đường phân giác ngắn hơn.
Định lý 1.24. Trong tam giác ABC ứng với cạnh dài hơn là đường cao,
đường trung tuyến và đường phân giác ngắn hơn.


         www.MATHVN.com - HOANG NGOC QUANG, Yen Bai
15

Chứng minh. Giả sử c < b, ta sẽ chứng minh rằng hb < hc , mb < mc và
lb < lc .
                             S      S
    Vì c < b nên suy ra hb = 2b < 2c = hc .
    Để chứng minh mb < mc , ta gọi M, N và
P là trung điểm của các cạnh AB, AC và
BC, tương ứng (hình 1.6). Áp dụng định
lý 1.22 cho ∆P AB và ∆P AC là hai tam
giác có hai cặp cạnh bằng nhau (AP chung
và BP = CP ), ta có AP B < AP C. Gọi G
                                                       Hình 1.6
là trọng tâm của tam giác ABC. Xét hai
tam giác GP B và GP C là hai tam giác có hai cặp cạnh bằng nhau
(GP chung và P B = P C). Do có AP B < AP C, nên BG < CG. Vậy
        3      3
mb = BG < CG = mc .
        2      2
                                  Gọi phân giác của góc B là BL và phân
                              giác xuất phát từ C là CK. Theo định lý
                              đường phân giác ta có LC = a ⇒ CL =
                                                      LA      c
                               ab
                              a+c . Tương tự KB = a ⇒ BK = a+b .
                                              KA    b
                                                                ac

                                  Do c < b, nên BK < CL. Dựng hình
                              bình hành BKCT (hình 1.7), ta có BT C =
                              BKC = A + C và ta có BT C < BLC.
                                            2
                                  Mặt khác, vì T C = BK, và BK < CL
                              nên T C < CL. Trong tam giác T LC, ứng
           Hình 1.7           với cạnh lớn hơn là góc lớn hơn theo định
lí 1.21, cho nên CLT < CT L. Từ các bất đẳng thức BLC > BT C và
CLT < CT L, ta có BLT < BT L. Theo định lý 1.21 ta có BT > BL
mà CK = BT suy ra CK > BL.
Định lý 1.25. Trong tam giác ABC ta luôn có
                            ma ≥ la ≥ ha .
Chứng minh. Gọi H là chân đường cao, L là chân đường phân giác và M
là chân đường trung tuyến xuất phát từ đỉnh A. Ta chứng minh rằng L
nằm trên đoạn thẳng nối HM , và áp dụng định lý 1.23 để có bất đẳng
thức cần chứng minh.


        www.MATHVN.com - HOANG NGOC QUANG, Yen Bai
16

   Định lí hiển nhiên đúng cho trường hợp
tam giác ABC cân tại đỉnh A. Để tiện
chứng minh trong trường hợp tam giác
không cân tại A, không mất tính tổng quát
ta giả sử AB < AC. Gọi A đối xứng với A
qua M , ta có BACA là hình bình hành.
   Trong tam giác AA C ta có AC >
A C = AB và do đó theo định lý 1.21 ta
                                                Hình 1.8
có BAM = M A C > CAM và đó điểm L
nằm trong góc BAM . Mặt khác, do BAH phụ với góc B và CAH phụ
với góc C, cho nên BAH < CAH. Do đó L phải nằm trong góc CAH.
   Tóm lại, điểm L nằm giữa điểm H và điểm M và ta có HM > HL.
Theo định lý 1.23, ta có AH < AL < AM .

Định lý 1.26. Đường trung tuyến AM của tam giác ABC nhỏ hơn nửa
tổng các cạnh AB và AC cùng xuất phát từ một đỉnh A.

Chứng minh. Gọi A là điểm đối xứng với điểm A qua điểm M , ta có
                                        1       1
ABA C là hình bình hành. Do đó AM = AA < (A C + AC) =
                                        2       2
1
  (AB + AC).
2
Bài toán 1.4. Chứng minh rằng nếu M là điểm nằm trên đường phân
giác ngoài của góc C của tam giác ABC (M khác C) thì M A + M B >
CA + CB.

Giải.
                                       Giả sử A là điểm đối xứng
                                    với điểm A qua đường phân giác
                                    ngoài của góc C. Khi đó các điểm
                                    A , C, B thẳng hàng và M A =
                                    M A. Do đó M A + M B = M A +
                                    M B > A B = CA + CB =
                                    CA + CB.


             Hình 1.9

        www.MATHVN.com - HOANG NGOC QUANG, Yen Bai
17

1.4.   Các bất đẳng thức sinh ra từ các công thức hình học

Định lý 1.27. (Công thức Euler) Gọi R và r lần lượt là bán kính của
đường tròn ngoại tiếp và đường tròn nội tiếp tam giác ABC, d là khoảng
cách giữa tâm hai đường tròn đó. Ta có

                            d2 = R2 − 2Rr.                        (1.34)

Chứng minh.
   Gọi O, I lần lượt là tâm đường
tròn ngoại tiếp, nội tiếp ∆ABC.
Biết rằng đường tròn ngoại tiếp
tam giác BCI có tâm D là trung
điểm của cung BC. Gọi M là
trung điểm của BC và Q là hình
chiếu của I trên OD. Khi đó

 OB 2 − OI 2 =OB 2 − DB 2 + DI 2 − OI 2
             =OM 2 − M D2 + DQ2 − QO2            Hình 1.10
             = (M O + DM ) (M O − DM ) + (DQ + QO) (DQ − QO)
             =DO (M O − DM + DQ + OQ) = R (2M Q) = 2Rr.

Vậy OI 2 = R2 − 2Rr, nghĩa là d2 = R2 − 2Rr.

Hệ quả 1.4. (Bất đẳng thức Euler) Kí hiệu R, r lần lượt là bán kính
đường tròn ngoại tiếp và bán kính đường tròn nội tiếp tam giác ABC.
Khi đó

                                R ≥ 2r.                           (1.35)

Đẳng thức xảy ra khi và khi tam giác ABC đều.

Bài toán 1.5. Cho tam giác ABC với R là bán kính đường tròn ngoại
tiếp, r là bán kính đường tròn nội tiếp và p là nửa chu vi tam giác ABC.
                         p     R
Chứng minh rằng r ≤ √ ≤ .
                       3 3      2



        www.MATHVN.com - HOANG NGOC QUANG, Yen Bai
18
                                                         √        √
Giải. Ta có [ABC] = abc = pr, suy ra 2p = a+b+c ≥ 3 3 abc = 3 3 4Rrp.
                       4R                                 √
Do đó 8p ≥ 27(4Rrp) ≥ 27(8r2 p), vì R ≥ 2r. Vậy p ≥ 3 3r.
         3
                              p
                                                                    √
   Bất đẳng thức thứ hai, 3√3 ≤ R tương đương với a + b + c ≤ 3 3R.
                                     2
Sử dụng định lí hàm số sin, bất đẳng thức này tương đương với sin A +
                  √
                 3 3
sin B + sin C ≤ 2 . Bất đẳng thức này đúng vì hàm số f (x) = sin √ làx
hàm lồi trên (0, π), do đó sin A+sin B+sin C
                                   3         ≤ sin A+B+C
                                                     3   = sin 600 = 23 .
Định lý 1.28. (Công thức Leibniz) Cho tam giác ABC với độ dài các
cạnh là a, b, c. Gọi G là trọng tâm và (O, R) là đường tròn ngoại tiếp
tam giác. Khi đó
                                      1 2
                      OG2 = R2 −         a + b 2 + c2 .           (1.36)
                                      9
Chứng minh.
   Để chứng minh bài toán này ta sử dụng
định lí Stewart "Nếu L là điểm nằm trên
cạnh BC của ∆ABC và nếu AL = l, BL =
m, LC = n, thì a(l2 + mn) = b2 m + c2 n".
Áp dụng định lí Stewart cho ∆OAA ,
trong đó A là trung điểm của BC, ta
được AA (OG2 + AG.GA ) = A O2 .AG +
AO2 .GA . Vì AO = R, AG = 2 AA , GA =
                                 3
1              2   2    2     2     2   1 2
3 AA nên OG + 9 A A = 3 A O + 3 R .                     Hình 1.11
                   2  2(b2 +c2 )−a2       2      2   a2
Mặt khác, vì A A =          4       và A O = R − 4 , ta được
            2       a2 2 1 2 2 2(b2 + c2 ) − a2
                    2
         OG = R −       + R −
                    4 3 3          9         4
                  a2 2(b2 + c2 ) − a2        a2 + b2 + c2
            =R2 −    −                = R2 −              .
                  6        18                     9

Hệ quả 1.5. (Bất đẳng thức Leibniz) Cho tam giác ABC với độ dài
các cạnh là a, b, c. (O, R) là đường tròn ngoại tiếp tam giác. Ta có bất
đẳng thức sau
                          9R2 ≥ a2 + b2 + c2 .                     (1.37)
Đẳng thức xảy ra khi và chỉ khi O là trọng tâm của tam giác ABC.


        www.MATHVN.com - HOANG NGOC QUANG, Yen Bai
19

Bài toán 1.6. Cho tam giác ABC với độ dài các cạnh là a, b, c. Chứng
           √           9abc
minh rằng 4 3 [ABC] ≤ a+b+c .

Giải. Sử dụng bất đẳng thức Leibniz với lưu ý 4R. [ABC] = abc ta có
                         a2 b2 c2 2  2 2
9R2 ≥ a2 + b2 + c2 ⇔ 16[ABC]2 ≥ a +b +c ⇔ 4 [ABC] ≤ √a23abc+c2 . Mặt
                                    9                      2
                                                    √ √ +b
khác, Bất đẳng thức Cauchy-Schwarz cho a + b + c ≤ 3 a2 + b2 + c2 .
        √            9abc
Do đó 4 3 [ABC] ≤ a+b+c .

Bài toán 1.7. Giả sử đường tròn nội tiếp tam giác ABC tiếp xúc các
cạnh AB, BC, CA tại D, E, F , tương ứng.Kí hiệu p là nửa chu vi của
                                                      2
tam giác ABC. Chứng minh rằng EF 2 + F D2 + DE 2 ≤ p3 .

Giải. Thấy rằng đường tròn nội tiếp tam giác ABC là đường tròn ngoại
tiếp tam giác DEF . Áp dụng bất đẳng thức Leibniz cho tam giác DEF ,
ta được EF 2 + F D2 + DE 2 ≤ 9r2 . Mặt khác, theo bài toán 1.5 ta có
                                         2
p2 ≥ 27r2 . Do đó EF 2 + F D2 + DE 2 ≤ p3 .

Định lý 1.29. (Định lí Euler) Cho tam giác ABC nội tiếp trong đường
tròn tâm O, bán kính R. M là một điểm bất kì nằm trong mặt phẳng
tam giác. Gọi X, Y, Z lần lượt là hình chiếu vuông góc của M lên các
cạnh BC, CA, AB. Khi đó diện tích của tam giác XY Z được tính theo
diện tích tam giác ABC và khoảng cách M O bởi công thức sau
                                1    M O2
                     [XY Z] =     1−      [ABC] .               (1.38)
                                4     R2

Chứng minh.
   Kéo dài AM, BM, CM cắt đường tròn
ngoại tiếp tại các điểm X , Y , Z tương
ứng. Ta có ZXM = M BZ (tứ giác
BZM X nội tiếp), M BZ = ABY (B, Z, A
thẳng hàng và B, M, Y thẳng hàng),
ABY     = AX Y (cùng chắn cung AY ).
Từ đó   suy ra ZXM = AX Y . Tương tự
Y XM    = AX Z . Từ đó suy ra ZXY =
ZXY     . Ta sẽ kí hiệu hai góc này là X và
                                                    Hình 1.12


        www.MATHVN.com - HOANG NGOC QUANG, Yen Bai
20

X . Ta có
           1
 [XY Z] = XY.XZ. sin X
           2
 1
= M C. sin C.M B sin B. sin X (định lí hàm số sin)
 2
 1            MC
= M B.M Y .       sin B. sin C. sin X
 2            MY
 1               BC
= M O 2 − R2        . sin B sin C. sin X(phương tích, ∆M BC ∼ ∆M Z Y )
 2              ZY
 1                                sin X
= M O2 − R2 .BC. sin C. sin B.
 2                                 Y Z
            2
 1     MO                                  AC sin X       1
= 1−           AC.BC. sin C (vì sin B =       ,        =    )
 8       R2                                 2R Y Z       2R
 1     M O2
= 1−           [ABC] .
 4       R2


Chú ý 1.1.

   1) Tam giác XY Z nêu trong định lí được gọi là tam giác Pedal.
   2) Nếu M nằm trên đường tròn ngoại tiếp tam giác ABC thì [XY Z] =
0. Điều đó có nghĩa là tam giác XY Z suy biến thành đường thẳng, đó
chính là đường thẳng Euler.
   3) Nếu M ≡ I (I là tâm đường tròn nội tiếp tam giác ABC) thì XY Z
là tam giác nội tiếp đường tròn tâm I, bán kính r có các góc X, Y, Z
tương ứng bằng π − A , π − B , π − C . Bằng các phép biến đổi sơ cấp từ
                 2   2 2    2 2    2
công thức (1.38) sẽ suy ra công thức Euler OI 2 = R2 − 2Rr.

Hệ quả 1.6. Cho tam giác ABC và một điểm M bất kì nằm trong mặt
phẳng tam giác. Gọi X, Y, Z lần lượt là hình chiếu vuông góc của M lên
các cạnh BC, CA, AB. Khi đó
                                    1
                         [XY Z] ≤     [ABC] .                    (1.39)
                                    4
Đẳng thức xảy ra khi và chỉ khi M là tâm của đường tròn ngoại tiếp
tam giác ABC.



        www.MATHVN.com - HOANG NGOC QUANG, Yen Bai
21

Bài toán 1.8. Cho tứ giác lồi ABCD nội tiếp trong đường tròn tâm O
(với O nằm bên trong tứ giác). Gọi M N P Q là tứ giác mà các đỉnh lần
lượt là hình chiếu của giao điểm 2 đường chéo của tứ giác ABCD đến
các cạnh AB, BC, CD, DA. Chứng minh rằng
                                    1
                        [M N P Q] ≤ [ABCD] .
                                    2
Giải. Gọi K là giao điểm 2 đường chéo AC và BD của tứ giác ABCD.
   Dễ thấy KM N là tam giác Pedal dựng
từ điểm K của tam giác ABC. Do đó
áp dụng hệ quả 1.6 ta được [KM N ] ≤
1
4 [ABC]. Làm tương tự cho các tam giác
KN P, KP Q, KQM và cộng các kết quả lại
  [KM N ] + [KN P ] + [KP Q] + [KQM ]
  1
≤ ([ABC] + [BCD] + [CDA] + [DAB]) .
  4
                   1
Suy ra [M N P Q] ≤ 2 [ABCD].                      Hình 1.13
Đẳng thức xảy ra khi và chỉ khi K là tâm của đường tròn ngoại tiếp tứ
giác ABCD.
Bài toán 1.9. (Balkan, 1999) Cho ABC là một tam giác nhọn và
L, M, N là các chân đường cao hạ từ trọng tâm G của ∆ABC tới các
cạnh BC, CA, AB, tương ứng. Chứng minh rằng
                         4    [LM N ] 1
                            <         ≤ .
                        27     [ABC]    4
Giải. Ta có tam giác LM N là tam giác Pedal dựng từ trọng tâm
G của tam giác ABC. Áp dụng định lí 1.12, ta có [LM N ] =
           2        2
1                     −OG2
4 1 − OG [ABC] = R 4R2 [ABC] (vì G nằm trong tam giác ABC).
       R 2

                      1
+ Dễ thấy [LM N ] ≤ 4 [ABC]. Đẳng đẳng thức xảy ra khi và chỉ khi
G ≡ O hay khi và chỉ khi tam giác ABC đều.
+ Ta chứng minh bất đẳng thức còn lại. Thật vậy, để ý rằng OG = 1 OH.
                                                                3
Vì tam giác nhọn, H nằm trong tam giác và OH ≤ R nên
            [LM N ] R2 − 1 OH 2
                         9        R2 − 1 R2
                                       9     2  4
                   =            ≥           = > .
             [ABC]     4R2          4R2      9 27


       www.MATHVN.com - HOANG NGOC QUANG, Yen Bai
22

Định lý 1.30. (Công thức hình bình hành) Cho tứ giác ABCD, gọi x
là khoảng cách giữa trung điểm của hai đường chéo AC và BD. Ta có

         AB 2 + BC 2 + CD2 + DA2 = AC 2 + BD2 + 4x2 .                             (1.40)

Chứng minh. Gọi M, N lần lượt là trung điểm của AC và BD. Áp dụng
công thức đường trung tuyến ta có
                                AB 2 +DA2       BD2       BC 2 +CD2       BD2
  2  N A2 + N C 2 AC 2               2      −    4    +        2      −    4      AC 2
 x =             −     =                                                        −
          2        4                                  2                            4
hay AB 2 + BC 2 + CD2 + DA2 = AC 2 + BD2 + 4x2 .

Hệ quả 1.7. (Bất đẳng thức hình bình hành) Cho tứ giác ABCD. Ta
có

               AB 2 + BC 2 + CD2 + DA2 ≥ AC 2 + BD2 .                             (1.41)

Đẳng thức xảy ra khi và chỉ khi tứ giác ABCD là hình bình hành.

Bài toán 1.10. (Địa trung Hải, 2000) Cho P, Q, R, S là trung điểm của
các cạnh BC, CD, DA, AB, tương ứng, của tứ giác lồi ABCD. Chứng
minh rằng

   4 AP 2 + BQ2 + CR2 + DS 2 ≤ 5 AB 2 + BC 2 + CD2 + DA2 .

Giải. Ta biết công thức đường trung tuyến XM của tam giác XY Z
là XM 2 = 1 XY 2 + 1 XZ 2 − 1 Y Z 2 . Ta thay bộ (X, Y, Z, M ) bằng
               2         2         4
(A, B, C, P ), (B, C, D, Q), (C, D, A, R), (D, A, B, S) vào trong công thức
này và cộng 4 công thức lại với nhau để thu được công thức thứ 5. Nhân
cả hai vế của công thức thứ 5 với 4, ta được
     4 AP 2 + BQ2 + CR2 + DS 2 = AB 2 + BC 2 + CD2 + DA2 + 4 AC 2 + BC 2 .

Do đó ta chỉ cần chứng minh bất đẳng thức

               AC 2 + BC 2 ≤ AB 2 + BC 2 + CD2 + DA2 .

Đây là bất đẳng thức hình bình hành. Dấu "=" xảy ra khi và chỉ khi tứ
giác ABCD là hình bình hành. Điều phải chứng minh.


        www.MATHVN.com - HOANG NGOC QUANG, Yen Bai
23

1.5.     Bất đẳng thức trong các tam giác đặc biệt

1.5.1.    Các bất đẳng thức trong tam giác đều
   Tam giác đều có một số tính chất đặc biệt, nói chung không còn đúng
trong một tam giác tùy ý. Trong mục này, ta chỉ nghiên cứu một số bất
đẳng thức trong tam giác đều ABC liên quan mối quan hệ giữa pa , pb , pc
với P A, P B, P C (trong đó pa , pb , pc lần lượt là khoảng cách từ P đến
các cạnh BC, CA, AB).

Bài toán 1.11. Cho ABC là tam giác đều cạnh a, gọi P là một điểm
nằm trong tam giác. Chứng minh rằng
                           √
         1     1    1     6 3
   i)       + + ≥              ,
         pa pb p c          a              √
             1         1          1       3 3
   ii)           +          +           ≥      .
          pa + pb pb + pc pc + pa           a
Giải. Gọi D, E, F là chân đường vuông
góc của P lên các cạnh BC, CA, AB tương
ứng. Ta có [ABC] = [BCP ] + [CAP ] +
[ABP ], do đó ah = apa +√ b + apc . Vì
      √                          ap
        3                         3
h=        a nên pa + pb + pc =      a. Áp dụng
       2                         2
bất đẳng thức (1.2), ta được                              Hình 1.14
                                                       √
                     1     1     1          9         6 3
                        + + ≥                       =     .
                    p a p b pc         pa + pb + pc    a
Lại áp dụng bất đẳng thức (1.2) có                           √
   1        1        1                    9                 3 3
       +         +        ≥                               =     .
pa + pb pb + pc pc + pa      pa + pb + pb + pc + p c + pa    a
Bài toán 1.12. Cho tam giác đều ABC cạnh a và P là một điểm tùy
ý nằm trong tam giác. Chứng minh rằng
                                          8a3
                     P A2 .P B 2 .P C 2 ≥ √ pa pb pc .              (1.42)
                                         3 3
  Trước hết ta chứng minh bổ đề sau


          www.MATHVN.com - HOANG NGOC QUANG, Yen Bai
24

Bổ đề 1.1. Cho P là một điểm tùy ý nằm trong tam giác đều ABC
cạnh a. Kí hiệu α = BP C, β = CP A,γ = AP B, ta có đẳng thức
                                  2      2     2   a3 p a p b p c
                               P A .P B .P C =                    .   (1.43)
                                               sin α sin β sin γ
Chứng minh. Viết lại diện tích tam giác BP C theo hai cách ta được
BP.CP. sin α = a.pa , tương tự CP.AP. sin β = a.pb , AP.BP. sin γ = a.pc .
Nhân theo vế 3 đẳng thức này, ta thu được đẳng thức (1.43).

Giải. Gọi f (x) = ln(sin x), x ∈ (0, π) .Vì f (x) = − sin12 x < 0, nên f là
hàm lồi. Áp dụng bất đẳng thức Jensen, ta có
           ln(sin α) + ln(sin β) + ln(sin γ)          α+β+γ
                                             ≤ ln(sin       ).
                           3                            3
                                          √
Suy ra sin α. sin β. sin γ ≤ 3 8 3 . Thay bất đẳng thức này vào (1.43) ta
được bất đẳng thức (1.42).
Bài toán 1.13. Cho tam giác đều ABC cạnh a và P là một điểm tùy
ý nằm trong tam giác. Chứng minh rằng

                                      P A.P B.P C ≥ 8pa pb pc .       (1.44)
                                                  1      1       1
Giải. Vì [BP C] + [CP A] + [AP B] = [ABC] nên apa + apb + apc =
   √                           √                  2      2       2
 2                                                                   √
a 3                           a 3                              3   3
      . Suy ra pa + pb + pc =     . Do đó pa pb pc ≤ pa +pb +pc = a 72 3 .
                                                         3
   4                           2
Thay bất đẳng thức này vào (1.42), ta được bất đẳng thức (1.44).
Bài toán 1.14. Cho tam giác đều ABC cạnh a và P là một điểm tùy
ý nằm trong tam giác. Chứng minh rằng

                           P A.P B + P B.P C + P C.P A ≥ a2 .         (1.45)
           α       β       γ
Giải. Vì   2   +   2   +   2   = 1800 nên
                                                          3
                                   cos α + cos β + cos γ ≥ ,          (1.46)
                                                          2
dấu đẳng thức xảy ra khi và chỉ khi α = β = γ = 1200 .
Bây giờ, áp dụng định lí hàm số cosin cho tam giác P AB, ta có a2 =


        www.MATHVN.com - HOANG NGOC QUANG, Yen Bai
25

                                                      2     2   2
                                               +P B −a
P A2 + P B 2 − 2P A.P B. cos γ hay cos γ = P A2P A.P B . Tương tự cos α =
P B 2 +P C 2 −a2              2
                                +P A2 −a2
   2P B.P C      , cos β = P C2P C.P A .
Cộng theo vế 3 bất đẳng thức trên và sử dụng (1.46), ta được
  P A2 + P B 2 − a2 P B 2 + P C 2 − a2 P C 2 + P A2 − a2 3
                   +                  +                 + ≥ 0,
      2P A.P B          2P B.P C           2P C.P A      2
Do đó
     P A2 .P B + P B 2 .P A + P A.P B.P C + P C 2 .P B + P B 2 .P C + P A.P B.P C +
               + P A2 .P C + P C 2 .P A + P A.P B.P C − a2 (P A + P B + P C) ≥ 0,

tương đương với
      (P A + P B + P C) P A.P B + P B.P C + P C.P A − a2 ≥ 0.
Từ đó có bất đẳng thức P A.P B + P B.P C + P C.P A ≥ a2 .
Dấu đẳng thức xảy ra khi và chỉ khi P ≡ O.
Tam giác Pompeiu

  Năm 1936, nhà toán học Rumani, Dimitrie Pompeiu phát hiện kết
quả đơn giản nhưng đẹp sau đây trong hình học phẳng Euclide
Định lý 1.31. (Định lý Pompeiu). Cho P là một điểm tùy ý nằm
trong mặt phẳng chứa tam giác đều ABC. Khi đó các khoảng cách
P A, P B, P C là độ dài các cạnh của một tam giác. Tam giác này suy
biến nếu điểm P nằm trên đường tròn ngoại tiếp tam giác ABC.
Chứng minh.
   Xét trường hợp điểm P không nằm trên
đường tròn ngoại tiếp tam giác ABC. Áp
dụng bất đẳng thức Ptolemy (Xem định lí
2.2) cho 4 điểm A, B, P, C ta có
      P A.BC < P C.AB + P B.AC,
      P B.AC < P A.BC + P C.AB,
      P C.AB < P A.BC + P B.AC.
Vì tam giác ABC đều nên AB = BC =                  Hình 1.15
CA = l, do đó P A < P C + P B, P B < P A + P C, P C < P A + P B. Vậy
P A, P B, P C là độ dài 3 cạnh của một tam giác.


        www.MATHVN.com - HOANG NGOC QUANG, Yen Bai
26

Chú ý 1.2.
   + Tam giác với độ dài các cạnh bằng P A, P B, P C được gọi là tam
giác Pompeiu.
   + Khi P nằm ở trong tam giác ABC, tam giác Pompeiu có thể được
xây dựng một cách dễ dàng như sau: Quay tam giác ABP quanh tâm A,
một góc 600 , được tam giác AB C. Khi đó AP = AB = P B , BP = CB ,
tam giác Pompeiu sẽ là ∆P CB .
Bài toán 1.15. Cho tam giác đều ABC cạnh a và P là một điểm tùy
ý nằm trong tam giác. Chứng minh rằng
                     √                      √
                 3Rp 3 ≥ P A + P B + P C ≥ a 3.           (1.47)
Trong đó Rp là bán kính đường tròn ngoại tiếp tam giác Pompeiu có độ
dài các cạnh bằng P A, P B, P C.
Giải. Gọi M là trung điểm của BC, theo bất đẳng thức tam giác AP +
P M ≥ AM , mặt khác lại có P M ≤ P B+P C . Suy ra 2P A + P B + P C ≥
 √                                   √ 2                    √
a 3, tương tự 2P B + P C + P A ≥ a 3, 2P C + P A + P B ≥ a 3. Cộng
                                                          √
theo vế 3 bất đẳng thức này ta được P A + P B + P C ≥ a 3.
Áp dụng P A2 + P B 2 + P C 2 ≤ 9Rp và bất đẳng thức Cauchy-Schwarz,
                                   2

ta có 9Rp ≥ P A2 + P B 2 + P C 2 ≥ 3 (P A + P B + P C)2 .
         2                         1
           √
Suy ra 3Rp 3 ≥ P A + P B + P C.
Bài toán 1.16. Cho P là một điểm nằm trong tam giác đều ABC cạnh
a. Gọi T là diện tích của tam giác Pompeiu với độ dài các cạnh bằng
P A, P B, P C. Khi đó
                             √
                               3 2
                         T =      a − 3d2 .                   (1.48)
                              12
Trong đó d = OP (O là tâm của tam giác đều ABC).
Giải.
   Ta thực hiện 3 phép quay tương tự như hình 1.15, đó là quay tam
giác AP B quanh tâm A một góc 600 , quay tam giác BP C quanh tâm
B một góc 600 và quay tam giác CP A quanh tâm C một góc 600 . Ta
sẽ được một hình lục giác AB CA BC , trong đó các tam giác Pompeiu
P BA , P AC , P CB có cùng diện tích T .


       www.MATHVN.com - HOANG NGOC QUANG, Yen Bai
27

   Vì ∆AP C = ∆BA C, ∆AP B =
∆AB C, ∆AC B = ∆BP C nên √ tích diện
                              2
                            2a 3
của lục giác = 2. [ABC] =          . Mặt
                              4
khác ∆AP B , BP C , CP A là các tam giác
đều. Do đó
   √               √       √           √
2a2 3         P A2 3 P B 2 3 P C 2 3
       = 3T +        +        +
  4              4       4           4
Luôn có

P A2 +P B 2 +P C 2 = 3OP 2 +OA2 +OB 2 +OC 2
                                    √
                                  a2 3                Hình 1.16
Vì OP = d, OA = OB = OC =               nên
                                    3
P A2 + P B 2 + P C 2 = 3d2 + a2 . Từ đó ta được công thức (1.48).
Nhận xét 1.1. Từ công thức (1.47) và (1.48) ta có các hệ quả sau:
        √
            3 2
1) T ≤       a . Đẳng thức xảy ra khi và chỉ khi P ≡ O.
         12
                R
2) rp ≤ a ≤ 2p , trong đó Rp , rp lần lượt là bán kính vòng tròn ngoại
          6
tiếp, nội tiếp tam giác Pompeiu với độ dài các cạnh bằng P A, P B, P C.

1.5.2.    Các bất đẳng thức trong tam giác vuông và tam giác
          cân
Bài toán 1.17. Cho góc vuông xAy. B là điểm trên tia Ax, C là điểm
                                                 √
trên tia Ay (B = A; C = A). Chứng minh rằng AB + 3AC ≤ 2BC.
Giải.
                                Trong góc xAy vẽ tia Az sao cho
                             xAz = 300 , do đó yAz = 600 . Vẽ
                             BH⊥Az, CK⊥Az(H, K ∈ Az), Az cắt
                             BC tại I. Xét ∆ABH có AHB =
                             900 , BAH = 300 nên là nửa tam giác đều,
                             cạnh AB. Suy ra BH = 1 AB mà BH ≤
                                                      2
                             BI. Do đó
           Hình 1.17
                              AB ≤ 2BI.                             (1.49)


          www.MATHVN.com - HOANG NGOC QUANG, Yen Bai
28

Xét ∆ACK có AKC = 900 , CAK = 600 nên là nửa tam giác đều, cạnh
                     √
AC. Suy ra CK = AC2 3 mà CK ≤ IC. Do đó
                            √
                              3AC ≤ 2IC.                   (1.50)
                                       √
Cộng theo vế (1.49) và (1.50) được AB + 3AC ≤ (BI + CI) = 2BC.

Bài toán 1.18. Cho tam giác ABC cân tại A. D là điểm trên cạnh BC,
E là điểm trên trên tia đối của tia CB sao cho CE = BD. Chứng minh
rằng AD + AE > 2AB.

Giải.
   Trên tia đối của tia BC lấy điểm
F sao cho BF = BD. Áp dụng
định lí 1.26 ta có AD + AF > 2AB.
Mặt khác, xét ∆ABF và ∆ACE có
AB = AC, ABF = ACE (vì ABC =
ACB, ABC + ABF = ACE + ACB =
1800 ), BF = CE. Do đó ∆ABF =
∆ACE (c.g.c). Suy ra AF = AE. Vậy
AD + AE > 2AB.
                                                            Hình 1.18
Bài toán 1.19. (Iran, 2005) Cho tam giác ABC vuông tại A. Gọi D là
giao điểm của phân giác trong của góc A với cạnh BC và Ia là tâm của
đường tròn bàng tiếp cạnh BC của tam giác ABC. Chứng minh rằng
AD √
     ≤ 2 − 1.
DIa
Giải.
                              Gọi E, F lần lượt là điểm tiếp xúc của
                           đường tròn bàng tiếp với các cạnh AB, AC
                           tương ứng. Ta có ra = Ia E = AF = F Ia =
                           p, trong đó p là nửa chu vi của tam giác
                           ABC. Hơn nữa, DIa = ha . Vì aha = bc,
                                             AD
                                                    r
                                                      a


                                    AD        ha       bc       bac     4Rr    1
                           ta có    DIa   =   ra   =   ap   =   4R       a2   rp   =
                            4Rr
                             a2 .   Vì 2R = a và 2r = b + c − a nên

         Hình 1.19
        www.MATHVN.com - HOANG NGOC QUANG, Yen Bai
29

AD         b+c−a       b+c
                                               √                   (b+c)2       b+c
DIa    =     a    − 1. Mặt khác a =
                   =    a                          b2 + c2 ≥         2      =   √     hay
             √     AD
                        √                                                         2
b + c ≤ a 2. Do đó DIa ≤ 2 − 1.
Bài toán 1.20. (Rumani, 2007) Cho ABC là một tam giác vuông cân
tại A. Với điểm P tùy ý nằm trong tam giác, xét đường tròn tâm A và
bán kính AP cắt các cạnh AB và AC tại M và N , tương ứng. Hãy xác
định vị trí của P để M N + BP + CP đạt giá trị nhỏ nhất.
                             Giải.
   Xét điểm Q trên đường trung trực
của BC thỏa mãn AQ = AP . Gọi S
là giao điểm của BP và tiếp tuyến với
đường tròn tại Q. Khi đó SP + P C ≥
SC. Do đó BP + P C = BS + SP +
P C ≥ BS + SC.
Mặt khác, BS + SC ≥ BQ + QC, nên
BP + P C đạt giá trị nhỏ nhất nếu
P ≡ Q.
Gọi T là trung điểm của M N . Vì
∆AM Q cân và M T là một trong
những chiều cao của nó, khi đó M T =             Hình 1.20
ZQ trong đó Z là chân đường vuông
góc hạ từ Q xuống AB. Khi đó M N + BQ + QC = 2(M T + QC) =
2(ZQ + QC) đạt giá trị nhỏ nhất khi Z, Q, C thẳng hàng và điều này có
nghĩa CZ là chiều cao. Bằng phép đối xứng, BQ cũng là chiều cao và
do đó P là trực tâm.

1.6.       Các bất đẳng thức khác trong tam giác

Bài toán 1.21. (Bất đẳng thức Weitzenbock, IMO 1961) Gọi a, b, c là
độ dài các cạnh của một tam giác. Chứng minh rằng
                                        √
                        a2 + b2 + c2 ≥ 4 3S.               (1.51)
Giải. Theo bất đẳng thức AM - GM ta có                                 √
                                           (p−a)+(p−b)+(p−c)
                                                               3        3 2
S=         p(p − a)(p − b)(p − c) ≤   p            3               =     p.
                                                                       9

            www.MATHVN.com - HOANG NGOC QUANG, Yen Bai
30

Do đó, theo bất đẳng thức Cauchy - Schwarz ta có
                       √              2
         √        √      3 a+b+c           1
        4 3S ≤ 4 3                      = (a + b + c)2
                        9      2           3
                 1 2
              ≤    a + b2 + c2 12 + 12 + 12 = a2 + b2 + c2 .
                 3
Bài toán sau cho ta một kết quả mạnh hơn bất đẳng thức Weitzenbock.


Bài toán 1.22. (Bất đẳng thức Hadwiger - Finsler) Gọi a, b, c là độ dài
các cạnh của một tam giác. Chứng minh rằng
                       √
       a2 + b2 + c2 ≥ 4 3S + (a − b)2 + (b − c)2 + (c − a)2      (1.52)

Đẳng thức xảy ra khi và chỉ khi a = b = c.
                                        1
Giải. Vì a2 = b2 + c2 − 2bccosA và S = 2 bcsinA nên ta có
                    √                             √
   a2 + b2 + c2 − 4 3S = 2(b2 + c2 ) − 2bccosA − 2 3bcsinA
                         π
 =2(b2 + c2 ) − 4bccos     − A ≥ 2(b2 + c2 − 2bc) = 2 (b − c)2 . (1.53)
                         3
 Mặt khác, không mất tính tổng quát ta giả sử b ≤ a ≤ c. Khi đó ta có
bất đẳng thức

                    (b − c)2 ≥ (a − b)2 + (c − a)2 .             (1.54)

Vì bất đẳng thức trên tương đương với bất đẳng thức (a − b)(a − c) ≤ 0.
Từ bất đẳng thức (1.53) và (1.54) ta được bất đẳng thức (1.52).
                                 cos π − A = 1
                                
                                       3
Đẳng thức xảy ra khi và chỉ khi      a=b           ⇔ a = b = c.
                                     a=c
                                

Bài toán 1.23. (Điểm Torricelli) Tìm điểm O trong tam giác ABC cho
trước sao cho tổng khoảng cách từ điểm O tới ba đỉnh của tam giác là
nhỏ nhất có thể.

Giải. Xét làm hai trường hợp:
a) Tam giác ABC có ba góc nhỏ hơn 1200 .


        www.MATHVN.com - HOANG NGOC QUANG, Yen Bai
31

   Dựng tam giác đều BCD ở phía ngoài của tam giác ABC. Gọi T là
giao điểm của đường tròn ngoại tiếp tam giác BCD với AD. Dễ chứng
minh rằng T nhìn ba cạnh của tam giác ABC dưới ba góc bằng nhau.
Ta chứng minh rằng với một điểm O tùy ý ở trong tam giác ABC khác
điểm T thì ta có OA + OB + OC ≥ T A + T B + T C.
   Điểm T được gọi là điểm
Torricelli của tam giác ABC
và có tổng các khoảng cách
tới các đỉnh của tam giác
ABC nhỏ nhất.
   Thật vậy, theo định lí
Pompeiu, ta có OB + OC ≥
OD, do đó                                   Hình 1.21

                 OA + OB + OC ≥ OA + OD ≥ AD.                      (1.55)

Mặt khác, vì T nằm trên đường tròn ngoại tiếp tam giác đều BCD nên

                 T A + T B + T C = T A + T D = AD.                 (1.56)

Từ (1.55) và (1.56) suy ra OA + OB + OC ≥ T A + T B + T C. Đẳng
thức xảy ra khi và chỉ khi O ≡ T .
b) Tam giác ABC có một góc, chẳng hạn B > 1200
                                  Dựng tam giác đều BCD ở phía ngoài
                               tam giác ABC. Do B > 1200 , cho nên với
                               điểm O tùy ý ở trong tam giác ABC, điểm
                               B nằm trong tam giác ODA. Theo định lí
                               Pompeiu, ta có OB + OC ≥ OD.
                                  Mặt khác, theo định lí 1.20 đối với tam
           Hình 1.22           giác ODA, ta có OA + OD ≥ BA + BD.
Từ đó ta có OA + OB + OC ≥ OA + OD ≥ BA + BD = BA + BC.
Như vậy, khi O ≡ B, tổng khoảng cách từ O tới các đỉnh của tam giác
ABC là nhỏ nhất có thể.
   Tóm lại, trong trường hợp tam giác ABC có một đỉnh không nhỏ hơn
1200 , thì chính đỉnh này là đỉnh cần tìm.


        www.MATHVN.com - HOANG NGOC QUANG, Yen Bai
32

Bài toán 1.24. Cho tam giác ABC nội tiếp đường tròn (O) với
H là trực tâm và AD, BE, CE là các đường cao. Kí hiệu D =
AD ∩ (O), E = BE ∩ (O), F = CF ∩ (O). Chứng minh rằng
       AD   BE   CF              HD HE HF         3
 (i)      +    +     ≥ 9,   (ii)    +    +      ≥ ,
       HD HE HF                  HA HB HC         2
        AD   BE    CF            AD   BE     CF    9
 (iii)     +     +     ≥ 9, (iv)    +     +      ≥ .
        DD   EE    FF            AD   BE     CF    4
Giải.
   + Chứng minh i).
Ta có [HBC] = HD , [HCA] = HE , [HAB] =
       [ABC]  AD [ABC]      BE [ABC]
HF       HD  HE      HF
CF nên AD + BE + CF = 1. Bây giờ, áp
                                  AD
dụng bất đẳng thức (1.2) ta được HD +
BE    CF        9
HE + HF ≥ HD + HE + HF = 9.
               AD   BE   CF
  + Chứng minh ii).
                                     [HBC]
Ta có HD = AD−HD =
      HA
              HD
                                  [ABC]−[HBC] =
   [HBC]                       HE       [HCA]
[HCA]+[HAB] . tương      tự,   HB = [HAB]+[HBC] ,
HF       [HAB]
HC = [HBC]+[HCA] . Cộng 3 bất đẳng thức
này, sau đó áp dụng bất đẳng thức Nesbitt
ta được bất đẳng thức HD + HB + HC ≥ 3 .
                       HA
                             HE   HF
                                        2
                                                Hình 1.23

    + Chứng minh iii).
Dễ chứng minh được HD = DD , HE = EE và HF = F F . Thay vào
                          AD    BE    CF
(i) ta được bất đẳng thức DD + EE + F F ≥ 9.
    + Chứng minh iv).
Ta có AD = AD+DD = 1 + HD . Tương tự, BE = 1 + HE , CF = 1 + HF .
         AD     AD         AD             BE    BE CF         CF
                               AD   BE     CF    HD     HE HF
Cộng 3 đẳng thức này ta được AD + BE + CF = 3 + AD + BE + CF =
                                               AD     BE   CF
4. Bây giờ áp dụng bất đẳng thức (1.2) ta được AD + BE + CF ≥
       9
AD
    + BE + CF
              = 9.
                4
AD   BE   CF


Bài toán 1.25. (Tạp chí THTT số 266) Giả sử M là điểm nằm trong
tam giác ABC. Gọi A1 , B1 , C1 lần lượt là hình chiếu của M trên các
đường thẳng BC, CA, AB. Chứng minh rằng
          M A2            M B2            M C2
                     +               +                ≥ 3.
      (M B1 + M C1 )2 (M C1 + M A1 )2 (M A1 + M B1 )2


          www.MATHVN.com - HOANG NGOC QUANG, Yen Bai
33

Đẳng thức xảy ra khi nào ?

Giải.
  Ta có
      M B1 + M C1     M B1 M C1
                   =      +
          MA          MA    MA
    = sin M AB1 + sin M AC1
                 M AB1 + M AC1        A
        ≤2 sin                 = 2 sin .
                       2              2
            MA             1
Suy ra   M B1 +M C1 ≥ 2 sin A . Tương tự
                             2
                                                     ta có
   MB            1        MC            1
M C1 +M A1 ≥ 2 sin B , M A1 +M B1 ≥ 2 sin C .
                   2                      2
                                                     Vậy     Hình 1.24

        M A2            M B2            M C2
                   +               +
    (M B1 + M C1 )2 (M C1 + M A1 )2 (M A1 + M B1 )2
    1          1      1      1
≥                  +      +
    4       sin2 A sin2 B sin2 C
                 2      2      2

    3                   1           3       1
≥       3
                                  ≥     3         = 3.
    4       sin2 A      2 B   2 C
                     sin 2 sin 2    4       1 2
                 2                          8

Đẳng thức xảy ra khi và chỉ khi ∆ABC đều và M là tâm của tam giác.


Bài toán 1.26. (Trích đề thi học sinh giỏi Quốc gia, 1991) Cho tam
giác ABC với trọng tâm là G và nội tiếp trong đường tròn bán kính R,
các đường trung tuyến của tam giác này xuất phát từ các đỉnh A, B, C
kéo dài cắt đường tròn lần lượt tại D, E, F . Chứng minh rằng
                3    1   1   1  √   1   1   1
                  ≤    +   +   ≤ 3    +   +   .
                R   GD GE GF       AB BC CA
Giải. Gọi M, N, P lần lượt là trung điểm của cạnh BC, CA, AB. Từ
                                a2
AM.M D = BM.M C có M D = 4ma . Suy ra

                                            ma    a2
                            GD = GM + M D =    +                         (1.57)
                                            3    4ma



              www.MATHVN.com - HOANG NGOC QUANG, Yen Bai
34

    Từ (1.57), áp dụng bất đẳng thức AM −
                                   a2       √ ⇒ 1 ≤
                                             a
GM ta được GD ≥ 2                  12   =     3   GD
√                                       √           √
 3                  1                     3    1      3
BC .Tương tự, có GE ≤                   CA  , GF ≤ AB .
Cộng theo vế 3 bất đẳng thức này ta được
 1    1     1
                 √    1    1     1
GD + GE + GF ≤ 3 AB + BC + CA .
                 GA          2ma
Lại từ (1.57) có GD =                 =
                           ma    a2
                         3 3 + 4ma
   8m2a
   2 + 3a2
           . Áp dụng công thức đường
4ma                                                  Hình 1.25
                              2     2    2
                   GA      2b + 2c − a
trung tuyến được GD =                      .
                            a2 + b2 + c2                   2    2
                                                                  +3c2
Tương tự tính GB và GC rồi cộng lại được GD + GB + GC = 3aa2+3b2 +c2 = 3.
              GE    GF
                                          GA
                                              GE   GF        +b
Từ đó GD + BE + GF = 6. Để ý rằng AD, BE, CF đều không lớn hơn
        AD
             GE
                  CF

                      1     1     1     3
2R, thay vào ta được GD + GE + GF ≥ R .
Bài toán 1.27. (Tây ban nha, 1998) Một đường thẳng chứa trọng tâm
G của tam giác ABC cắt cạnh AB tại P và cạnh CA tại Q. Chứng minh
rằng P B . QC ≤ 1 .
     P A QA     4
                                             2
           P B QC         1   PB        QC                            PB       QC
Giải. Vì   P A . QA   ≤   4   PA   +    QA       , ta sẽ chứng minh   PA   +   QA   = 1.
                                                     Vẽ BB , CC song song với trung
                                                  tuyến AA mà B , C nằm trên P Q.
                                                  Các tam giác AP G và BP B đồng
                                                  dạng ; tam giác AQG và CQC cũng
                                                  đồng dạng, do đó P B = BB và QC =
                                                                     PA    AG     QA
                                                  CC
                                                   AG . Cộng 2 đẳng thức này lại với lưu
                                                  ý rằng AG = 2GA = BB + CC ta
                                                  được P B + QC = 1. Từ đó được bất
                                                         PA   QA
              Hình 1.26
                                                  đẳng thức cần chứng minh.
Bài toán 1.28. (Ba Lan, 1999) Cho D là một điểm trên cạnh BC
của tam giác ABC sao cho AD > BC. Điểm E trên CA thỏa mãn
AE       BD
    =          . Chứng minh rằng AD > BE.
EC    AD − BC
Giải.


        www.MATHVN.com - HOANG NGOC QUANG, Yen Bai
35

   Lấy F trên AD sao cho AF = BC
và Gọi E là giao điểm của BF và
AC. Áp dụng định lí hàm số sin cho
tam giác AE F, BCE và BDF , ta
      AE
được E C = AF. sin AF E . sin BE C =
                  sin AE F BC. sin CBE
sin BF D    BD     AE
       =    FD   = EC . Do đó E ≡ E.            Hình 1.27
sin DBF
   Lấy G trên BD sao cho BG = AD
và H là giao điểm của GE với đường
thẳng qua A và song song với BC. Dễ thấy các tam giác ECG và EAH
đồng dạng nên AH = EC = AD−BC = BG−BC = BD , suy ra AH = DB.
                CG
                     AE      BD       BD
                                              CG
Do đó BDAH là một hình bình hành, suy ra BH = AD và ∆BHG cân.
Vậy BH = BG = AD > BE.

Bài toán 1.29. (Tạp chí THTT số 265) Gọi AD, BE, CF là các đường
phân giác trong của tam giác ABC. Chứng minh rằng
                                          1
                                 p(DEF ) ≥ p(ABC),
                                          2
trong đó kí hiệu p(XY Z) là chu vi của tam giác XY Z. Đẳng thức xảy
ra khi nào ?

Giải.
   Từ tính chất của đường
                   AE    c
phân giác BE ta có CE = a ,
suy ra AE = AE+CE = a+c .
        b
                AE     c
              bc
Do đó AE = a+c . Tương tự
       bc
AF = a+b .
   Theo định lí cosin
trong ∆AEF và ∆ABC
có

                                                        Hình 1.28
           2       2         2
      EF =AE + AF − 2AF.AF. cos A
                         2               2
                  bc                bc               b 2 c2       b2 + c2 − a2
               =             +               −2                 .
                 a+c               a+b          (a + c) (a + b)       2bc


           www.MATHVN.com - HOANG NGOC QUANG, Yen Bai
36

                 a2 bc        abc (a + b + c) (b − c)2
          =                 −
            (a + c) (a + b)      (a + c)2 (a + b)2
                   a2 bc
Suy ra EF 2 ≤   (a+c)(a+b) .   Từ đó
                                                     √          √           2
           2   a2 bc  1√ √        1                      ac +       ab
         EF ≤ √    √ = . ac. ab ≤
             4 ac. ab 4           4                         2
                                        2                           2
                 1     a+c a+b                 1   2a + b + c
              ≤           +                 =                           .
                16      2   2                 16       2

Do đó EF ≤ 2a+b+c . Tương tự F D ≤ 2b+c+a , DE ≤ 2c+a+b .
                8                      8            8
                                                          1
Cộng theo vế 3 bất đẳng thức trên được DE + EF + F D ≤ 2 (a + b + c).
hay p(DEF ) ≥ 1 p(ABC).
                2

Bài toán 1.30. (IMO, 1991 ) Cho tam giác ABC. Gọi I là tâm đường
tròn nội tiếp tam giác. Đường phân giác trong của các góc A, B, C lần
lượt cắt các cạnh đối diện tương ứng tại L, M, N . Chứng minh rằng
                          1   AI.BI.CI   8
                            ≤          ≤ .
                          4 AL.BM.CN    27
Giải.
   Sử dụng tính chất đường phân
giác có BL = c , để ý rằng BL +
         LC      b
                                ac
LC = a, ta được BL = b+c và
        ab
LC = b+c . Tiếp tục, áp dụng tích
chất đường phân giác cho phân
giác BI của góc ABL ta thu được
IL      BL        ac       a
AI = AB = (b+c)c = b+c . Do đó
AL     AI+IL          IL         a
AI =     AI   = 1 + AI = 1 + b+c =
a+b+c             AI     b+c                    Hình 1.29
 b+c . Khi đó, AL = a+b+c . Tương
     BI      c+a     CI     a+b
tự, BM = a+b+c , CN = a+b+c . Do đó bất đẳng thức cần chứng minh đưa
về dạng chứa các biến a, b và c
                     1 (b + c)(c + a)(a + b)   8
                       <                     ≤ .
                     4     (a + b + c)3       27



       www.MATHVN.com - HOANG NGOC QUANG, Yen Bai
37


Áp dụng bất đẳng thức AM − GM có
                                                        3
                          (b + c) + (c + a) + (a + b)            8
(b + c)(c + a)(a + b) ≤                                     ≤      (a + b + c)3 .
                                       3                        27
Bất đẳng thức phải được chứng minh.
Để chứng minh bất đẳng thức trái, trước hết để ý rằng
     (b + c)(c + a)(a + b) (a + b + c)(ab + bc + ca) − abc
                          =                                .             (1.58)
         (a + b + c)3                (a + b + c)3
Biết rằng a + b + c = 2p, ab + bc + ca = p2 + r2 + 4rR, abc = 4Rrp thay
vào (1.58) được
          (b + c)(c + a)(a + b) 2p(p2 + r2 + 4rR) − 4Rrp
                               =                         =
              (a + b + c)3                 8p3
          2p2 + 2pr2 + 4Rrp 1 2r2 + 4Rr        1
        =                     = +            > .
                  8p3          4      8p2      4


Bài toán 1.31. (IMO Shorlist, 1996) Cho ABC là tam giác đều và
P là một điểm trong nó. Các đường thẳng AP, BP, CP cắt các cạnh
BC, CA, AB tại các điểm A1 , B1 , C1 , tương ứng. Chứng minh rằng

                  A1 B1 .B1 C1 .C1 A1 ≥ A1 B.B1 C.C1 A.

Giải. Áp dụng định lí hàm số cosin cho ∆CA1 B1 , ta được

            A1 B1 = A1 C 2 + B1 C 2 − A1 C.B1 C ≥ A1 C.B1 C.
                2


              2
Tương tự, B1 C1 ≥ B1 A.C1 A, C1 A1 ≥ C1 B.A1 B. Nhân 3 bất đẳng thức
này, ta được

       A1 B1 .B1 C1 .C1 A2 ≥ A1 C.B1 C.B1 A.C1 A.C1 B.A1 B.
           2      2
                         1                                               (1.59)

Bây giờ, các đường thẳng AA1 , BB1 , CC1 đồng quy, vì vậy áp dụng định
lí Ceva ta có A1 B.B1 C.C1 A = AB1 .BC1 .CA1 , thay vào (1.59) ta thu
được bất đẳng thức cần chứng minh. Đẳng thức xảy ra khi và chỉ khi
CA1 = CB1 , BA1 = BC1 và AB1 = AC1 . Điều này xảy ra khi và chỉ khi
P là tâm của đường tròn ngoại tiếp tam giác ABC.


        www.MATHVN.com - HOANG NGOC QUANG, Yen Bai
38

Bài toán 1.32. (IMO Shorlist, 1999) Cho tam giác ABC và M là một
điểm nằm trong nó. Chứng minh rằng

    min {M A, M B, M C} + M A + M B + M C < AB + BC + CA.

  Trước hết, ta chứng minh bổ đề sau:

Bổ đề 1.2. Nếu M là một điểm nằm trong tứ giác lồi ABCD thì

                     M A + M B < AD + DC + CB.

Chứng minh.




              Hình 1.30                        Hình 1.31

   Gọi N là giao điểm của AM và CD (Hình 1.30). Khi đó M A + M B <
M A + M N + N B ≤ AN + N C + CB ≤ AD + DN + N C + CB =
AD + DC + CB.
Giải. Lấy D, E, F theo thứ tự là trung điểm của BC, CA, AB (Hình
1.31). Xét điểm M nằm trong tam giác, M sẽ thuộc ít nhất hai trong ba
hình thang ABDE, BCEF, CAF D. Không mất tính tổng quát, giả sử
M ∈ ABDE, BCEF . Áp dụng bổ đề trên ta có

                     M A + M B < AE + ED + DB,
                     M B + M C < BF + F E + EC.

Cộng theo vế hai bất đẳng thức trên, ta được

           M B + (M A + M B + M C) < AB + BC + CA.

Vậy min {M A, M B, M C} + M A + M B + M C < AB + BC + CA.


       www.MATHVN.com - HOANG NGOC QUANG, Yen Bai
39

Bài toán 1.33. (IMO Shorlist, 2002) Cho tam giác ABC và F là một
điểm trong nó thỏa mãn AF B = BF C = CF A . Các đường thẳng BF
và CF cắt các cạnh AC và AB tại D và E, tương ứng. Chứng minh
rằng
                          AB + AC ≥ 4DE.
  Trước hết, ta chứng minh bổ đề sau:
Bổ đề 1.3. Cho tam giác ABC, các điểm P và Q nằm trên các tia FD,
FE tương ứng, sao cho P F ≥ λDF, QF ≥ λEF , trong đó λ > 0. Nếu
P F Q ≥ 900 thì P Q ≥ λDE.
Chứng minh. Đặt P F Q = θ. Vì θ ≥ 900 , ta có cos θ ≤ 0. Bây giờ, áp
dụng định lý hàm số cosin, ta có P Q2 = P F 2 + QF 2 − 2P F.QF. cos θ ≥
(λDF )2 + (λEF )2 − 2 (λDF ) . (λEF ) . cos θ = (λDE)2 . Do đó P Q ≥
λDE.
Giải. Lưu ý rằng AF E = BF E = CF D = AF D = 600 . Gọi P, Q là giao
điểm của các đường thẳng BF, EF với đường tròn ngoại tiếp tam giác
CF A, AF B tương ứng. Khi đó, dễ dàng thấy cả hai tam giác CP A, AQB
là đều. Gọi P1 là chân đường vuông góc hạ từ F xuống cạnh AC và giả
sử đường trung trực của AC cắt đường tròn ngoại tiếp tam giác CF A
tại P và P2 .
    Gọi M là trung điểm
                   PD
của AC. Khi đó DF =
PM        PM
F P1 ≥ M P2 = 3 vì
vậy P F ≥ 4DF . Tương
tự, ta có QF ≥ 4EF .
Áp dụng hệ quả trên
với λ = 4 và θ =
DEF = 1200 ta được
P Q ≥ 4DE. Cuối cùng,
áp dụng bất đẳng thức                        Hình 1.32
tam giác, AB + AC = AQ + AP ≥ P Q ≥ 4DE.
Bài toán 1.34. (IMO, 2006) Cho tam giác ABC với I là tâm đường
tròn nội tiếp. Một điểm P nằm trong tam giác thỏa mãn P BA+ P CA =


        www.MATHVN.com - HOANG NGOC QUANG, Yen Bai
40

P BC + P CB. Chứng minh rằng AP > AI và đẳng thức xảy ra khi và
chỉ khi P ≡ I.

Giải. Đặt A = α, B = β, C = γ. Vì P BA + P CA + P BC + P CB =
                                                        β+γ
β + γ nên từ điều kiện bài toán ta có P BC + P CB =           . Suy ra
               α                                           2
BP C = 900 + .
               2
                              β+γ
   Mặt khác, BIC = 1800 −          =
       α                        2
900 + . Do đó BP C = BIC, và vì
       2
P và I nằm cùng phía với BC nên
các điểm B, I, P, C cùng nằm trên một
đường tròn. Nói cách khác, P nằm
trên đường tròn ω ngoại tiếp tam giác
BCI. Gọi Ω là đường tròn ngoại tiếp
tam giác ABC. Rõ ràng tâm của ω
là trung điểm M của cung BC của
Ω. Đây cũng là giao điểm thứ hai của
phân giác AI và ω. Từ tam giác AP M              Hình 1.33
ta có AP + P M ≥ AM = AI + IM = AI + P M . Do đó AP ≥ AI. Đẳng
thức xảy ra khi và chỉ khi P nằm trên đoạn AM , điều này xảy ra khi và
chỉ khi P ≡ I.

1.7.   Các bất đẳng thức trong tứ giác

   Kí hiệu ABCD là tứ giác lồi với các đỉnh là A, B, C, D được vẽ theo
một chiều nhất định nào đó (cùng chiều kim đồng hồ hay ngược chiều
kim đồng hồ). Để đơn giản, độ lớn của góc ứng với các đỉnh A, B, C, D
cũng được kí hiệu là A, B, C, D.
   Độ dài các cạnh của tam giác: AB = a, BC = b, CD = c, DA = d.
                               a+b+c+d
   Nủa chu vi của tứ giác: p =             .
                                    2
   Độ dài các đường chéo: AC = m, BD = n.
   Diện tích của tứ giác: S = SABCD hay [ABCD]




        www.MATHVN.com - HOANG NGOC QUANG, Yen Bai
41

1.7.1.     Các bất đẳng thức cơ bản trong tứ giác
Bài toán 1.35. Cho tứ giác ABCD, có AB + BD ≤ AC + DC. Chứng
minh AB < AC.
Giải. Gọi O là giao điểm của AC và BD . Xét các tam giác OAB và
ODC ta có
  AB < OA + OB, DC < OC + OD.
Do đó
 AB + CD < (OA + OC) + (OB + OD)
             =AC + BD.            (1.60)
Mặt khác, theo giả thiết ta có
   AB + BD ≤ AC + DC.             (1.61)           Hình 1.34

Cộng theo vế (1.60) và (1.61) ta được 2AB + DC + BD < 2AC + BD +
DC. Suy ra AB < AC.
Bài toán 1.36. Tổng hai đường chéo của một tứ giác lồi ABCD nhỏ
hơn chu vi của tứ giác và lớn hơn nửa chu vi của nó.
Giải. Gọi O là giao điểm của AC và BD, ta có
         AC + BD = (OA + OB) + (OC + OD) > AB + CD.            (1.62)
         AC + BD = (OA + OD) + (OB + OC) > AD + BC.            (1.63)
Cộng theo vế các bất đẳng thức (1.62) và (1.63) ta thu được
                             AB + BC + CD + DA
                 AC + BD >                            .
                                        2
Mặt khác, AC < AB + BC và AC < DA + CD. Cộng theo vế hai bất
đẳng thức này, ta có
                          AB + BC + CD + DA
                    AC <                          .           (1.64)
                                     2
Tương tự,
                          AB + BC + CD + DA
                    BD <                          .           (1.65)
                                     2
Cộng theo vế hai bất đẳng thức (1.64) và (1.65) , ta thu được
                  AC + BD < AB + BC + CD + DA.


          www.MATHVN.com - HOANG NGOC QUANG, Yen Bai
42

Bài toán 1.37. Cho tứ giác ABCD, gọi M, N, P, Q lần lượt là trung
điểm của các cạnh AB, BC, CD, DA. Chứng minh rằng
                        AD + BC
               a)M P ≤          .                           (1.66)
                           2
                             AB + BC + CD + DA
               b)M P + N Q ≤                     .          (1.67)
                                      2
Giải.
   a) Gọi I là trung điểm của BD, M I
là đường trung bình của ∆ABD. Suy ra
M I = AD . IP là đường trung bình của
        2
∆DBC, suy ra IP = BC . Xét 3 điểm
                         2
I, M, P ta có M P ≤ M I + IP , suy ra
M P ≤ AD+BC .
          2
  b) Áp dụng ý a) ta có N Q ≤ AB+DC .
                                 2
                    AD+BC    AB+DC
Do đó M P + N Q ≤      2   +   2   =
AB+BC+CD+DA                                        Hình 1.35
      2     .
Bài toán 1.38. Cho hình vuông ABCD cạnh a. M, N là hai điểm ở
                                                √
trong hình vuông đã cho. Chứng minh rằng M N ≤ a 2.
Giải.
   Vì ABCD là hình vuông cạnh
                √      √
a nên AC = AB 2 = a 2.
Vẽ đường tròn (O) ngoại tiếp
hình vuông ABCD. Ta có đường
                 √
kính của (O) là a 2, M và N là
hai điểm nằm trong O.
Gọi M N là dây cung đi qua M
và N . Ta có M N ≤ M N mà
          √
M N ≤ a 2 (đường kinh là dây
cung lớn nhất trong đường tròn).
               √
Do đó M N ≤ a 2.                              Hình 1.36
Bài toán 1.39. Cho hình thang ABCD có đáy nhỏ là AB và C + D ≤
900 . Gọi M và N lần lượt là trung điểm của các cạnh AB và CD. Chứng
minh rằng M N ≤ CD−AB .
                      2



       www.MATHVN.com - HOANG NGOC QUANG, Yen Bai
43

Giải.
   Qua M vẽ đường thẳng song song
với AD cắtDC tại E và qua M vẽ
đường thẳng song song với BC cắt
DC tại F . Suy ra D = E1 , C = F1 .
Suy ra E1 + F1 = D + C ≤ 900 ⇒
EM F ≥ 900 . Theo bài toán 1.3 tam
giác M EF có EM F ≥ 900 và M N là      Hình 1.37
trung tuyến nên M N ≤ EF .
                        2
Mặt khác, có AB//DC và AD//M E nên ADEM là hình bình hành.
Suy ra DE = AM = 1 AB, tương tự F C = M B = 1 AB. Do đó
                       2                        2
EF = CD − AB.
Vậy M N ≤ CD−AB .
              2

Bài toán 1.40. Cho tứ giác ABCD, M là một điểm thuộc cạnh CD
(M khác C, D). Chứng minh rằng

             M A + M B < max {CA + CB; DA + DB} .          (1.68)

Giải.
                             Gọi A là điểm đối xứng của A qua CD.
                          A B cắt CD ở P .
                          Vì M thuộc đoạn CD nên M thuộc
                          ∆A BC hoặc ∆A BD. Theo định lí 1.20
                          ta có
                                M A + M B < CA + CB
                                M A + M B < DA + DB
                                  M A + M B < CA + CB
         Hình 1.38             ⇒
                                  M A + M B < DA + DB

Do đó M A + M B < max {CA + CB; DA + DB}.

Chú ý 1.3. Từ bài toán 1.40 ta có các kết quả sau:
1) Cho tứ giác ABCD, M là một điểm thuộc cạnh CD (M có
thể trùng với C hoặc D). Ta có bất đẳng thức M A + M B ≤
max {CA + CB; DA + DB}.


        www.MATHVN.com - HOANG NGOC QUANG, Yen Bai
44

2) Cho ABCD là hình chữ nhật và điểm M nằm trên cạnh CD. Ta có
bất đẳng thức M A + M B ≤ CA + CB.
3) Cho ABCD là hình vuông cạnh a và điểm M nằm trên cạnh CD. Ta
                                   √
có bất đẳng thức M A + M B ≤ (1 + 2)a.
   Đẳng thức trong các bất đẳng thức trên xảy ra khi và chỉ khi M ≡ C
hoặc M ≡ D.
Bài toán 1.41. (Đề thi vào lớp 10 chuyên toán-tin, ĐHSP, ĐHQG Hà
Nội 1998-1999) Cho hình chữ nhật ABCD và điểm M nằm trong hình
chữ nhật và có thể nằm trên các cạnh của ABCD. Chứng minh rằng
M A + M B + M C + M D ≤ AB + AC + AD.
Giải.
   Qua M vẽ đường thẳng song song với
AD cắt AB, DE lần lượt tại E, F . Áp
dụng chú ý 1.3 của bài toán 1.40 vào các
hình chữ nhật AEF D, EBCF và ABCD
ta có M A+M D ≤ EA+ED, M B+M C ≤
EB + EC, ED + EC ≤ AD + AC. Do đó
M A + M B + M C + M D ≤ (EA + EB) +
                                                     Hình 1.39
(ED + EC) ≤ AB + AC + AD.
   Ta có bài toán tổng quát hơn sau đây
Bài toán 1.42. (Tuyển tập 5 năm tạp chí THTT) Cho tứ giác ABCD,
M là một điểm trong tứ giác. Đặt dA = AB + AC + AD, dB = BC +
BD + BA, dC = CD + CA + CB, dD = DA + DB + DC. Chứng minh
rằng

          M A + M B + M C + M D < max {dA ; dB ; dC ; dD } .

Giải. Kéo dài AM một đoạn M B bằng M B. Qua M kẻ đường trung
trực của BB . Đường này theo thứ tự cắt hai cạnh tứ giác tại I, J. Có
thể xảy ra một trong ba trường hợp hình (A), (B), (C). Vì trong các hình
(B), (C) bài toán được chứng minh tương tự nhưng đơn giản hơn trong
trường hợp (A) nên ở đây ta chỉ chứng minh trong trường hợp (A). Không
mất tính tổng quát giả sử rằng IC + ID = max {IC + ID, JC + JD} .


        www.MATHVN.com - HOANG NGOC QUANG, Yen Bai
45




                                   Hình 1.40

  Áp dụng bài toán 1.40 cho tứ giác CIJD ta có M C +M D < IC +ID.
Lại có M A + M B = M A + M B = AB < IA + IB = IA + IB. Do đó
M A + M B + M C + M D < IA + IB + IC + ID = IA + ID + BC. Áp
dụng bài toán 1.40 cho tứ giác ABCD ta có
                IA + ID < max {CA + CD; BA + BD} .

Vậy M A + M B + M C + M D < max {CA + CD; BA + BD} + BC =
    = max {BC + BD + BA; CD + CA + CB} = max {dA ; dC }
    ≤ max {dA ; dB ; dC ; dD } .
Chú ý 1.4. Từ bài toán 1.42 ta có các kết quả sau:
1) Cho hình chữ nhật ABCD có độ dài các cạnh là a, b và độ dài đường
chéo là c. M là một điểm nằm bên trong hình chữ nhật đó. Ta có bất
đẳng thức M A + M B + M C + M D < a + b + c.
2) Cho hình vuông ABCD cạnh và M là một điểm nằm bên trong hình
                                                               √
vuông đó. Ta có bất đẳng thức M A + M B + M C + M D < (2 + 2)a.

1.7.2.   Các bất đẳng thức khác trong tứ giác
Bài toán 1.43. (IMO shorlist) Diện tích của một tứ giác với các cạnh
a, b, c và d là S. Chứng minh rằng
                                 a+c b+d
                            S≤       .   .
                                   2   2
Giải. Trước tiên, giả sử tứ giác ABCD không lồi. Khi đó một trong các
đường chéo của nó, chẳng hạn BD sẽ không có điểm chung với phần
trong của tứ giác.


         www.MATHVN.com - HOANG NGOC QUANG, Yen Bai
Một số bất đẳng thức hình học   luận văn của thầy hoàng ngọc quang
Một số bất đẳng thức hình học   luận văn của thầy hoàng ngọc quang
Một số bất đẳng thức hình học   luận văn của thầy hoàng ngọc quang
Một số bất đẳng thức hình học   luận văn của thầy hoàng ngọc quang
Một số bất đẳng thức hình học   luận văn của thầy hoàng ngọc quang
Một số bất đẳng thức hình học   luận văn của thầy hoàng ngọc quang
Một số bất đẳng thức hình học   luận văn của thầy hoàng ngọc quang
Một số bất đẳng thức hình học   luận văn của thầy hoàng ngọc quang
Một số bất đẳng thức hình học   luận văn của thầy hoàng ngọc quang
Một số bất đẳng thức hình học   luận văn của thầy hoàng ngọc quang
Một số bất đẳng thức hình học   luận văn của thầy hoàng ngọc quang
Một số bất đẳng thức hình học   luận văn của thầy hoàng ngọc quang
Một số bất đẳng thức hình học   luận văn của thầy hoàng ngọc quang
Một số bất đẳng thức hình học   luận văn của thầy hoàng ngọc quang
Một số bất đẳng thức hình học   luận văn của thầy hoàng ngọc quang
Một số bất đẳng thức hình học   luận văn của thầy hoàng ngọc quang
Một số bất đẳng thức hình học   luận văn của thầy hoàng ngọc quang
Một số bất đẳng thức hình học   luận văn của thầy hoàng ngọc quang
Một số bất đẳng thức hình học   luận văn của thầy hoàng ngọc quang
Một số bất đẳng thức hình học   luận văn của thầy hoàng ngọc quang
Một số bất đẳng thức hình học   luận văn của thầy hoàng ngọc quang
Một số bất đẳng thức hình học   luận văn của thầy hoàng ngọc quang
Một số bất đẳng thức hình học   luận văn của thầy hoàng ngọc quang
Một số bất đẳng thức hình học   luận văn của thầy hoàng ngọc quang
Một số bất đẳng thức hình học   luận văn của thầy hoàng ngọc quang
Một số bất đẳng thức hình học   luận văn của thầy hoàng ngọc quang
Một số bất đẳng thức hình học   luận văn của thầy hoàng ngọc quang
Một số bất đẳng thức hình học   luận văn của thầy hoàng ngọc quang
Một số bất đẳng thức hình học   luận văn của thầy hoàng ngọc quang
Một số bất đẳng thức hình học   luận văn của thầy hoàng ngọc quang
Một số bất đẳng thức hình học   luận văn của thầy hoàng ngọc quang
Một số bất đẳng thức hình học   luận văn của thầy hoàng ngọc quang
Một số bất đẳng thức hình học   luận văn của thầy hoàng ngọc quang
Một số bất đẳng thức hình học   luận văn của thầy hoàng ngọc quang
Một số bất đẳng thức hình học   luận văn của thầy hoàng ngọc quang
Một số bất đẳng thức hình học   luận văn của thầy hoàng ngọc quang
Một số bất đẳng thức hình học   luận văn của thầy hoàng ngọc quang
Một số bất đẳng thức hình học   luận văn của thầy hoàng ngọc quang
Một số bất đẳng thức hình học   luận văn của thầy hoàng ngọc quang
Một số bất đẳng thức hình học   luận văn của thầy hoàng ngọc quang
Một số bất đẳng thức hình học   luận văn của thầy hoàng ngọc quang
Một số bất đẳng thức hình học   luận văn của thầy hoàng ngọc quang
Một số bất đẳng thức hình học   luận văn của thầy hoàng ngọc quang
Một số bất đẳng thức hình học   luận văn của thầy hoàng ngọc quang
Một số bất đẳng thức hình học   luận văn của thầy hoàng ngọc quang
Một số bất đẳng thức hình học   luận văn của thầy hoàng ngọc quang
Một số bất đẳng thức hình học   luận văn của thầy hoàng ngọc quang
Một số bất đẳng thức hình học   luận văn của thầy hoàng ngọc quang
Một số bất đẳng thức hình học   luận văn của thầy hoàng ngọc quang
Một số bất đẳng thức hình học   luận văn của thầy hoàng ngọc quang
Một số bất đẳng thức hình học   luận văn của thầy hoàng ngọc quang
Một số bất đẳng thức hình học   luận văn của thầy hoàng ngọc quang
Một số bất đẳng thức hình học   luận văn của thầy hoàng ngọc quang
Một số bất đẳng thức hình học   luận văn của thầy hoàng ngọc quang
Một số bất đẳng thức hình học   luận văn của thầy hoàng ngọc quang
Một số bất đẳng thức hình học   luận văn của thầy hoàng ngọc quang
Một số bất đẳng thức hình học   luận văn của thầy hoàng ngọc quang
Một số bất đẳng thức hình học   luận văn của thầy hoàng ngọc quang
Một số bất đẳng thức hình học   luận văn của thầy hoàng ngọc quang
Một số bất đẳng thức hình học   luận văn của thầy hoàng ngọc quang
Một số bất đẳng thức hình học   luận văn của thầy hoàng ngọc quang
Một số bất đẳng thức hình học   luận văn của thầy hoàng ngọc quang
Một số bất đẳng thức hình học   luận văn của thầy hoàng ngọc quang
Một số bất đẳng thức hình học   luận văn của thầy hoàng ngọc quang
Một số bất đẳng thức hình học   luận văn của thầy hoàng ngọc quang
Một số bất đẳng thức hình học   luận văn của thầy hoàng ngọc quang
Một số bất đẳng thức hình học   luận văn của thầy hoàng ngọc quang
Một số bất đẳng thức hình học   luận văn của thầy hoàng ngọc quang
Một số bất đẳng thức hình học   luận văn của thầy hoàng ngọc quang
Một số bất đẳng thức hình học   luận văn của thầy hoàng ngọc quang
Một số bất đẳng thức hình học   luận văn của thầy hoàng ngọc quang
Một số bất đẳng thức hình học   luận văn của thầy hoàng ngọc quang
Một số bất đẳng thức hình học   luận văn của thầy hoàng ngọc quang

More Related Content

What's hot

Các chuyên đề bồi dưỡng HSG môn Toán THCS hay nhất
Các chuyên đề bồi dưỡng HSG môn Toán THCS hay nhấtCác chuyên đề bồi dưỡng HSG môn Toán THCS hay nhất
Các chuyên đề bồi dưỡng HSG môn Toán THCS hay nhấtBồi dưỡng Toán lớp 6
 
Cđ một số dạng pt vô tỷ và cách giải
Cđ một số dạng pt vô tỷ và cách giảiCđ một số dạng pt vô tỷ và cách giải
Cđ một số dạng pt vô tỷ và cách giảiCảnh
 
chuyên đề cực trị GTLN và GTNN , rất chi tiết và đầy đủ
chuyên đề cực trị GTLN và GTNN , rất chi tiết và đầy đủ chuyên đề cực trị GTLN và GTNN , rất chi tiết và đầy đủ
chuyên đề cực trị GTLN và GTNN , rất chi tiết và đầy đủ Jackson Linh
 
Bất đẳng thức mathscope
Bất đẳng thức mathscopeBất đẳng thức mathscope
Bất đẳng thức mathscopePhúc Võ
 
Phương Tích - Trục Đẳng Phương
Phương Tích - Trục Đẳng PhươngPhương Tích - Trục Đẳng Phương
Phương Tích - Trục Đẳng PhươngNhập Vân Long
 
Tuyển tập chuyên đề bất đẳng thức có lời giải chi tiết 2
Tuyển tập chuyên đề bất đẳng thức có lời giải chi tiết 2Tuyển tập chuyên đề bất đẳng thức có lời giải chi tiết 2
Tuyển tập chuyên đề bất đẳng thức có lời giải chi tiết 2https://www.facebook.com/garmentspace
 
Hệ Hoán Vị Vòng Quanh
Hệ Hoán Vị Vòng QuanhHệ Hoán Vị Vòng Quanh
Hệ Hoán Vị Vòng QuanhNhập Vân Long
 
Chuyên đề hệ phương trình bằng phương pháp hàm số
Chuyên đề hệ phương trình bằng phương pháp hàm sốChuyên đề hệ phương trình bằng phương pháp hàm số
Chuyên đề hệ phương trình bằng phương pháp hàm sốVui Lên Bạn Nhé
 
Đặt ẩn phụ giải phương trình chứa căn
Đặt ẩn phụ giải phương trình chứa cănĐặt ẩn phụ giải phương trình chứa căn
Đặt ẩn phụ giải phương trình chứa căntuituhoc
 
Kĩ thuật giải hệ phương trình
Kĩ thuật giải hệ phương trìnhKĩ thuật giải hệ phương trình
Kĩ thuật giải hệ phương trìnhToàn Đinh
 
9 phương pháp giải phương trình nghiệm nguyên
9 phương pháp giải phương trình nghiệm nguyên9 phương pháp giải phương trình nghiệm nguyên
9 phương pháp giải phương trình nghiệm nguyênThấy Tên Tao Không
 
Sach bat dang thuc rat hay
Sach bat dang thuc rat haySach bat dang thuc rat hay
Sach bat dang thuc rat hayTuân Ngô
 
72 hệ phương trình
72 hệ phương trình72 hệ phương trình
72 hệ phương trìnhHades0510
 
Bài toán số học liên quan tới lũy thữa
Bài toán số học liên quan tới lũy thữaBài toán số học liên quan tới lũy thữa
Bài toán số học liên quan tới lũy thữaThế Giới Tinh Hoa
 
Luận án tiến sĩ toán học định lý điểm bất động cho một số ánh xạ co suy rộng ...
Luận án tiến sĩ toán học định lý điểm bất động cho một số ánh xạ co suy rộng ...Luận án tiến sĩ toán học định lý điểm bất động cho một số ánh xạ co suy rộng ...
Luận án tiến sĩ toán học định lý điểm bất động cho một số ánh xạ co suy rộng ...https://www.facebook.com/garmentspace
 
Luận văn: Dạy học một số chủ đề trong môn Toán lớp 10 theo định hướng giáo dụ...
Luận văn: Dạy học một số chủ đề trong môn Toán lớp 10 theo định hướng giáo dụ...Luận văn: Dạy học một số chủ đề trong môn Toán lớp 10 theo định hướng giáo dụ...
Luận văn: Dạy học một số chủ đề trong môn Toán lớp 10 theo định hướng giáo dụ...Dịch vụ viết thuê Khóa Luận - ZALO 0932091562
 
Tuyển tập các bài Toán Hình học lớp 9 ôn thi vào 10
Tuyển tập các bài Toán Hình học lớp 9 ôn thi vào 10Tuyển tập các bài Toán Hình học lớp 9 ôn thi vào 10
Tuyển tập các bài Toán Hình học lớp 9 ôn thi vào 10BOIDUONGTOAN.COM
 

What's hot (20)

Luận văn: Các bài toán về hệ thức lượng trong tam giác, HOT, 9đ
Luận văn: Các bài toán về hệ thức lượng trong tam giác, HOT, 9đLuận văn: Các bài toán về hệ thức lượng trong tam giác, HOT, 9đ
Luận văn: Các bài toán về hệ thức lượng trong tam giác, HOT, 9đ
 
Các chuyên đề bồi dưỡng HSG môn Toán THCS hay nhất
Các chuyên đề bồi dưỡng HSG môn Toán THCS hay nhấtCác chuyên đề bồi dưỡng HSG môn Toán THCS hay nhất
Các chuyên đề bồi dưỡng HSG môn Toán THCS hay nhất
 
Cđ một số dạng pt vô tỷ và cách giải
Cđ một số dạng pt vô tỷ và cách giảiCđ một số dạng pt vô tỷ và cách giải
Cđ một số dạng pt vô tỷ và cách giải
 
chuyên đề cực trị GTLN và GTNN , rất chi tiết và đầy đủ
chuyên đề cực trị GTLN và GTNN , rất chi tiết và đầy đủ chuyên đề cực trị GTLN và GTNN , rất chi tiết và đầy đủ
chuyên đề cực trị GTLN và GTNN , rất chi tiết và đầy đủ
 
Bất đẳng thức mathscope
Bất đẳng thức mathscopeBất đẳng thức mathscope
Bất đẳng thức mathscope
 
Phương Tích - Trục Đẳng Phương
Phương Tích - Trục Đẳng PhươngPhương Tích - Trục Đẳng Phương
Phương Tích - Trục Đẳng Phương
 
Tuyển tập chuyên đề bất đẳng thức có lời giải chi tiết 2
Tuyển tập chuyên đề bất đẳng thức có lời giải chi tiết 2Tuyển tập chuyên đề bất đẳng thức có lời giải chi tiết 2
Tuyển tập chuyên đề bất đẳng thức có lời giải chi tiết 2
 
Hệ Hoán Vị Vòng Quanh
Hệ Hoán Vị Vòng QuanhHệ Hoán Vị Vòng Quanh
Hệ Hoán Vị Vòng Quanh
 
Chuyên đề hệ phương trình bằng phương pháp hàm số
Chuyên đề hệ phương trình bằng phương pháp hàm sốChuyên đề hệ phương trình bằng phương pháp hàm số
Chuyên đề hệ phương trình bằng phương pháp hàm số
 
Luận văn: Bất đẳng thức trong lớp hàm siêu việt, HAY, 9đ
Luận văn: Bất đẳng thức trong lớp hàm siêu việt, HAY, 9đLuận văn: Bất đẳng thức trong lớp hàm siêu việt, HAY, 9đ
Luận văn: Bất đẳng thức trong lớp hàm siêu việt, HAY, 9đ
 
Đặt ẩn phụ giải phương trình chứa căn
Đặt ẩn phụ giải phương trình chứa cănĐặt ẩn phụ giải phương trình chứa căn
Đặt ẩn phụ giải phương trình chứa căn
 
Kĩ thuật giải hệ phương trình
Kĩ thuật giải hệ phương trìnhKĩ thuật giải hệ phương trình
Kĩ thuật giải hệ phương trình
 
9 phương pháp giải phương trình nghiệm nguyên
9 phương pháp giải phương trình nghiệm nguyên9 phương pháp giải phương trình nghiệm nguyên
9 phương pháp giải phương trình nghiệm nguyên
 
Sach bat dang thuc rat hay
Sach bat dang thuc rat haySach bat dang thuc rat hay
Sach bat dang thuc rat hay
 
72 hệ phương trình
72 hệ phương trình72 hệ phương trình
72 hệ phương trình
 
Bài toán số học liên quan tới lũy thữa
Bài toán số học liên quan tới lũy thữaBài toán số học liên quan tới lũy thữa
Bài toán số học liên quan tới lũy thữa
 
Dãy số và giới hạn
Dãy số và giới hạnDãy số và giới hạn
Dãy số và giới hạn
 
Luận án tiến sĩ toán học định lý điểm bất động cho một số ánh xạ co suy rộng ...
Luận án tiến sĩ toán học định lý điểm bất động cho một số ánh xạ co suy rộng ...Luận án tiến sĩ toán học định lý điểm bất động cho một số ánh xạ co suy rộng ...
Luận án tiến sĩ toán học định lý điểm bất động cho một số ánh xạ co suy rộng ...
 
Luận văn: Dạy học một số chủ đề trong môn Toán lớp 10 theo định hướng giáo dụ...
Luận văn: Dạy học một số chủ đề trong môn Toán lớp 10 theo định hướng giáo dụ...Luận văn: Dạy học một số chủ đề trong môn Toán lớp 10 theo định hướng giáo dụ...
Luận văn: Dạy học một số chủ đề trong môn Toán lớp 10 theo định hướng giáo dụ...
 
Tuyển tập các bài Toán Hình học lớp 9 ôn thi vào 10
Tuyển tập các bài Toán Hình học lớp 9 ôn thi vào 10Tuyển tập các bài Toán Hình học lớp 9 ôn thi vào 10
Tuyển tập các bài Toán Hình học lớp 9 ôn thi vào 10
 

Viewers also liked

19 phương phap chứng minh bất đẳng thức
19 phương phap chứng minh bất đẳng thức19 phương phap chứng minh bất đẳng thức
19 phương phap chứng minh bất đẳng thứcThế Giới Tinh Hoa
 
Một số bất đẳng thức hình học luận văn của thầy hoàng ngọc quang
Một số bất đẳng thức hình học   luận văn của thầy hoàng ngọc quangMột số bất đẳng thức hình học   luận văn của thầy hoàng ngọc quang
Một số bất đẳng thức hình học luận văn của thầy hoàng ngọc quangThế Giới Tinh Hoa
 
CHUYÊN ĐỀ HÌNH HỌC ÔN THI VÀO LỚP 10 CÁC TRƯỜNG CHUYÊN
CHUYÊN ĐỀ HÌNH HỌC ÔN THI VÀO LỚP 10 CÁC TRƯỜNG CHUYÊNCHUYÊN ĐỀ HÌNH HỌC ÔN THI VÀO LỚP 10 CÁC TRƯỜNG CHUYÊN
CHUYÊN ĐỀ HÌNH HỌC ÔN THI VÀO LỚP 10 CÁC TRƯỜNG CHUYÊNBOIDUONGTOAN.COM
 
Boi duong hinh hoc phang toan cuc tri
Boi duong hinh hoc phang  toan cuc triBoi duong hinh hoc phang  toan cuc tri
Boi duong hinh hoc phang toan cuc trihaisuoicat
 
Chứng minh bất đẳng thức bằng phương pháp chọn điểm rơi. (1)
Chứng minh bất đẳng thức bằng phương pháp chọn điểm rơi. (1)Chứng minh bất đẳng thức bằng phương pháp chọn điểm rơi. (1)
Chứng minh bất đẳng thức bằng phương pháp chọn điểm rơi. (1)Sao Băng Lạnh Giá
 
Hướng dẫn giải bài tập chuỗi - Toán cao cấp
Hướng dẫn giải bài tập chuỗi - Toán cao cấpHướng dẫn giải bài tập chuỗi - Toán cao cấp
Hướng dẫn giải bài tập chuỗi - Toán cao cấpVan-Duyet Le
 
Hình giải tích 12 1đ
Hình giải tích 12   1đHình giải tích 12   1đ
Hình giải tích 12 1đQuốc Nguyễn
 

Viewers also liked (7)

19 phương phap chứng minh bất đẳng thức
19 phương phap chứng minh bất đẳng thức19 phương phap chứng minh bất đẳng thức
19 phương phap chứng minh bất đẳng thức
 
Một số bất đẳng thức hình học luận văn của thầy hoàng ngọc quang
Một số bất đẳng thức hình học   luận văn của thầy hoàng ngọc quangMột số bất đẳng thức hình học   luận văn của thầy hoàng ngọc quang
Một số bất đẳng thức hình học luận văn của thầy hoàng ngọc quang
 
CHUYÊN ĐỀ HÌNH HỌC ÔN THI VÀO LỚP 10 CÁC TRƯỜNG CHUYÊN
CHUYÊN ĐỀ HÌNH HỌC ÔN THI VÀO LỚP 10 CÁC TRƯỜNG CHUYÊNCHUYÊN ĐỀ HÌNH HỌC ÔN THI VÀO LỚP 10 CÁC TRƯỜNG CHUYÊN
CHUYÊN ĐỀ HÌNH HỌC ÔN THI VÀO LỚP 10 CÁC TRƯỜNG CHUYÊN
 
Boi duong hinh hoc phang toan cuc tri
Boi duong hinh hoc phang  toan cuc triBoi duong hinh hoc phang  toan cuc tri
Boi duong hinh hoc phang toan cuc tri
 
Chứng minh bất đẳng thức bằng phương pháp chọn điểm rơi. (1)
Chứng minh bất đẳng thức bằng phương pháp chọn điểm rơi. (1)Chứng minh bất đẳng thức bằng phương pháp chọn điểm rơi. (1)
Chứng minh bất đẳng thức bằng phương pháp chọn điểm rơi. (1)
 
Hướng dẫn giải bài tập chuỗi - Toán cao cấp
Hướng dẫn giải bài tập chuỗi - Toán cao cấpHướng dẫn giải bài tập chuỗi - Toán cao cấp
Hướng dẫn giải bài tập chuỗi - Toán cao cấp
 
Hình giải tích 12 1đ
Hình giải tích 12   1đHình giải tích 12   1đ
Hình giải tích 12 1đ
 

Similar to Một số bất đẳng thức hình học luận văn của thầy hoàng ngọc quang

3 câu thi thử;
3 câu thi thử;3 câu thi thử;
3 câu thi thử;nam nam
 
bộ 3 câu hỏi khó phân loại trong đề thi thử THPT Quốc gia môn Toán​ năm 2015
bộ 3 câu hỏi khó phân loại trong đề thi thử THPT Quốc gia môn Toán​ năm 2015bộ 3 câu hỏi khó phân loại trong đề thi thử THPT Quốc gia môn Toán​ năm 2015
bộ 3 câu hỏi khó phân loại trong đề thi thử THPT Quốc gia môn Toán​ năm 2015Linh Nguyễn
 
Đề tài: Đại cương về không gian Vec-tơ tô-pô, HAY - Gửi miễn phí qua zalo=> 0...
Đề tài: Đại cương về không gian Vec-tơ tô-pô, HAY - Gửi miễn phí qua zalo=> 0...Đề tài: Đại cương về không gian Vec-tơ tô-pô, HAY - Gửi miễn phí qua zalo=> 0...
Đề tài: Đại cương về không gian Vec-tơ tô-pô, HAY - Gửi miễn phí qua zalo=> 0...Dịch vụ viết bài trọn gói ZALO: 0909232620
 
Xây dựng hệ thống câu hỏi nêu vấn đề trong dạy học hai tác phẩm "Tràng Giang"...
Xây dựng hệ thống câu hỏi nêu vấn đề trong dạy học hai tác phẩm "Tràng Giang"...Xây dựng hệ thống câu hỏi nêu vấn đề trong dạy học hai tác phẩm "Tràng Giang"...
Xây dựng hệ thống câu hỏi nêu vấn đề trong dạy học hai tác phẩm "Tràng Giang"...nataliej4
 
Luận văn: Tương đương bảo giác giữa các miền n-Liên trong mặt phẳng phức
Luận văn: Tương đương bảo giác giữa các miền n-Liên trong mặt phẳng phứcLuận văn: Tương đương bảo giác giữa các miền n-Liên trong mặt phẳng phức
Luận văn: Tương đương bảo giác giữa các miền n-Liên trong mặt phẳng phứcViết thuê trọn gói ZALO 0934573149
 
Giáo trình Hóa phân tích (Dùng cho Sinh viên chuyên hóa).pdf
Giáo trình Hóa phân tích (Dùng cho Sinh viên chuyên hóa).pdfGiáo trình Hóa phân tích (Dùng cho Sinh viên chuyên hóa).pdf
Giáo trình Hóa phân tích (Dùng cho Sinh viên chuyên hóa).pdfMan_Ebook
 
Luận án tiến sĩ toán học một số hệ phương trình cặp trong cơ học chất lỏng
Luận án tiến sĩ toán học một số hệ phương trình cặp trong cơ học chất lỏngLuận án tiến sĩ toán học một số hệ phương trình cặp trong cơ học chất lỏng
Luận án tiến sĩ toán học một số hệ phương trình cặp trong cơ học chất lỏnghttps://www.facebook.com/garmentspace
 
Luận án tiến sĩ toán học ngưỡng chính tắc của hàm chỉnh hình và hàm đa điều h...
Luận án tiến sĩ toán học ngưỡng chính tắc của hàm chỉnh hình và hàm đa điều h...Luận án tiến sĩ toán học ngưỡng chính tắc của hàm chỉnh hình và hàm đa điều h...
Luận án tiến sĩ toán học ngưỡng chính tắc của hàm chỉnh hình và hàm đa điều h...https://www.facebook.com/garmentspace
 
Phát triển năng lực giải quyết vấn đề cho học sinh thông qua dạy học phần hiđ...
Phát triển năng lực giải quyết vấn đề cho học sinh thông qua dạy học phần hiđ...Phát triển năng lực giải quyết vấn đề cho học sinh thông qua dạy học phần hiđ...
Phát triển năng lực giải quyết vấn đề cho học sinh thông qua dạy học phần hiđ...nataliej4
 
Luận văn thạc sĩ sư phạm.
Luận văn thạc sĩ sư phạm.Luận văn thạc sĩ sư phạm.
Luận văn thạc sĩ sư phạm.ssuser499fca
 

Similar to Một số bất đẳng thức hình học luận văn của thầy hoàng ngọc quang (20)

Luận văn: Giải bài toán bất đẳng thức biến phân giả đơn điệu, 9đ
Luận văn: Giải bài toán bất đẳng thức biến phân giả đơn điệu, 9đLuận văn: Giải bài toán bất đẳng thức biến phân giả đơn điệu, 9đ
Luận văn: Giải bài toán bất đẳng thức biến phân giả đơn điệu, 9đ
 
Luận án: Xử lý tri thức không nhất quán trong Ontology, HAY
Luận án: Xử lý tri thức không nhất quán trong Ontology, HAYLuận án: Xử lý tri thức không nhất quán trong Ontology, HAY
Luận án: Xử lý tri thức không nhất quán trong Ontology, HAY
 
Phương pháp xử lý tri thức không nhất quán trong Ontology, HAY
Phương pháp xử lý tri thức không nhất quán trong Ontology, HAYPhương pháp xử lý tri thức không nhất quán trong Ontology, HAY
Phương pháp xử lý tri thức không nhất quán trong Ontology, HAY
 
3 câu thi thử;
3 câu thi thử;3 câu thi thử;
3 câu thi thử;
 
bộ 3 câu hỏi khó phân loại trong đề thi thử THPT Quốc gia môn Toán​ năm 2015
bộ 3 câu hỏi khó phân loại trong đề thi thử THPT Quốc gia môn Toán​ năm 2015bộ 3 câu hỏi khó phân loại trong đề thi thử THPT Quốc gia môn Toán​ năm 2015
bộ 3 câu hỏi khó phân loại trong đề thi thử THPT Quốc gia môn Toán​ năm 2015
 
Luận án: Khái niệm các hệ thống thông tin dựa trên logic mô tả
Luận án: Khái niệm các hệ thống thông tin dựa trên logic mô tảLuận án: Khái niệm các hệ thống thông tin dựa trên logic mô tả
Luận án: Khái niệm các hệ thống thông tin dựa trên logic mô tả
 
Luận văn: Vận dụng quy trình mô hình hoá vào dạy học số tự nhiên ở tiểu học
Luận văn: Vận dụng quy trình mô hình hoá vào dạy học số tự nhiên ở tiểu họcLuận văn: Vận dụng quy trình mô hình hoá vào dạy học số tự nhiên ở tiểu học
Luận văn: Vận dụng quy trình mô hình hoá vào dạy học số tự nhiên ở tiểu học
 
Luận văn: Vận dụng quy trình mô hình hoá vào dạy học số tự nhiên
Luận văn: Vận dụng quy trình mô hình hoá vào dạy học số tự nhiênLuận văn: Vận dụng quy trình mô hình hoá vào dạy học số tự nhiên
Luận văn: Vận dụng quy trình mô hình hoá vào dạy học số tự nhiên
 
Đề tài: Đại cương về không gian Vec-tơ tô-pô, HAY - Gửi miễn phí qua zalo=> 0...
Đề tài: Đại cương về không gian Vec-tơ tô-pô, HAY - Gửi miễn phí qua zalo=> 0...Đề tài: Đại cương về không gian Vec-tơ tô-pô, HAY - Gửi miễn phí qua zalo=> 0...
Đề tài: Đại cương về không gian Vec-tơ tô-pô, HAY - Gửi miễn phí qua zalo=> 0...
 
Xây dựng hệ thống câu hỏi nêu vấn đề trong dạy học hai tác phẩm "Tràng Giang"...
Xây dựng hệ thống câu hỏi nêu vấn đề trong dạy học hai tác phẩm "Tràng Giang"...Xây dựng hệ thống câu hỏi nêu vấn đề trong dạy học hai tác phẩm "Tràng Giang"...
Xây dựng hệ thống câu hỏi nêu vấn đề trong dạy học hai tác phẩm "Tràng Giang"...
 
Luận văn: Tương đương bảo giác giữa các miền n-Liên trong mặt phẳng phức
Luận văn: Tương đương bảo giác giữa các miền n-Liên trong mặt phẳng phứcLuận văn: Tương đương bảo giác giữa các miền n-Liên trong mặt phẳng phức
Luận văn: Tương đương bảo giác giữa các miền n-Liên trong mặt phẳng phức
 
Giáo trình Hóa phân tích (Dùng cho Sinh viên chuyên hóa).pdf
Giáo trình Hóa phân tích (Dùng cho Sinh viên chuyên hóa).pdfGiáo trình Hóa phân tích (Dùng cho Sinh viên chuyên hóa).pdf
Giáo trình Hóa phân tích (Dùng cho Sinh viên chuyên hóa).pdf
 
Luận án tiến sĩ toán học một số hệ phương trình cặp trong cơ học chất lỏng
Luận án tiến sĩ toán học một số hệ phương trình cặp trong cơ học chất lỏngLuận án tiến sĩ toán học một số hệ phương trình cặp trong cơ học chất lỏng
Luận án tiến sĩ toán học một số hệ phương trình cặp trong cơ học chất lỏng
 
Luận án tiến sĩ toán học ngưỡng chính tắc của hàm chỉnh hình và hàm đa điều h...
Luận án tiến sĩ toán học ngưỡng chính tắc của hàm chỉnh hình và hàm đa điều h...Luận án tiến sĩ toán học ngưỡng chính tắc của hàm chỉnh hình và hàm đa điều h...
Luận án tiến sĩ toán học ngưỡng chính tắc của hàm chỉnh hình và hàm đa điều h...
 
Luận án: Giải bất đẳng thức biến phân trên tập điểm bất động, HAY
Luận án: Giải bất đẳng thức biến phân trên tập điểm bất động, HAYLuận án: Giải bất đẳng thức biến phân trên tập điểm bất động, HAY
Luận án: Giải bất đẳng thức biến phân trên tập điểm bất động, HAY
 
Giao trinh toan roi rac toan tap
Giao trinh toan roi rac   toan tapGiao trinh toan roi rac   toan tap
Giao trinh toan roi rac toan tap
 
Kỹ thuật biến đổi tâm tỷ cự và ứng dụng vào giải Toán.doc
Kỹ thuật biến đổi tâm tỷ cự và ứng dụng vào giải Toán.docKỹ thuật biến đổi tâm tỷ cự và ứng dụng vào giải Toán.doc
Kỹ thuật biến đổi tâm tỷ cự và ứng dụng vào giải Toán.doc
 
Đề tài: Cực trị và ứng dụng trong đo lường rủi ro tài chính, HAY, 9đ
Đề tài: Cực trị và ứng dụng trong đo lường rủi ro tài chính, HAY, 9đĐề tài: Cực trị và ứng dụng trong đo lường rủi ro tài chính, HAY, 9đ
Đề tài: Cực trị và ứng dụng trong đo lường rủi ro tài chính, HAY, 9đ
 
Phát triển năng lực giải quyết vấn đề cho học sinh thông qua dạy học phần hiđ...
Phát triển năng lực giải quyết vấn đề cho học sinh thông qua dạy học phần hiđ...Phát triển năng lực giải quyết vấn đề cho học sinh thông qua dạy học phần hiđ...
Phát triển năng lực giải quyết vấn đề cho học sinh thông qua dạy học phần hiđ...
 
Luận văn thạc sĩ sư phạm.
Luận văn thạc sĩ sư phạm.Luận văn thạc sĩ sư phạm.
Luận văn thạc sĩ sư phạm.
 

More from Thế Giới Tinh Hoa

Cách chụp ảnh công ty đẹp 2019
Cách chụp ảnh công ty đẹp 2019Cách chụp ảnh công ty đẹp 2019
Cách chụp ảnh công ty đẹp 2019Thế Giới Tinh Hoa
 
Bảng báo giá sản phẩm rèm bạch dương
Bảng báo giá sản phẩm rèm bạch dươngBảng báo giá sản phẩm rèm bạch dương
Bảng báo giá sản phẩm rèm bạch dươngThế Giới Tinh Hoa
 
Album sổ mẫu Rèm cửa Bạch Dương
Album sổ mẫu Rèm cửa Bạch DươngAlbum sổ mẫu Rèm cửa Bạch Dương
Album sổ mẫu Rèm cửa Bạch DươngThế Giới Tinh Hoa
 
Cách tắm cho bé vào mùa đông
Cách tắm cho bé vào mùa đôngCách tắm cho bé vào mùa đông
Cách tắm cho bé vào mùa đôngThế Giới Tinh Hoa
 
Giáo trình tự học illustrator cs6
Giáo trình tự học illustrator cs6  Giáo trình tự học illustrator cs6
Giáo trình tự học illustrator cs6 Thế Giới Tinh Hoa
 
Nữ quái sân trườngtruonghocso.com
Nữ quái sân trườngtruonghocso.comNữ quái sân trườngtruonghocso.com
Nữ quái sân trườngtruonghocso.comThế Giới Tinh Hoa
 
Những chàng trai xấu tính nguyễn nhật ánhtruonghocso.com
Những chàng trai xấu tính  nguyễn nhật ánhtruonghocso.comNhững chàng trai xấu tính  nguyễn nhật ánhtruonghocso.com
Những chàng trai xấu tính nguyễn nhật ánhtruonghocso.comThế Giới Tinh Hoa
 
Những bài văn mẫu dành cho học sinh lớp 10truonghocso.com
Những bài văn mẫu dành cho học sinh lớp 10truonghocso.comNhững bài văn mẫu dành cho học sinh lớp 10truonghocso.com
Những bài văn mẫu dành cho học sinh lớp 10truonghocso.comThế Giới Tinh Hoa
 

More from Thế Giới Tinh Hoa (20)

Cách chụp ảnh công ty đẹp 2019
Cách chụp ảnh công ty đẹp 2019Cách chụp ảnh công ty đẹp 2019
Cách chụp ảnh công ty đẹp 2019
 
Lỗi web bachawater
Lỗi web bachawaterLỗi web bachawater
Lỗi web bachawater
 
Bảng báo giá sản phẩm rèm bạch dương
Bảng báo giá sản phẩm rèm bạch dươngBảng báo giá sản phẩm rèm bạch dương
Bảng báo giá sản phẩm rèm bạch dương
 
Album sổ mẫu Rèm cửa Bạch Dương
Album sổ mẫu Rèm cửa Bạch DươngAlbum sổ mẫu Rèm cửa Bạch Dương
Album sổ mẫu Rèm cửa Bạch Dương
 
thong tin lam viec tren lamchame
thong tin lam viec tren lamchamethong tin lam viec tren lamchame
thong tin lam viec tren lamchame
 
Cách tắm cho bé vào mùa đông
Cách tắm cho bé vào mùa đôngCách tắm cho bé vào mùa đông
Cách tắm cho bé vào mùa đông
 
Giáo trình tự học illustrator cs6
Giáo trình tự học illustrator cs6  Giáo trình tự học illustrator cs6
Giáo trình tự học illustrator cs6
 
Nang luc truyen thong
Nang luc truyen thongNang luc truyen thong
Nang luc truyen thong
 
Huongdansudung izishop
Huongdansudung izishopHuongdansudung izishop
Huongdansudung izishop
 
Ho so nang luc cong ty
Ho so nang luc cong tyHo so nang luc cong ty
Ho so nang luc cong ty
 
seo contract
seo contractseo contract
seo contract
 
di google cong
di google congdi google cong
di google cong
 
E1 f4 bộ binh
E1 f4 bộ binhE1 f4 bộ binh
E1 f4 bộ binh
 
E2 f2 bộ binh
E2 f2 bộ binhE2 f2 bộ binh
E2 f2 bộ binh
 
E3 f1 bộ binh
E3 f1 bộ binhE3 f1 bộ binh
E3 f1 bộ binh
 
E2 f1 bộ binh
E2 f1 bộ binhE2 f1 bộ binh
E2 f1 bộ binh
 
E1 f1 bộ binh
E1 f1 bộ binhE1 f1 bộ binh
E1 f1 bộ binh
 
Nữ quái sân trườngtruonghocso.com
Nữ quái sân trườngtruonghocso.comNữ quái sân trườngtruonghocso.com
Nữ quái sân trườngtruonghocso.com
 
Những chàng trai xấu tính nguyễn nhật ánhtruonghocso.com
Những chàng trai xấu tính  nguyễn nhật ánhtruonghocso.comNhững chàng trai xấu tính  nguyễn nhật ánhtruonghocso.com
Những chàng trai xấu tính nguyễn nhật ánhtruonghocso.com
 
Những bài văn mẫu dành cho học sinh lớp 10truonghocso.com
Những bài văn mẫu dành cho học sinh lớp 10truonghocso.comNhững bài văn mẫu dành cho học sinh lớp 10truonghocso.com
Những bài văn mẫu dành cho học sinh lớp 10truonghocso.com
 

Một số bất đẳng thức hình học luận văn của thầy hoàng ngọc quang

  • 1. ĐẠI HỌC THÁI NGUYÊN TRƯỜNG ĐẠI HỌC KHOA HỌC Hoàng Ngọc Quang MỘT SỐ BẤT ĐẲNG THỨC HÌNH HỌC Chuyên Nghành: PHƯƠNG PHÁP TOÁN SƠ CẤP MÃ SỐ: 60.46.40 LUẬN VĂN THẠC SĨ TOÁN HỌC Người hướng dẫn khoa học: TS. Nguyễn Văn Ngọc Thái Nguyên - 2011 www.MATHVN.com - HOANG NGOC QUANG, Yen Bai
  • 2. Công trình được hoàn thành tại Trường Đại học Khoa học - Đại học Thái Nguyên Người hướng dẫn khoa học: TS. Nguyễn Văn Ngọc Phản biện 1: . . . . . . . . . . . . . . . . . . . . . . . . . . . . . . . . . . . . . . . . . . . . . . . . . . . . . . . .................................................................... Phản biện 2: . . . . . . . . . . . . . . . . . . . . . . . . . . . . . . . . . . . . . . . . . . . . . . . . . . . . . . . .................................................................... Luận văn sẽ được bảo vệ trước hội đồng chấm luận văn họp tại: Trường Đại học Khoa học - Đại học Thái Nguyên Ngày .... tháng .... năm 2011 Có thể tìm hiểu tại Thư viện Đại học Thái Nguyên www.MATHVN.com - HOANG NGOC QUANG, Yen Bai
  • 3. 1 Mục lục Mục lục . . . . . . . . . . . . . . . . . . . . . . . . . . . . . 1 Mở đầu . . . . . . . . . . . . . . . . . . . . . . . . . . . . . . 3 Chương 1. Các bất đẳng thức trong tam giác và tứ giác 6 1.1. Các bất đẳng thức đại số cơ bản . . . . . . . . . . . . . . 6 1.2. Các đẳng thức và bất đẳng thức cơ bản trong tam giác . 8 1.2.1. Các đẳng thức cơ bản trong tam giác . . . . . . . 8 1.2.2. Các bất đẳng thức cơ bản trong tam giác . . . . . 10 1.3. Bất đẳng thức trong tam giác . . . . . . . . . . . . . . . 11 1.3.1. Bất đẳng thức về độ dài các cạnh . . . . . . . . . 11 1.3.2. Bất đẳng thức về các đại lượng đặc biệt . . . . . 14 1.4. Các bất đẳng thức sinh ra từ các công thức hình học . . 17 1.5. Bất đẳng thức trong các tam giác đặc biệt . . . . . . . . 23 1.5.1. Các bất đẳng thức trong tam giác đều . . . . . . 23 1.5.2. Các bất đẳng thức trong tam giác vuông và tam giác cân . . . . . . . . . . . . . . . . . . . . . . . 27 1.6. Các bất đẳng thức khác trong tam giác . . . . . . . . . . 29 1.7. Các bất đẳng thức trong tứ giác . . . . . . . . . . . . . . 40 1.7.1. Các bất đẳng thức cơ bản trong tứ giác . . . . . . 41 1.7.2. Các bất đẳng thức khác trong tứ giác . . . . . . . 45 Chương 2. Bất đẳng thức Ptolemy và các mở rộng 48 2.1. Định lí Ptolemy . . . . . . . . . . . . . . . . . . . . . . . 48 2.2. Bất đẳng thức Ptolemy . . . . . . . . . . . . . . . . . . . 53 2.3. Định lí Bretschneider . . . . . . . . . . . . . . . . . . . . 63 2.4. Định lí Casey . . . . . . . . . . . . . . . . . . . . . . . . 63 2.5. Mở rộng bất đẳng thức Ptolemy trong không gian . . . . 68 www.MATHVN.com - HOANG NGOC QUANG, Yen Bai
  • 4. 2 Chương 3. Bất đẳng thức Erdos-Mordell và các mở rộng 70 3.1. Bất đẳng thức Erdos-Mordell trong tam giác . . . . . . . 70 3.2. Bất đẳng thức Erdos-Mordell trong tam giác mở rộng . . 79 3.3. Mở rộng bất đẳng thức Erdos-Mordell trong tứ giác . . . 85 3.4. Mở rộng bất đẳng thức Erdos-Mordell trong đa giác . . . 87 3.5. Mở rộng bất đẳng thức Erdos-Mordell trong tứ diện . . . 90 Chương 4. Các bất đẳng thức có trọng 92 4.1. Bất đẳng thức dạng Hayashi và các hệ quả . . . . . . . . 92 4.1.1. Bất đẳng thức Hayashi . . . . . . . . . . . . . . . 92 4.1.2. Các hệ quả của bất đẳng thức hyashi . . . . . . . 94 4.1.3. Bài toán áp dụng . . . . . . . . . . . . . . . . . . 94 4.2. Bất đẳng thức Weizenbock suy rộng và các hệ quả . . . 96 4.2.1. Bất đẳng thức Weizenbock suy rộng . . . . . . . 96 4.2.2. Các hệ quả của bất đẳng thức Weizenbock suy rộng101 4.3. Bất đẳng thức Klamkin và các hệ quả . . . . . . . . . . 105 4.3.1. Bất đẳng thức Klamkin . . . . . . . . . . . . . . 105 4.3.2. Các hệ quả của bất đẳng thức Klamkin . . . . . . 106 4.4. Bất đẳng thức Jian Liu và các hệ quả . . . . . . . . . . 108 4.4.1. Bất đẳng thức Jian Liu . . . . . . . . . . . . . . . 108 4.4.2. Các hệ quả của bất đẳng thức Jian Liu . . . . . . 110 Kết luận . . . . . . . . . . . . . . . . . . . . . . . . . . . . . 116 Tài liệu tham khảo . . . . . . . . . . . . . . . . . . . . . . . 117 www.MATHVN.com - HOANG NGOC QUANG, Yen Bai
  • 5. 3 Mở đầu Các bài toán về bất đẳng thức và cực trị hình học thuộc loại những bài toán khó, làm cho học sinh phổ thông, nhất là phổ thông cơ sở kể cả học sinh giỏi lúng túng khi gặp các bài toán loại này. Thực sự nó là một phần rất quan trọng của hình học và những kiến thức về bất đẳng thức trong hình học cũng làm phong phú hơn phạm vi ứng dụng của toán học. So với các bất đẳng thức đại số, các bất đẳng thức hình học chưa được quan tâm nhiều. Một trong những nguyên nhân gây khó giải quyết vấn đề này là vì phương pháp tiếp cận không phải là các phương pháp thông thường hay được áp dụng trong hình học và càng không phải là phương pháp đại số thuần túy. Để giải một bài toán về bất đẳng thức hình học cần thiết phải biết vận dụng các kiến thức hình học và đại số một cách thích hợp và nhạy bén. Luận văn này giới thiệu một số bất đẳng thức hình học từ cơ bản đến nâng cao và mở rộng. Các bài toán về bất đẳng thức hình học được trình bày trong luận văn này có thể tạm phân thành các nhóm sau: I. Nhóm các bài toán mà trong lời giải đòi hỏi nhất thiết phải có hình vẽ. Phương pháp giải các bài toán nhóm này chủ yếu là "phương pháp hình học", như vẽ thêm đường phụ, sử dụng tính chất giữa đường vuông góc và đường xiên, giữa đường thẳng và đường gấp khúc, quan hệ giữa các cạnh, giữa cạnh và góc trong một tam giác, hay tứ giác v.v.. Bất đẳng thức và cực trị trong hình học phẳng thuộc nhóm này là nội dung thường gặp trong các kì thi chọn học sinh giỏi toán hay thi vào các trường chuyên. II. Nhóm thứ hai gồm các bài toán mà khi giải chúng cần phải sử dụng các hệ thức lượng đã biết, như các hệ thức lượng giác, hệ thức đường trung tuyến, đường phân giác, công thức các bán kính, công thức www.MATHVN.com - HOANG NGOC QUANG, Yen Bai
  • 6. 4 diện tích của tam giác v.v.. Các bài toán này đã được quan tâm nhiều và chúng được trình bày khá phong phú trong các tài liệu [4,7], vì thế luận văn này sẽ không đề cập nhiều đến các bất đẳng thức trong tam giác có trong các tài liệu trên mặc dù chúng rất hay mà chỉ nêu ra một số bất đẳng thức cơ bản nhất để tiện sử dụng sau này. III. Nhóm thứ ba gồm các bài toán liên quan đến các bất đẳng thức hình học nổi tiếng, đặc biệt là bất đẳng thức Ptolemy và bất đẳng thức Erdos-Mordell và các bất đẳng thức có trọng như bất đẳng thức Hayshi, bất đẳng thức Weizenbock, bất đẳng thức Klamkin v.v.. Các bất đẳng thức này còn ít được giới thiệu bằng Tiếng Việt và thường gặp trong các đề thi Olympic Quốc tế. Bản luận văn "Một số bất đẳng thức hình học" gồm có mở đầu, bốn chương nội dung, kết luận và tài liệu tham khảo. Chương 1. Các bất đẳng thức trong tam giác và tứ giác. Chương này trình bày một số bất đẳng thức thuộc nhóm I và nhóm II. Chương 2. Bất đẳng thức Ptolemy và các mở rộng. Chương này trình bày đẳng thức Ptolemy, bất đẳng thức Ptolemy và các bài toán áp dụng. Các bài toán này chủ yếu được trích ra từ các đề thi vô địch các nước, đề thi vô địch khu vực và đề thi IMO, một số là do tác giả sáng tác. Ngoài ra, còn trình bày một số mở rộng bất đẳng thức Ptolemy trong tứ giác và trong tứ diện. Chương 3. Bất đẳng thức Erdos - Mordell và các mở rộng. Chương này trình bày bất đẳng thức Edos-Mordell và các bài toán liên quan. Ngoài ra, còn trình bày một số mở rộng bất đẳng thức này trong tam giác, trong tứ giác và trong đa giác [11-13]. Chương 4. Các bất đẳng thức có trọng. Chương này trình bày một số bất đẳng thức liên quan đến tổng khoảng cách từ một hay nhiều điểm của mặt phẳng đến các đỉnh hoặc các cạnh của tam giác với các tham số dương tùy ý được gọi là trọng số hay gọi tắt là trọng. Đó là các bất đẳng thức Hyashi, Weizenbock, Klamkin, Jian www.MATHVN.com - HOANG NGOC QUANG, Yen Bai
  • 7. 5 Liu, v.v.. Các bất đẳng thức này còn ít được giới thiệu bằng Tiếng Việt, một số là kết quả nghiên cứu của các chuyên gia Quốc tế trong lĩnh vực bất đẳng thức hình học [9,13-14]. Luận văn này được hoàn thành tại trường Đại học Khoa học - Đại học Thái Nguyên với sự hướng dẫn của TS. Nguyễn Văn Ngọc. Tác giả xin được bày tỏ lòng biết ơn sâu sắc đối với sự quan tâm hướng dẫn của Thầy, tới các thầy cô trong Ban Giám hiệu, Phòng Đào tạo và Khoa Toán-Tin Trường Đại học Khoa học. Đồng thời tác giả xin cảm ơn tới Sở GD - ĐT tỉnh Yên Bái, Ban Giám đốc, các đồng nghiệp Trung tâm GDTX - HNDN Hồ Tùng Mậu huyện Lục Yên đã tạo điều kiện cho tác giả học tập và hoàn thành kế hoạch học tập. Thái Nguyên, ngày 20 tháng 6 năm 2011. Tác giả Hoàng Ngọc Quang www.MATHVN.com - HOANG NGOC QUANG, Yen Bai
  • 8. 6 Chương 1 Các bất đẳng thức trong tam giác và tứ giác Chương này trình bày các bất đẳng thức trong tam giác và tứ giác từ cơ bản đến nâng cao. Nội dung chủ yếu được hình thành từ các tài liệu [1-7], [10], [12] và [15]. Kí hiệu ∆ABC là tam giác ABC với các đỉnh là A, B, C. Để thuận tiện, độ lớn của các góc ứng với các đỉnh A, B, C cũng được kí hiệu tương ứng là A, B, C. Độ dài các cạnh của tam giác: BC = a, CA = b, AB = c. a+b+c Nửa chu vi của tam giác: p = . 2 Đường cao với các cạnh: ha , hb , hc . Đường trung tuyến với các cạnh: ma , mb , mc . Đường phân giác với các cạnh: la , lb , lc . Bán kính đường tròn ngoại tiếp và đường tròn nội tiếp: R và r. Bán kính đường tròn bàng tiếp các cạnh: ra , rb , rc . Diện tích tam giác ABC: S, SABC hay [ABC]. Để giải được các bài toán bất đẳng thức hình học, trước hết ta cần trang bị những kiến thức cơ sở đó là các bất đẳng thức đại số cơ bản và các đẳng thức, bất đẳng thức đơn giản trong tam giác. 1.1. Các bất đẳng thức đại số cơ bản Định lý 1.1. (Bất đẳng thức AM-GM) Giả sử a1 , a2 , · · · , an là các số thực không âm. Khi đó a1 + a2 + · · · + an √ ≥ n a1 a2 ...an . (1.1) n Đẳng thức xảy ra khi và chỉ khi a1 = a2 = · · · = an . www.MATHVN.com - HOANG NGOC QUANG, Yen Bai
  • 9. 7 Hệ quả 1.1. Với mọi bộ số dương a1 , a2 , · · · , an ta có √ n n a1 a2 ...an ≥ 1 . (1.2) a1 + a2 + · · · + a1n 1 Đẳng thức xảy ra khi và chỉ khi a1 = a2 = · · · = an . Hệ quả 1.2. Với mọi bộ số dương a1 , a2 , · · · , an ta có 1 1 1 n2 + + ··· + ≥ . (1.3) a1 a2 an a1 + a2 + · · · + an Đẳng thức xảy ra khi và chỉ khi a1 = a2 = ... = an . Hệ quả 1.3. Với mọi bộ số không âm a1 , a2 , · · · , an và m = 1, 2, · · · ta có m am + am + · · · + am 1 2 n a1 + a2 + · · · + an ≥ . (1.4) n n Đẳng thức xảy ra khi và chỉ khi a1 = a2 = · · · = an . Định lý 1.2. (Bất đẳng thức Cauchy - Schwarz) Cho hai dãy số thực a1 , a2 , · · · , an và b1 , b2 , · · · , bn . Khi đó (a1 b1 + a2 b2 + · · · + an bn )2 ≤ a2 + a2 + · · · + a2 1 2 n b2 + b2 + · · · + b2 . 1 2 n (1.5) a1 a2 an Đẳng thức xảy ra khi và chỉ khi b1 = b2 = ··· = bn . Định lý 1.3. (Bất đẳng thức Jensen) Cho f (x) là hàm số liên tục và có đạo hàm cấp hai trên I (a, b) và n điểm x1 , x2 , · · · , xn tùy ý trên đoạn I (a, b). Khi đó i, Nếu f (x) > 0 với mọi x ∈ I (a, b) thì x1 + x2 + · · · + xn f (x1 ) + f (x2 ) + · · · + f (xn ) ≥ nf . n ii, Nếu f (x) < 0 với mọi x ∈ I (a, b) thì x1 + x2 + · · · + xn f (x1 ) + f (x2 ) + · · · + f (xn ) ≤ nf . n Ở đây I (a, b) nhằm ngầm định là một trong bốn tập hợp (a, b) , [a, b) , (a, b] , [a, b]. www.MATHVN.com - HOANG NGOC QUANG, Yen Bai
  • 10. 8 Định lý 1.4. (Bất đẳng thức Chebyshev) Cho hai dãy số thực đơn điệu cùng chiều a1 , a2 , · · · , an và b1 , b2 , · · · , bn . Khi đó ta có 1 a1 b1 + a2 b2 · · · + an bn ≥ (a1 + a2 + · · · + an ) (b1 + b2 + · · · + bn ) . (1.6) n Nếu hai dãy số thực a1 , a2 , · · · , an và b1 , b2 , · · · , bn đơn điệu ngược chiều thì bất đẳng thức trên đổi chiều. Định lý 1.5. (Bất đẳng thức Nesbitt) Cho a, b, c là các số thực dương. Bất đẳng thức sau luôn đúng a b c 3 + + ≥ . (1.7) b+c c+a a+b 2 Đẳng thức xảy ra khi và chỉ khi a = b = c. 1.2. Các đẳng thức và bất đẳng thức cơ bản trong tam giác 1.2.1. Các đẳng thức cơ bản trong tam giác Định lý 1.6. (Định lý hàm số sin) Trong tam giác ABC ta có a b c = = = 2R. sin A sin B sin C Định lý 1.7. (Định lý hàm số cosin) Trong tam giác ABC ta có a2 = b2 + c2 − 2bc cos A, b2 = c2 + a2 − 2ca cos B, c2 = a2 + b2 − 2ab cos C. Định lý 1.8. (Các công thức về diện tích) Diện tích tam giác ABC được tính theo một trong các công thức sau 1 1 1 S = aha = bhb = chc (1.8) 2 2 2 1 1 1 = bc sin A = ca sin B = ab sin C (1.9) 2 2 2 = pr (1.10) abc = (1.11) 4R = (p − a)ra = (p − b)rb = (p − c)rc (1.12) = p (p − a) (p − b) (p − c). (1.13) Công thức (1.13) được gọi là công thức Hê-rông. www.MATHVN.com - HOANG NGOC QUANG, Yen Bai
  • 11. 9 Định lý 1.9. (Định lý đường phân giác) Trong một tam giác, đường phân giác của một góc chia cạnh đối diện thành hai đoạn thẳng tỉ lệ với hai cạnh kề hai đoạn ấy . Định lý 1.10. (Công thức đường phân giác) Trong tam giác ABC ta có 2bc A 2ca B 2ab C la = cos , lb = cos , lc = cos . b+c 2 c+a 2 a+b 2 Định lý 1.11. (Định lý đường trung tuyến) Trong một tam giác, ba đường trung tuyến gặp nhau tại một điểm được gọi là trọng tâm của tam giác. Trên mỗi đường trung tuyến, khoảng cách từ trọng tâm đến đỉnh bằng hai lần khoảng cách trọng tâm đến chân đường trung tuyến. Định lý 1.12. (Công thức đường trung tuyến) Trong tam giác ABC ta có b2 + c2 a2 c2 + a2 b2 a2 + b2 c2 m2 a = − , m2 b = − , m2 c = − . 2 4 2 4 2 4 Định lý 1.13. (Công thức bán kính đường tròn nội tiếp) Trong tam giác ABC ta có A B C r = (p − a) tan = (p − b) tan = (p − c) tan . 2 2 2 Định lý 1.14. (Công thức bán kính đường tròn bàng tiếp) Trong tam giác ABC ta có A B C ra = p tan , rb = p tan , rc = p tan . 2 2 2 Định lý 1.15. (Các hệ thức lượng giác cơ bản) Với mọi tam giác ABC ta luôn có các hệ thức sau A B C sin A + sin B + sin C = 4 cos cos cos , (1.14) 2 2 2 sin 2A + sin 2B + sin 2C = 4 sin A sin B sin C, (1.15) A B C cos A + cos B + cos C = 1 + 4 sin sin sin , (1.16) 2 2 2 cos 2A + cos 2B + cos 2C = −1 − 4 cos A cos B cos C, (1.17) www.MATHVN.com - HOANG NGOC QUANG, Yen Bai
  • 12. 10 sin2 A + sin2 B + sin2 C = 2 (1 + sin A sin B sin C) , (1.18) cos2 A + cos2 B + cos2 C = 1 − 2 cos A cos B cos C, (1.19) tan A + tan B + tan C = tan A tan B tan C, (1.20) A B C A B C cot + cot + cot = cot cot cot , (1.21) 2 2 2 2 2 2 A B B C C A tan tan + tan tan + tan tan = 1, (1.22) 2 2 2 2 2 2 cot A cot B + cot B cot C + cot C cot A = 1. (1.23) Riêng với hệ thức (1.20) thì tam giác ABC cần giả thiết không vuông. 1.2.2. Các bất đẳng thức cơ bản trong tam giác Định lý 1.16. (Bất đẳng thức tam giác) Trong tam giác ABC ta có |b − c| < a < b + c, |c − a| < b < c + a, |a − b| < c < a + b. Định lý 1.17. (Các bất đẳng thức lượng giác cơ bản) Với mọi tam giác ABC ta luôn có các bất đẳng thức sau √ 3 3 sin A + sin B + sin C ≤ , (1.24) 2 3 cos A + cos B + cos C ≤ , (1.25) 2 √ A B C 3 3 cos + cos + cos ≤ , (1.26) 2 2 2 2 A B C 3 sin + sin + sin ≤ , (1.27) 2 2 2 2 A B C 1 sin sin sin ≤ , (1.28) 2 2 2 8 1 cos A cos B cos C ≤ , (1.29) 8 9 sin2 A + sin2 B + sin2 C ≤ , (1.30) 4 A B C √ tan + tan + tan ≥ 3, (1.31) 2 2 2 √ tan A + tan B + tan C ≥ 3 3, (1.32) √ cot A + cot B + cot C ≥ 3. (1.33) www.MATHVN.com - HOANG NGOC QUANG, Yen Bai
  • 13. 11 Riêng với bất đẳng thức (1.32) thì tam giác ABC cần giả thiết không vuông. Đẳng thức xảy ra trong các bất đẳng thức trên khi và chỉ khi ABC là tam giác đều. 1.3. Bất đẳng thức trong tam giác Tam giác là hình đơn giản nhất trong các đa giác, mỗi đa giác bất kì đều có thể chia thành các tam giác và sử dụng tính chất của nó. Vì vậy, nghiên cứu các bất đẳng thức trong tam giác sẽ hữu ích trong việc giải quyết các bất đẳng thức trong đa giác. Trước hết, chúng ta nghiên cứu các bất đẳng thức cơ bản sau đây: 1.3.1. Bất đẳng thức về độ dài các cạnh Định lý 1.18. Cho hai đường tròn có bán kính lần lượt là R và R (R ≥ R ), khoảng cách giữa tâm của chúng bằng d. Điều kiện cần và đủ để hai đường tròn đó cắt nhau là R − R ≤ d ≤ R + R . Chứng minh. Hình 1.1 Hai đường tròn không cắt nhau. Rõ ràng nếu hai đường tròn ở ngoài nhau (hình 1.1 A) thì ta có R + R < d. Nếu hai đường tròn chứa nhau (hình 1.1 B) thì ta cũng có ngay d < R − R . Nếu hai đường tròn cắt nhau tại một điểm M thì theo bất đẳng thức về ba cạnh của tam giác OO M , với O và O lần lượt là tâm của đường tròn bán kính R và R , ta có R − R ≤ d ≤ R + R . Đảo lại, nếu R − R ≤ d ≤ R + R thì hai đường tròn đã cho không thể ngoài nhau hoặc chứa nhau được (nếu không phải có R + R < d hoặc d < R − R ). Do đó chúng chỉ có thể cắt nhau. www.MATHVN.com - HOANG NGOC QUANG, Yen Bai
  • 14. 12 Định lý 1.19. Các số dương a, b, c là độ dài 3 cạnh của một tam giác khi và chỉ khi a + b > c, b + c > a, c + a > b. Chứng minh. Nếu a, b, c là độ dài 3 cạnh của tam giác thì theo bất đẳng thức về 3 cạnh của tam giác ta có a + b > c, b + c > a, c + a > b. Ngược lại, nếu có a, b và c là 3 số thực dương thỏa mãn a+b > c, b+c > a, c + a > b, thì ta có thể chọn hai điểm A và B trên mặt phẳng cách nhau một khoảng c. Lấy A và B làm tâm dựng hai đường tròn bán kính tương ứng là a và b. Từ các bất đẳng thức a + b > c, b + c > a, c + a > b ta có |a − b| < c < a + b. Theo định lý 1.18 thì hai đường tròn tâm A và B phải cắt nhau tại một điểm C. Vậy a, b, c là độ dài các cạnh của tam giác ABC theo cách dựng trên. Định lý 1.20. Cho trước tam giác ABC và một điểm M ở trong tam giác. Khi đó ta có M B + M C < AB + AC. Chứng minh. Kéo dài BM về phía M cắt cạnh AC tại điểm N . Theo định lý 1.19 ta có MB + MC < MB + MN + NC =BN + N C < AB + AN + N C =AB + AC. Hình 1.2 Bài toán 1.1. Cho M là một điểm nằm trong tam giác ABC. Chứng minh rằng p < M A + M B + M C < 2p. Trong đó p là nửa chu vi của tam giác ABC. Giải. Áp dụng định lý 1.19 cho các tam giác M AB, M BC và M CA ta có AB < M A+M B, BC < M B +M C, CA < M C +M A. Cộng theo vế ba bất đẳng thức trên rồi chia cả hai vế cho 2 ta được p < M A+M B +M C. Mặt khác, theo định lý 1.20 ta có M A+M B < CA+CB, M B+M C < AB + AC, M C + M A < BC + BA. Cộng theo vế ba bất đẳng thức trên và đem chia cả hai vế cho 2 ta được AM + BM + CM < 2p. www.MATHVN.com - HOANG NGOC QUANG, Yen Bai
  • 15. 13 Định lý 1.21. Trong một tam giác ứng với góc lớn hơn là cạnh dài hơn và ngược lại. Chứng minh. Xét tam giác ABC. Ta chứng minh nếu ABC > ACB thì AC > AB và ngược lại. Thật vậy, trong góc ABC ta kẻ tia Bx tạo với cạnh BC góc bằng góc ACB. Do ABC > ACB, nên Bx cắt cạnh AC tại điểm D và tạo thành tam giác cân DBC, do đó DB = DC. Mặt khác, trong tam giác ABD ta có AD + DB > AB. Do đó Hình 1.3 AC = AD + DC = AD + DB > AB. Phần ngược lại của định lý là hiển nhiên. Vì nếu ABC < ACB thì ta phải có AC < AB là điều vô lí. Bài toán 1.2. Chứng minh rằng đường vuông góc AH hạ từ điểm A xuống đường thẳng d cho trước luôn nhỏ hơn đường xiên AB. Giải. Tam giác AHB là tam giác vuông tại H, do đó AHB = 900 > ABH. Theo định lý trên, ta có AB > AH. Sự tương ứng giữa độ lớn cạnh và góc còn đúng cho cạnh của hai tam giác khác nhau. Dùng định lý 1.21 ta dễ dàng chứng minh kết quả sau. Định lý 1.22. Cho trước hai tam giác ABC và A B C có hai cặp cạnh bằng nhau AB = A B và AC = A C . Ta có bất đẳng thức BAC > B A C khi và chỉ khi BC > B C . Chứng minh. Trước hết, giả sử rằng BAC > B A C , ta sẽ chứng minh BC > B C . Không mất tính tổng quát giả sử AB ≥ AC. Ta đem hình tam giác ABC đặt chồng lên hình tam giác A B C sao cho A ≡ A , C ≡ C và đỉnh B, B nằm cùng phía so với Hình 1.4 đường thẳng đi qua AC. www.MATHVN.com - HOANG NGOC QUANG, Yen Bai
  • 16. 14 Do AB = A B , nên ta có ABB = AB B. Vì CBB < ABB và CB B > AB B, nên ta có CBB < CB B. Theo định lý 1.21, ta có CB > CB , hay là CB > C B . Nếu như BAC = B A C thì ta cũng dễ dàng thấy rằng BC = B C , do ∆ABC và ∆A B C (c.g.c). Vậy ta có BAC > B A C khi và chỉ khi BC > B C . Bài toán 1.3. Cho tam giác ABC và AM là trung tuyến. Chứng minh 1 rằng BAC ≥ 900 khi và chỉ khi AM ≤ 2 BC. Giải. Gọi A là điểm đối xứng với A qua trung điểm M của cạnh BC. Tứ giác ABA C là tứ giác có hai đường chéo cắt nhau tại trung điểm của mỗi đường nên ABA C là hình bình hành. Xét hai tam giác ABA và ABC có cạnh AB là cạnh chung và có cặp cạnh A B và AC bằng nhau. Theo định lý 1.22, ta có BC ≥ AA khi và chỉ khi BAC ≥ ABA . Do BAC + ABA = 1800 , cho nên BAC ≥ ABA khi và chỉ khi BAC ≥ 900 . Tóm lại, AM = 1 1 2 AA ≤ 2 BC khi và chỉ khi BAC ≥ Hình 1.5 900 . Định lý 1.23. Trong những đường xiên nối một điểm M cho trước với điểm N trên một đường thẳng d cho trước, đường xiên nào có hình chiếu dài hơn thì dài hơn. 1.3.2. Bất đẳng thức về các đại lượng đặc biệt Trong một tam giác, mối quan hệ giữa các cạnh dẫn đến mối quan hệ với các đại lượng đặc biệt. Với đường cao ta dễ thấy là đường cao tương ứng với cạnh lớn hơn thì ngắn hơn. Đối với đường trung tuyến và đường phân giác ta cũng sẽ chứng minh rằng ứng với cạnh dài hơn là đường trung tuyến và đường phân giác ngắn hơn. Định lý 1.24. Trong tam giác ABC ứng với cạnh dài hơn là đường cao, đường trung tuyến và đường phân giác ngắn hơn. www.MATHVN.com - HOANG NGOC QUANG, Yen Bai
  • 17. 15 Chứng minh. Giả sử c < b, ta sẽ chứng minh rằng hb < hc , mb < mc và lb < lc . S S Vì c < b nên suy ra hb = 2b < 2c = hc . Để chứng minh mb < mc , ta gọi M, N và P là trung điểm của các cạnh AB, AC và BC, tương ứng (hình 1.6). Áp dụng định lý 1.22 cho ∆P AB và ∆P AC là hai tam giác có hai cặp cạnh bằng nhau (AP chung và BP = CP ), ta có AP B < AP C. Gọi G Hình 1.6 là trọng tâm của tam giác ABC. Xét hai tam giác GP B và GP C là hai tam giác có hai cặp cạnh bằng nhau (GP chung và P B = P C). Do có AP B < AP C, nên BG < CG. Vậy 3 3 mb = BG < CG = mc . 2 2 Gọi phân giác của góc B là BL và phân giác xuất phát từ C là CK. Theo định lý đường phân giác ta có LC = a ⇒ CL = LA c ab a+c . Tương tự KB = a ⇒ BK = a+b . KA b ac Do c < b, nên BK < CL. Dựng hình bình hành BKCT (hình 1.7), ta có BT C = BKC = A + C và ta có BT C < BLC. 2 Mặt khác, vì T C = BK, và BK < CL nên T C < CL. Trong tam giác T LC, ứng Hình 1.7 với cạnh lớn hơn là góc lớn hơn theo định lí 1.21, cho nên CLT < CT L. Từ các bất đẳng thức BLC > BT C và CLT < CT L, ta có BLT < BT L. Theo định lý 1.21 ta có BT > BL mà CK = BT suy ra CK > BL. Định lý 1.25. Trong tam giác ABC ta luôn có ma ≥ la ≥ ha . Chứng minh. Gọi H là chân đường cao, L là chân đường phân giác và M là chân đường trung tuyến xuất phát từ đỉnh A. Ta chứng minh rằng L nằm trên đoạn thẳng nối HM , và áp dụng định lý 1.23 để có bất đẳng thức cần chứng minh. www.MATHVN.com - HOANG NGOC QUANG, Yen Bai
  • 18. 16 Định lí hiển nhiên đúng cho trường hợp tam giác ABC cân tại đỉnh A. Để tiện chứng minh trong trường hợp tam giác không cân tại A, không mất tính tổng quát ta giả sử AB < AC. Gọi A đối xứng với A qua M , ta có BACA là hình bình hành. Trong tam giác AA C ta có AC > A C = AB và do đó theo định lý 1.21 ta Hình 1.8 có BAM = M A C > CAM và đó điểm L nằm trong góc BAM . Mặt khác, do BAH phụ với góc B và CAH phụ với góc C, cho nên BAH < CAH. Do đó L phải nằm trong góc CAH. Tóm lại, điểm L nằm giữa điểm H và điểm M và ta có HM > HL. Theo định lý 1.23, ta có AH < AL < AM . Định lý 1.26. Đường trung tuyến AM của tam giác ABC nhỏ hơn nửa tổng các cạnh AB và AC cùng xuất phát từ một đỉnh A. Chứng minh. Gọi A là điểm đối xứng với điểm A qua điểm M , ta có 1 1 ABA C là hình bình hành. Do đó AM = AA < (A C + AC) = 2 2 1 (AB + AC). 2 Bài toán 1.4. Chứng minh rằng nếu M là điểm nằm trên đường phân giác ngoài của góc C của tam giác ABC (M khác C) thì M A + M B > CA + CB. Giải. Giả sử A là điểm đối xứng với điểm A qua đường phân giác ngoài của góc C. Khi đó các điểm A , C, B thẳng hàng và M A = M A. Do đó M A + M B = M A + M B > A B = CA + CB = CA + CB. Hình 1.9 www.MATHVN.com - HOANG NGOC QUANG, Yen Bai
  • 19. 17 1.4. Các bất đẳng thức sinh ra từ các công thức hình học Định lý 1.27. (Công thức Euler) Gọi R và r lần lượt là bán kính của đường tròn ngoại tiếp và đường tròn nội tiếp tam giác ABC, d là khoảng cách giữa tâm hai đường tròn đó. Ta có d2 = R2 − 2Rr. (1.34) Chứng minh. Gọi O, I lần lượt là tâm đường tròn ngoại tiếp, nội tiếp ∆ABC. Biết rằng đường tròn ngoại tiếp tam giác BCI có tâm D là trung điểm của cung BC. Gọi M là trung điểm của BC và Q là hình chiếu của I trên OD. Khi đó OB 2 − OI 2 =OB 2 − DB 2 + DI 2 − OI 2 =OM 2 − M D2 + DQ2 − QO2 Hình 1.10 = (M O + DM ) (M O − DM ) + (DQ + QO) (DQ − QO) =DO (M O − DM + DQ + OQ) = R (2M Q) = 2Rr. Vậy OI 2 = R2 − 2Rr, nghĩa là d2 = R2 − 2Rr. Hệ quả 1.4. (Bất đẳng thức Euler) Kí hiệu R, r lần lượt là bán kính đường tròn ngoại tiếp và bán kính đường tròn nội tiếp tam giác ABC. Khi đó R ≥ 2r. (1.35) Đẳng thức xảy ra khi và khi tam giác ABC đều. Bài toán 1.5. Cho tam giác ABC với R là bán kính đường tròn ngoại tiếp, r là bán kính đường tròn nội tiếp và p là nửa chu vi tam giác ABC. p R Chứng minh rằng r ≤ √ ≤ . 3 3 2 www.MATHVN.com - HOANG NGOC QUANG, Yen Bai
  • 20. 18 √ √ Giải. Ta có [ABC] = abc = pr, suy ra 2p = a+b+c ≥ 3 3 abc = 3 3 4Rrp. 4R √ Do đó 8p ≥ 27(4Rrp) ≥ 27(8r2 p), vì R ≥ 2r. Vậy p ≥ 3 3r. 3 p √ Bất đẳng thức thứ hai, 3√3 ≤ R tương đương với a + b + c ≤ 3 3R. 2 Sử dụng định lí hàm số sin, bất đẳng thức này tương đương với sin A + √ 3 3 sin B + sin C ≤ 2 . Bất đẳng thức này đúng vì hàm số f (x) = sin √ làx hàm lồi trên (0, π), do đó sin A+sin B+sin C 3 ≤ sin A+B+C 3 = sin 600 = 23 . Định lý 1.28. (Công thức Leibniz) Cho tam giác ABC với độ dài các cạnh là a, b, c. Gọi G là trọng tâm và (O, R) là đường tròn ngoại tiếp tam giác. Khi đó 1 2 OG2 = R2 − a + b 2 + c2 . (1.36) 9 Chứng minh. Để chứng minh bài toán này ta sử dụng định lí Stewart "Nếu L là điểm nằm trên cạnh BC của ∆ABC và nếu AL = l, BL = m, LC = n, thì a(l2 + mn) = b2 m + c2 n". Áp dụng định lí Stewart cho ∆OAA , trong đó A là trung điểm của BC, ta được AA (OG2 + AG.GA ) = A O2 .AG + AO2 .GA . Vì AO = R, AG = 2 AA , GA = 3 1 2 2 2 2 2 1 2 3 AA nên OG + 9 A A = 3 A O + 3 R . Hình 1.11 2 2(b2 +c2 )−a2 2 2 a2 Mặt khác, vì A A = 4 và A O = R − 4 , ta được 2 a2 2 1 2 2 2(b2 + c2 ) − a2 2 OG = R − + R − 4 3 3 9 4 a2 2(b2 + c2 ) − a2 a2 + b2 + c2 =R2 − − = R2 − . 6 18 9 Hệ quả 1.5. (Bất đẳng thức Leibniz) Cho tam giác ABC với độ dài các cạnh là a, b, c. (O, R) là đường tròn ngoại tiếp tam giác. Ta có bất đẳng thức sau 9R2 ≥ a2 + b2 + c2 . (1.37) Đẳng thức xảy ra khi và chỉ khi O là trọng tâm của tam giác ABC. www.MATHVN.com - HOANG NGOC QUANG, Yen Bai
  • 21. 19 Bài toán 1.6. Cho tam giác ABC với độ dài các cạnh là a, b, c. Chứng √ 9abc minh rằng 4 3 [ABC] ≤ a+b+c . Giải. Sử dụng bất đẳng thức Leibniz với lưu ý 4R. [ABC] = abc ta có a2 b2 c2 2 2 2 9R2 ≥ a2 + b2 + c2 ⇔ 16[ABC]2 ≥ a +b +c ⇔ 4 [ABC] ≤ √a23abc+c2 . Mặt 9 2 √ √ +b khác, Bất đẳng thức Cauchy-Schwarz cho a + b + c ≤ 3 a2 + b2 + c2 . √ 9abc Do đó 4 3 [ABC] ≤ a+b+c . Bài toán 1.7. Giả sử đường tròn nội tiếp tam giác ABC tiếp xúc các cạnh AB, BC, CA tại D, E, F , tương ứng.Kí hiệu p là nửa chu vi của 2 tam giác ABC. Chứng minh rằng EF 2 + F D2 + DE 2 ≤ p3 . Giải. Thấy rằng đường tròn nội tiếp tam giác ABC là đường tròn ngoại tiếp tam giác DEF . Áp dụng bất đẳng thức Leibniz cho tam giác DEF , ta được EF 2 + F D2 + DE 2 ≤ 9r2 . Mặt khác, theo bài toán 1.5 ta có 2 p2 ≥ 27r2 . Do đó EF 2 + F D2 + DE 2 ≤ p3 . Định lý 1.29. (Định lí Euler) Cho tam giác ABC nội tiếp trong đường tròn tâm O, bán kính R. M là một điểm bất kì nằm trong mặt phẳng tam giác. Gọi X, Y, Z lần lượt là hình chiếu vuông góc của M lên các cạnh BC, CA, AB. Khi đó diện tích của tam giác XY Z được tính theo diện tích tam giác ABC và khoảng cách M O bởi công thức sau 1 M O2 [XY Z] = 1− [ABC] . (1.38) 4 R2 Chứng minh. Kéo dài AM, BM, CM cắt đường tròn ngoại tiếp tại các điểm X , Y , Z tương ứng. Ta có ZXM = M BZ (tứ giác BZM X nội tiếp), M BZ = ABY (B, Z, A thẳng hàng và B, M, Y thẳng hàng), ABY = AX Y (cùng chắn cung AY ). Từ đó suy ra ZXM = AX Y . Tương tự Y XM = AX Z . Từ đó suy ra ZXY = ZXY . Ta sẽ kí hiệu hai góc này là X và Hình 1.12 www.MATHVN.com - HOANG NGOC QUANG, Yen Bai
  • 22. 20 X . Ta có 1 [XY Z] = XY.XZ. sin X 2 1 = M C. sin C.M B sin B. sin X (định lí hàm số sin) 2 1 MC = M B.M Y . sin B. sin C. sin X 2 MY 1 BC = M O 2 − R2 . sin B sin C. sin X(phương tích, ∆M BC ∼ ∆M Z Y ) 2 ZY 1 sin X = M O2 − R2 .BC. sin C. sin B. 2 Y Z 2 1 MO AC sin X 1 = 1− AC.BC. sin C (vì sin B = , = ) 8 R2 2R Y Z 2R 1 M O2 = 1− [ABC] . 4 R2 Chú ý 1.1. 1) Tam giác XY Z nêu trong định lí được gọi là tam giác Pedal. 2) Nếu M nằm trên đường tròn ngoại tiếp tam giác ABC thì [XY Z] = 0. Điều đó có nghĩa là tam giác XY Z suy biến thành đường thẳng, đó chính là đường thẳng Euler. 3) Nếu M ≡ I (I là tâm đường tròn nội tiếp tam giác ABC) thì XY Z là tam giác nội tiếp đường tròn tâm I, bán kính r có các góc X, Y, Z tương ứng bằng π − A , π − B , π − C . Bằng các phép biến đổi sơ cấp từ 2 2 2 2 2 2 công thức (1.38) sẽ suy ra công thức Euler OI 2 = R2 − 2Rr. Hệ quả 1.6. Cho tam giác ABC và một điểm M bất kì nằm trong mặt phẳng tam giác. Gọi X, Y, Z lần lượt là hình chiếu vuông góc của M lên các cạnh BC, CA, AB. Khi đó 1 [XY Z] ≤ [ABC] . (1.39) 4 Đẳng thức xảy ra khi và chỉ khi M là tâm của đường tròn ngoại tiếp tam giác ABC. www.MATHVN.com - HOANG NGOC QUANG, Yen Bai
  • 23. 21 Bài toán 1.8. Cho tứ giác lồi ABCD nội tiếp trong đường tròn tâm O (với O nằm bên trong tứ giác). Gọi M N P Q là tứ giác mà các đỉnh lần lượt là hình chiếu của giao điểm 2 đường chéo của tứ giác ABCD đến các cạnh AB, BC, CD, DA. Chứng minh rằng 1 [M N P Q] ≤ [ABCD] . 2 Giải. Gọi K là giao điểm 2 đường chéo AC và BD của tứ giác ABCD. Dễ thấy KM N là tam giác Pedal dựng từ điểm K của tam giác ABC. Do đó áp dụng hệ quả 1.6 ta được [KM N ] ≤ 1 4 [ABC]. Làm tương tự cho các tam giác KN P, KP Q, KQM và cộng các kết quả lại [KM N ] + [KN P ] + [KP Q] + [KQM ] 1 ≤ ([ABC] + [BCD] + [CDA] + [DAB]) . 4 1 Suy ra [M N P Q] ≤ 2 [ABCD]. Hình 1.13 Đẳng thức xảy ra khi và chỉ khi K là tâm của đường tròn ngoại tiếp tứ giác ABCD. Bài toán 1.9. (Balkan, 1999) Cho ABC là một tam giác nhọn và L, M, N là các chân đường cao hạ từ trọng tâm G của ∆ABC tới các cạnh BC, CA, AB, tương ứng. Chứng minh rằng 4 [LM N ] 1 < ≤ . 27 [ABC] 4 Giải. Ta có tam giác LM N là tam giác Pedal dựng từ trọng tâm G của tam giác ABC. Áp dụng định lí 1.12, ta có [LM N ] = 2 2 1 −OG2 4 1 − OG [ABC] = R 4R2 [ABC] (vì G nằm trong tam giác ABC). R 2 1 + Dễ thấy [LM N ] ≤ 4 [ABC]. Đẳng đẳng thức xảy ra khi và chỉ khi G ≡ O hay khi và chỉ khi tam giác ABC đều. + Ta chứng minh bất đẳng thức còn lại. Thật vậy, để ý rằng OG = 1 OH. 3 Vì tam giác nhọn, H nằm trong tam giác và OH ≤ R nên [LM N ] R2 − 1 OH 2 9 R2 − 1 R2 9 2 4 = ≥ = > . [ABC] 4R2 4R2 9 27 www.MATHVN.com - HOANG NGOC QUANG, Yen Bai
  • 24. 22 Định lý 1.30. (Công thức hình bình hành) Cho tứ giác ABCD, gọi x là khoảng cách giữa trung điểm của hai đường chéo AC và BD. Ta có AB 2 + BC 2 + CD2 + DA2 = AC 2 + BD2 + 4x2 . (1.40) Chứng minh. Gọi M, N lần lượt là trung điểm của AC và BD. Áp dụng công thức đường trung tuyến ta có AB 2 +DA2 BD2 BC 2 +CD2 BD2 2 N A2 + N C 2 AC 2 2 − 4 + 2 − 4 AC 2 x = − = − 2 4 2 4 hay AB 2 + BC 2 + CD2 + DA2 = AC 2 + BD2 + 4x2 . Hệ quả 1.7. (Bất đẳng thức hình bình hành) Cho tứ giác ABCD. Ta có AB 2 + BC 2 + CD2 + DA2 ≥ AC 2 + BD2 . (1.41) Đẳng thức xảy ra khi và chỉ khi tứ giác ABCD là hình bình hành. Bài toán 1.10. (Địa trung Hải, 2000) Cho P, Q, R, S là trung điểm của các cạnh BC, CD, DA, AB, tương ứng, của tứ giác lồi ABCD. Chứng minh rằng 4 AP 2 + BQ2 + CR2 + DS 2 ≤ 5 AB 2 + BC 2 + CD2 + DA2 . Giải. Ta biết công thức đường trung tuyến XM của tam giác XY Z là XM 2 = 1 XY 2 + 1 XZ 2 − 1 Y Z 2 . Ta thay bộ (X, Y, Z, M ) bằng 2 2 4 (A, B, C, P ), (B, C, D, Q), (C, D, A, R), (D, A, B, S) vào trong công thức này và cộng 4 công thức lại với nhau để thu được công thức thứ 5. Nhân cả hai vế của công thức thứ 5 với 4, ta được 4 AP 2 + BQ2 + CR2 + DS 2 = AB 2 + BC 2 + CD2 + DA2 + 4 AC 2 + BC 2 . Do đó ta chỉ cần chứng minh bất đẳng thức AC 2 + BC 2 ≤ AB 2 + BC 2 + CD2 + DA2 . Đây là bất đẳng thức hình bình hành. Dấu "=" xảy ra khi và chỉ khi tứ giác ABCD là hình bình hành. Điều phải chứng minh. www.MATHVN.com - HOANG NGOC QUANG, Yen Bai
  • 25. 23 1.5. Bất đẳng thức trong các tam giác đặc biệt 1.5.1. Các bất đẳng thức trong tam giác đều Tam giác đều có một số tính chất đặc biệt, nói chung không còn đúng trong một tam giác tùy ý. Trong mục này, ta chỉ nghiên cứu một số bất đẳng thức trong tam giác đều ABC liên quan mối quan hệ giữa pa , pb , pc với P A, P B, P C (trong đó pa , pb , pc lần lượt là khoảng cách từ P đến các cạnh BC, CA, AB). Bài toán 1.11. Cho ABC là tam giác đều cạnh a, gọi P là một điểm nằm trong tam giác. Chứng minh rằng √ 1 1 1 6 3 i) + + ≥ , pa pb p c a √ 1 1 1 3 3 ii) + + ≥ . pa + pb pb + pc pc + pa a Giải. Gọi D, E, F là chân đường vuông góc của P lên các cạnh BC, CA, AB tương ứng. Ta có [ABC] = [BCP ] + [CAP ] + [ABP ], do đó ah = apa +√ b + apc . Vì √ ap 3 3 h= a nên pa + pb + pc = a. Áp dụng 2 2 bất đẳng thức (1.2), ta được Hình 1.14 √ 1 1 1 9 6 3 + + ≥ = . p a p b pc pa + pb + pc a Lại áp dụng bất đẳng thức (1.2) có √ 1 1 1 9 3 3 + + ≥ = . pa + pb pb + pc pc + pa pa + pb + pb + pc + p c + pa a Bài toán 1.12. Cho tam giác đều ABC cạnh a và P là một điểm tùy ý nằm trong tam giác. Chứng minh rằng 8a3 P A2 .P B 2 .P C 2 ≥ √ pa pb pc . (1.42) 3 3 Trước hết ta chứng minh bổ đề sau www.MATHVN.com - HOANG NGOC QUANG, Yen Bai
  • 26. 24 Bổ đề 1.1. Cho P là một điểm tùy ý nằm trong tam giác đều ABC cạnh a. Kí hiệu α = BP C, β = CP A,γ = AP B, ta có đẳng thức 2 2 2 a3 p a p b p c P A .P B .P C = . (1.43) sin α sin β sin γ Chứng minh. Viết lại diện tích tam giác BP C theo hai cách ta được BP.CP. sin α = a.pa , tương tự CP.AP. sin β = a.pb , AP.BP. sin γ = a.pc . Nhân theo vế 3 đẳng thức này, ta thu được đẳng thức (1.43). Giải. Gọi f (x) = ln(sin x), x ∈ (0, π) .Vì f (x) = − sin12 x < 0, nên f là hàm lồi. Áp dụng bất đẳng thức Jensen, ta có ln(sin α) + ln(sin β) + ln(sin γ) α+β+γ ≤ ln(sin ). 3 3 √ Suy ra sin α. sin β. sin γ ≤ 3 8 3 . Thay bất đẳng thức này vào (1.43) ta được bất đẳng thức (1.42). Bài toán 1.13. Cho tam giác đều ABC cạnh a và P là một điểm tùy ý nằm trong tam giác. Chứng minh rằng P A.P B.P C ≥ 8pa pb pc . (1.44) 1 1 1 Giải. Vì [BP C] + [CP A] + [AP B] = [ABC] nên apa + apb + apc = √ √ 2 2 2 2 √ a 3 a 3 3 3 . Suy ra pa + pb + pc = . Do đó pa pb pc ≤ pa +pb +pc = a 72 3 . 3 4 2 Thay bất đẳng thức này vào (1.42), ta được bất đẳng thức (1.44). Bài toán 1.14. Cho tam giác đều ABC cạnh a và P là một điểm tùy ý nằm trong tam giác. Chứng minh rằng P A.P B + P B.P C + P C.P A ≥ a2 . (1.45) α β γ Giải. Vì 2 + 2 + 2 = 1800 nên 3 cos α + cos β + cos γ ≥ , (1.46) 2 dấu đẳng thức xảy ra khi và chỉ khi α = β = γ = 1200 . Bây giờ, áp dụng định lí hàm số cosin cho tam giác P AB, ta có a2 = www.MATHVN.com - HOANG NGOC QUANG, Yen Bai
  • 27. 25 2 2 2 +P B −a P A2 + P B 2 − 2P A.P B. cos γ hay cos γ = P A2P A.P B . Tương tự cos α = P B 2 +P C 2 −a2 2 +P A2 −a2 2P B.P C , cos β = P C2P C.P A . Cộng theo vế 3 bất đẳng thức trên và sử dụng (1.46), ta được P A2 + P B 2 − a2 P B 2 + P C 2 − a2 P C 2 + P A2 − a2 3 + + + ≥ 0, 2P A.P B 2P B.P C 2P C.P A 2 Do đó P A2 .P B + P B 2 .P A + P A.P B.P C + P C 2 .P B + P B 2 .P C + P A.P B.P C + + P A2 .P C + P C 2 .P A + P A.P B.P C − a2 (P A + P B + P C) ≥ 0, tương đương với (P A + P B + P C) P A.P B + P B.P C + P C.P A − a2 ≥ 0. Từ đó có bất đẳng thức P A.P B + P B.P C + P C.P A ≥ a2 . Dấu đẳng thức xảy ra khi và chỉ khi P ≡ O. Tam giác Pompeiu Năm 1936, nhà toán học Rumani, Dimitrie Pompeiu phát hiện kết quả đơn giản nhưng đẹp sau đây trong hình học phẳng Euclide Định lý 1.31. (Định lý Pompeiu). Cho P là một điểm tùy ý nằm trong mặt phẳng chứa tam giác đều ABC. Khi đó các khoảng cách P A, P B, P C là độ dài các cạnh của một tam giác. Tam giác này suy biến nếu điểm P nằm trên đường tròn ngoại tiếp tam giác ABC. Chứng minh. Xét trường hợp điểm P không nằm trên đường tròn ngoại tiếp tam giác ABC. Áp dụng bất đẳng thức Ptolemy (Xem định lí 2.2) cho 4 điểm A, B, P, C ta có P A.BC < P C.AB + P B.AC, P B.AC < P A.BC + P C.AB, P C.AB < P A.BC + P B.AC. Vì tam giác ABC đều nên AB = BC = Hình 1.15 CA = l, do đó P A < P C + P B, P B < P A + P C, P C < P A + P B. Vậy P A, P B, P C là độ dài 3 cạnh của một tam giác. www.MATHVN.com - HOANG NGOC QUANG, Yen Bai
  • 28. 26 Chú ý 1.2. + Tam giác với độ dài các cạnh bằng P A, P B, P C được gọi là tam giác Pompeiu. + Khi P nằm ở trong tam giác ABC, tam giác Pompeiu có thể được xây dựng một cách dễ dàng như sau: Quay tam giác ABP quanh tâm A, một góc 600 , được tam giác AB C. Khi đó AP = AB = P B , BP = CB , tam giác Pompeiu sẽ là ∆P CB . Bài toán 1.15. Cho tam giác đều ABC cạnh a và P là một điểm tùy ý nằm trong tam giác. Chứng minh rằng √ √ 3Rp 3 ≥ P A + P B + P C ≥ a 3. (1.47) Trong đó Rp là bán kính đường tròn ngoại tiếp tam giác Pompeiu có độ dài các cạnh bằng P A, P B, P C. Giải. Gọi M là trung điểm của BC, theo bất đẳng thức tam giác AP + P M ≥ AM , mặt khác lại có P M ≤ P B+P C . Suy ra 2P A + P B + P C ≥ √ √ 2 √ a 3, tương tự 2P B + P C + P A ≥ a 3, 2P C + P A + P B ≥ a 3. Cộng √ theo vế 3 bất đẳng thức này ta được P A + P B + P C ≥ a 3. Áp dụng P A2 + P B 2 + P C 2 ≤ 9Rp và bất đẳng thức Cauchy-Schwarz, 2 ta có 9Rp ≥ P A2 + P B 2 + P C 2 ≥ 3 (P A + P B + P C)2 . 2 1 √ Suy ra 3Rp 3 ≥ P A + P B + P C. Bài toán 1.16. Cho P là một điểm nằm trong tam giác đều ABC cạnh a. Gọi T là diện tích của tam giác Pompeiu với độ dài các cạnh bằng P A, P B, P C. Khi đó √ 3 2 T = a − 3d2 . (1.48) 12 Trong đó d = OP (O là tâm của tam giác đều ABC). Giải. Ta thực hiện 3 phép quay tương tự như hình 1.15, đó là quay tam giác AP B quanh tâm A một góc 600 , quay tam giác BP C quanh tâm B một góc 600 và quay tam giác CP A quanh tâm C một góc 600 . Ta sẽ được một hình lục giác AB CA BC , trong đó các tam giác Pompeiu P BA , P AC , P CB có cùng diện tích T . www.MATHVN.com - HOANG NGOC QUANG, Yen Bai
  • 29. 27 Vì ∆AP C = ∆BA C, ∆AP B = ∆AB C, ∆AC B = ∆BP C nên √ tích diện 2 2a 3 của lục giác = 2. [ABC] = . Mặt 4 khác ∆AP B , BP C , CP A là các tam giác đều. Do đó √ √ √ √ 2a2 3 P A2 3 P B 2 3 P C 2 3 = 3T + + + 4 4 4 4 Luôn có P A2 +P B 2 +P C 2 = 3OP 2 +OA2 +OB 2 +OC 2 √ a2 3 Hình 1.16 Vì OP = d, OA = OB = OC = nên 3 P A2 + P B 2 + P C 2 = 3d2 + a2 . Từ đó ta được công thức (1.48). Nhận xét 1.1. Từ công thức (1.47) và (1.48) ta có các hệ quả sau: √ 3 2 1) T ≤ a . Đẳng thức xảy ra khi và chỉ khi P ≡ O. 12 R 2) rp ≤ a ≤ 2p , trong đó Rp , rp lần lượt là bán kính vòng tròn ngoại 6 tiếp, nội tiếp tam giác Pompeiu với độ dài các cạnh bằng P A, P B, P C. 1.5.2. Các bất đẳng thức trong tam giác vuông và tam giác cân Bài toán 1.17. Cho góc vuông xAy. B là điểm trên tia Ax, C là điểm √ trên tia Ay (B = A; C = A). Chứng minh rằng AB + 3AC ≤ 2BC. Giải. Trong góc xAy vẽ tia Az sao cho xAz = 300 , do đó yAz = 600 . Vẽ BH⊥Az, CK⊥Az(H, K ∈ Az), Az cắt BC tại I. Xét ∆ABH có AHB = 900 , BAH = 300 nên là nửa tam giác đều, cạnh AB. Suy ra BH = 1 AB mà BH ≤ 2 BI. Do đó Hình 1.17 AB ≤ 2BI. (1.49) www.MATHVN.com - HOANG NGOC QUANG, Yen Bai
  • 30. 28 Xét ∆ACK có AKC = 900 , CAK = 600 nên là nửa tam giác đều, cạnh √ AC. Suy ra CK = AC2 3 mà CK ≤ IC. Do đó √ 3AC ≤ 2IC. (1.50) √ Cộng theo vế (1.49) và (1.50) được AB + 3AC ≤ (BI + CI) = 2BC. Bài toán 1.18. Cho tam giác ABC cân tại A. D là điểm trên cạnh BC, E là điểm trên trên tia đối của tia CB sao cho CE = BD. Chứng minh rằng AD + AE > 2AB. Giải. Trên tia đối của tia BC lấy điểm F sao cho BF = BD. Áp dụng định lí 1.26 ta có AD + AF > 2AB. Mặt khác, xét ∆ABF và ∆ACE có AB = AC, ABF = ACE (vì ABC = ACB, ABC + ABF = ACE + ACB = 1800 ), BF = CE. Do đó ∆ABF = ∆ACE (c.g.c). Suy ra AF = AE. Vậy AD + AE > 2AB. Hình 1.18 Bài toán 1.19. (Iran, 2005) Cho tam giác ABC vuông tại A. Gọi D là giao điểm của phân giác trong của góc A với cạnh BC và Ia là tâm của đường tròn bàng tiếp cạnh BC của tam giác ABC. Chứng minh rằng AD √ ≤ 2 − 1. DIa Giải. Gọi E, F lần lượt là điểm tiếp xúc của đường tròn bàng tiếp với các cạnh AB, AC tương ứng. Ta có ra = Ia E = AF = F Ia = p, trong đó p là nửa chu vi của tam giác ABC. Hơn nữa, DIa = ha . Vì aha = bc, AD r a AD ha bc bac 4Rr 1 ta có DIa = ra = ap = 4R a2 rp = 4Rr a2 . Vì 2R = a và 2r = b + c − a nên Hình 1.19 www.MATHVN.com - HOANG NGOC QUANG, Yen Bai
  • 31. 29 AD b+c−a b+c √ (b+c)2 b+c DIa = a − 1. Mặt khác a = = a b2 + c2 ≥ 2 = √ hay √ AD √ 2 b + c ≤ a 2. Do đó DIa ≤ 2 − 1. Bài toán 1.20. (Rumani, 2007) Cho ABC là một tam giác vuông cân tại A. Với điểm P tùy ý nằm trong tam giác, xét đường tròn tâm A và bán kính AP cắt các cạnh AB và AC tại M và N , tương ứng. Hãy xác định vị trí của P để M N + BP + CP đạt giá trị nhỏ nhất. Giải. Xét điểm Q trên đường trung trực của BC thỏa mãn AQ = AP . Gọi S là giao điểm của BP và tiếp tuyến với đường tròn tại Q. Khi đó SP + P C ≥ SC. Do đó BP + P C = BS + SP + P C ≥ BS + SC. Mặt khác, BS + SC ≥ BQ + QC, nên BP + P C đạt giá trị nhỏ nhất nếu P ≡ Q. Gọi T là trung điểm của M N . Vì ∆AM Q cân và M T là một trong những chiều cao của nó, khi đó M T = Hình 1.20 ZQ trong đó Z là chân đường vuông góc hạ từ Q xuống AB. Khi đó M N + BQ + QC = 2(M T + QC) = 2(ZQ + QC) đạt giá trị nhỏ nhất khi Z, Q, C thẳng hàng và điều này có nghĩa CZ là chiều cao. Bằng phép đối xứng, BQ cũng là chiều cao và do đó P là trực tâm. 1.6. Các bất đẳng thức khác trong tam giác Bài toán 1.21. (Bất đẳng thức Weitzenbock, IMO 1961) Gọi a, b, c là độ dài các cạnh của một tam giác. Chứng minh rằng √ a2 + b2 + c2 ≥ 4 3S. (1.51) Giải. Theo bất đẳng thức AM - GM ta có √ (p−a)+(p−b)+(p−c) 3 3 2 S= p(p − a)(p − b)(p − c) ≤ p 3 = p. 9 www.MATHVN.com - HOANG NGOC QUANG, Yen Bai
  • 32. 30 Do đó, theo bất đẳng thức Cauchy - Schwarz ta có √ 2 √ √ 3 a+b+c 1 4 3S ≤ 4 3 = (a + b + c)2 9 2 3 1 2 ≤ a + b2 + c2 12 + 12 + 12 = a2 + b2 + c2 . 3 Bài toán sau cho ta một kết quả mạnh hơn bất đẳng thức Weitzenbock. Bài toán 1.22. (Bất đẳng thức Hadwiger - Finsler) Gọi a, b, c là độ dài các cạnh của một tam giác. Chứng minh rằng √ a2 + b2 + c2 ≥ 4 3S + (a − b)2 + (b − c)2 + (c − a)2 (1.52) Đẳng thức xảy ra khi và chỉ khi a = b = c. 1 Giải. Vì a2 = b2 + c2 − 2bccosA và S = 2 bcsinA nên ta có √ √ a2 + b2 + c2 − 4 3S = 2(b2 + c2 ) − 2bccosA − 2 3bcsinA π =2(b2 + c2 ) − 4bccos − A ≥ 2(b2 + c2 − 2bc) = 2 (b − c)2 . (1.53) 3 Mặt khác, không mất tính tổng quát ta giả sử b ≤ a ≤ c. Khi đó ta có bất đẳng thức (b − c)2 ≥ (a − b)2 + (c − a)2 . (1.54) Vì bất đẳng thức trên tương đương với bất đẳng thức (a − b)(a − c) ≤ 0. Từ bất đẳng thức (1.53) và (1.54) ta được bất đẳng thức (1.52).  cos π − A = 1  3 Đẳng thức xảy ra khi và chỉ khi a=b ⇔ a = b = c. a=c  Bài toán 1.23. (Điểm Torricelli) Tìm điểm O trong tam giác ABC cho trước sao cho tổng khoảng cách từ điểm O tới ba đỉnh của tam giác là nhỏ nhất có thể. Giải. Xét làm hai trường hợp: a) Tam giác ABC có ba góc nhỏ hơn 1200 . www.MATHVN.com - HOANG NGOC QUANG, Yen Bai
  • 33. 31 Dựng tam giác đều BCD ở phía ngoài của tam giác ABC. Gọi T là giao điểm của đường tròn ngoại tiếp tam giác BCD với AD. Dễ chứng minh rằng T nhìn ba cạnh của tam giác ABC dưới ba góc bằng nhau. Ta chứng minh rằng với một điểm O tùy ý ở trong tam giác ABC khác điểm T thì ta có OA + OB + OC ≥ T A + T B + T C. Điểm T được gọi là điểm Torricelli của tam giác ABC và có tổng các khoảng cách tới các đỉnh của tam giác ABC nhỏ nhất. Thật vậy, theo định lí Pompeiu, ta có OB + OC ≥ OD, do đó Hình 1.21 OA + OB + OC ≥ OA + OD ≥ AD. (1.55) Mặt khác, vì T nằm trên đường tròn ngoại tiếp tam giác đều BCD nên T A + T B + T C = T A + T D = AD. (1.56) Từ (1.55) và (1.56) suy ra OA + OB + OC ≥ T A + T B + T C. Đẳng thức xảy ra khi và chỉ khi O ≡ T . b) Tam giác ABC có một góc, chẳng hạn B > 1200 Dựng tam giác đều BCD ở phía ngoài tam giác ABC. Do B > 1200 , cho nên với điểm O tùy ý ở trong tam giác ABC, điểm B nằm trong tam giác ODA. Theo định lí Pompeiu, ta có OB + OC ≥ OD. Mặt khác, theo định lí 1.20 đối với tam Hình 1.22 giác ODA, ta có OA + OD ≥ BA + BD. Từ đó ta có OA + OB + OC ≥ OA + OD ≥ BA + BD = BA + BC. Như vậy, khi O ≡ B, tổng khoảng cách từ O tới các đỉnh của tam giác ABC là nhỏ nhất có thể. Tóm lại, trong trường hợp tam giác ABC có một đỉnh không nhỏ hơn 1200 , thì chính đỉnh này là đỉnh cần tìm. www.MATHVN.com - HOANG NGOC QUANG, Yen Bai
  • 34. 32 Bài toán 1.24. Cho tam giác ABC nội tiếp đường tròn (O) với H là trực tâm và AD, BE, CE là các đường cao. Kí hiệu D = AD ∩ (O), E = BE ∩ (O), F = CF ∩ (O). Chứng minh rằng AD BE CF HD HE HF 3 (i) + + ≥ 9, (ii) + + ≥ , HD HE HF HA HB HC 2 AD BE CF AD BE CF 9 (iii) + + ≥ 9, (iv) + + ≥ . DD EE FF AD BE CF 4 Giải. + Chứng minh i). Ta có [HBC] = HD , [HCA] = HE , [HAB] = [ABC] AD [ABC] BE [ABC] HF HD HE HF CF nên AD + BE + CF = 1. Bây giờ, áp AD dụng bất đẳng thức (1.2) ta được HD + BE CF 9 HE + HF ≥ HD + HE + HF = 9. AD BE CF + Chứng minh ii). [HBC] Ta có HD = AD−HD = HA HD [ABC]−[HBC] = [HBC] HE [HCA] [HCA]+[HAB] . tương tự, HB = [HAB]+[HBC] , HF [HAB] HC = [HBC]+[HCA] . Cộng 3 bất đẳng thức này, sau đó áp dụng bất đẳng thức Nesbitt ta được bất đẳng thức HD + HB + HC ≥ 3 . HA HE HF 2 Hình 1.23 + Chứng minh iii). Dễ chứng minh được HD = DD , HE = EE và HF = F F . Thay vào AD BE CF (i) ta được bất đẳng thức DD + EE + F F ≥ 9. + Chứng minh iv). Ta có AD = AD+DD = 1 + HD . Tương tự, BE = 1 + HE , CF = 1 + HF . AD AD AD BE BE CF CF AD BE CF HD HE HF Cộng 3 đẳng thức này ta được AD + BE + CF = 3 + AD + BE + CF = AD BE CF 4. Bây giờ áp dụng bất đẳng thức (1.2) ta được AD + BE + CF ≥ 9 AD + BE + CF = 9. 4 AD BE CF Bài toán 1.25. (Tạp chí THTT số 266) Giả sử M là điểm nằm trong tam giác ABC. Gọi A1 , B1 , C1 lần lượt là hình chiếu của M trên các đường thẳng BC, CA, AB. Chứng minh rằng M A2 M B2 M C2 + + ≥ 3. (M B1 + M C1 )2 (M C1 + M A1 )2 (M A1 + M B1 )2 www.MATHVN.com - HOANG NGOC QUANG, Yen Bai
  • 35. 33 Đẳng thức xảy ra khi nào ? Giải. Ta có M B1 + M C1 M B1 M C1 = + MA MA MA = sin M AB1 + sin M AC1 M AB1 + M AC1 A ≤2 sin = 2 sin . 2 2 MA 1 Suy ra M B1 +M C1 ≥ 2 sin A . Tương tự 2 ta có MB 1 MC 1 M C1 +M A1 ≥ 2 sin B , M A1 +M B1 ≥ 2 sin C . 2 2 Vậy Hình 1.24 M A2 M B2 M C2 + + (M B1 + M C1 )2 (M C1 + M A1 )2 (M A1 + M B1 )2 1 1 1 1 ≥ + + 4 sin2 A sin2 B sin2 C 2 2 2 3 1 3 1 ≥ 3 ≥ 3 = 3. 4 sin2 A 2 B 2 C sin 2 sin 2 4 1 2 2 8 Đẳng thức xảy ra khi và chỉ khi ∆ABC đều và M là tâm của tam giác. Bài toán 1.26. (Trích đề thi học sinh giỏi Quốc gia, 1991) Cho tam giác ABC với trọng tâm là G và nội tiếp trong đường tròn bán kính R, các đường trung tuyến của tam giác này xuất phát từ các đỉnh A, B, C kéo dài cắt đường tròn lần lượt tại D, E, F . Chứng minh rằng 3 1 1 1 √ 1 1 1 ≤ + + ≤ 3 + + . R GD GE GF AB BC CA Giải. Gọi M, N, P lần lượt là trung điểm của cạnh BC, CA, AB. Từ a2 AM.M D = BM.M C có M D = 4ma . Suy ra ma a2 GD = GM + M D = + (1.57) 3 4ma www.MATHVN.com - HOANG NGOC QUANG, Yen Bai
  • 36. 34 Từ (1.57), áp dụng bất đẳng thức AM − a2 √ ⇒ 1 ≤ a GM ta được GD ≥ 2 12 = 3 GD √ √ √ 3 1 3 1 3 BC .Tương tự, có GE ≤ CA , GF ≤ AB . Cộng theo vế 3 bất đẳng thức này ta được 1 1 1 √ 1 1 1 GD + GE + GF ≤ 3 AB + BC + CA . GA 2ma Lại từ (1.57) có GD = = ma a2 3 3 + 4ma 8m2a 2 + 3a2 . Áp dụng công thức đường 4ma Hình 1.25 2 2 2 GA 2b + 2c − a trung tuyến được GD = . a2 + b2 + c2 2 2 +3c2 Tương tự tính GB và GC rồi cộng lại được GD + GB + GC = 3aa2+3b2 +c2 = 3. GE GF GA GE GF +b Từ đó GD + BE + GF = 6. Để ý rằng AD, BE, CF đều không lớn hơn AD GE CF 1 1 1 3 2R, thay vào ta được GD + GE + GF ≥ R . Bài toán 1.27. (Tây ban nha, 1998) Một đường thẳng chứa trọng tâm G của tam giác ABC cắt cạnh AB tại P và cạnh CA tại Q. Chứng minh rằng P B . QC ≤ 1 . P A QA 4 2 P B QC 1 PB QC PB QC Giải. Vì P A . QA ≤ 4 PA + QA , ta sẽ chứng minh PA + QA = 1. Vẽ BB , CC song song với trung tuyến AA mà B , C nằm trên P Q. Các tam giác AP G và BP B đồng dạng ; tam giác AQG và CQC cũng đồng dạng, do đó P B = BB và QC = PA AG QA CC AG . Cộng 2 đẳng thức này lại với lưu ý rằng AG = 2GA = BB + CC ta được P B + QC = 1. Từ đó được bất PA QA Hình 1.26 đẳng thức cần chứng minh. Bài toán 1.28. (Ba Lan, 1999) Cho D là một điểm trên cạnh BC của tam giác ABC sao cho AD > BC. Điểm E trên CA thỏa mãn AE BD = . Chứng minh rằng AD > BE. EC AD − BC Giải. www.MATHVN.com - HOANG NGOC QUANG, Yen Bai
  • 37. 35 Lấy F trên AD sao cho AF = BC và Gọi E là giao điểm của BF và AC. Áp dụng định lí hàm số sin cho tam giác AE F, BCE và BDF , ta AE được E C = AF. sin AF E . sin BE C = sin AE F BC. sin CBE sin BF D BD AE = FD = EC . Do đó E ≡ E. Hình 1.27 sin DBF Lấy G trên BD sao cho BG = AD và H là giao điểm của GE với đường thẳng qua A và song song với BC. Dễ thấy các tam giác ECG và EAH đồng dạng nên AH = EC = AD−BC = BG−BC = BD , suy ra AH = DB. CG AE BD BD CG Do đó BDAH là một hình bình hành, suy ra BH = AD và ∆BHG cân. Vậy BH = BG = AD > BE. Bài toán 1.29. (Tạp chí THTT số 265) Gọi AD, BE, CF là các đường phân giác trong của tam giác ABC. Chứng minh rằng 1 p(DEF ) ≥ p(ABC), 2 trong đó kí hiệu p(XY Z) là chu vi của tam giác XY Z. Đẳng thức xảy ra khi nào ? Giải. Từ tính chất của đường AE c phân giác BE ta có CE = a , suy ra AE = AE+CE = a+c . b AE c bc Do đó AE = a+c . Tương tự bc AF = a+b . Theo định lí cosin trong ∆AEF và ∆ABC có Hình 1.28 2 2 2 EF =AE + AF − 2AF.AF. cos A 2 2 bc bc b 2 c2 b2 + c2 − a2 = + −2 . a+c a+b (a + c) (a + b) 2bc www.MATHVN.com - HOANG NGOC QUANG, Yen Bai
  • 38. 36 a2 bc abc (a + b + c) (b − c)2 = − (a + c) (a + b) (a + c)2 (a + b)2 a2 bc Suy ra EF 2 ≤ (a+c)(a+b) . Từ đó √ √ 2 2 a2 bc 1√ √ 1 ac + ab EF ≤ √ √ = . ac. ab ≤ 4 ac. ab 4 4 2 2 2 1 a+c a+b 1 2a + b + c ≤ + = . 16 2 2 16 2 Do đó EF ≤ 2a+b+c . Tương tự F D ≤ 2b+c+a , DE ≤ 2c+a+b . 8 8 8 1 Cộng theo vế 3 bất đẳng thức trên được DE + EF + F D ≤ 2 (a + b + c). hay p(DEF ) ≥ 1 p(ABC). 2 Bài toán 1.30. (IMO, 1991 ) Cho tam giác ABC. Gọi I là tâm đường tròn nội tiếp tam giác. Đường phân giác trong của các góc A, B, C lần lượt cắt các cạnh đối diện tương ứng tại L, M, N . Chứng minh rằng 1 AI.BI.CI 8 ≤ ≤ . 4 AL.BM.CN 27 Giải. Sử dụng tính chất đường phân giác có BL = c , để ý rằng BL + LC b ac LC = a, ta được BL = b+c và ab LC = b+c . Tiếp tục, áp dụng tích chất đường phân giác cho phân giác BI của góc ABL ta thu được IL BL ac a AI = AB = (b+c)c = b+c . Do đó AL AI+IL IL a AI = AI = 1 + AI = 1 + b+c = a+b+c AI b+c Hình 1.29 b+c . Khi đó, AL = a+b+c . Tương BI c+a CI a+b tự, BM = a+b+c , CN = a+b+c . Do đó bất đẳng thức cần chứng minh đưa về dạng chứa các biến a, b và c 1 (b + c)(c + a)(a + b) 8 < ≤ . 4 (a + b + c)3 27 www.MATHVN.com - HOANG NGOC QUANG, Yen Bai
  • 39. 37 Áp dụng bất đẳng thức AM − GM có 3 (b + c) + (c + a) + (a + b) 8 (b + c)(c + a)(a + b) ≤ ≤ (a + b + c)3 . 3 27 Bất đẳng thức phải được chứng minh. Để chứng minh bất đẳng thức trái, trước hết để ý rằng (b + c)(c + a)(a + b) (a + b + c)(ab + bc + ca) − abc = . (1.58) (a + b + c)3 (a + b + c)3 Biết rằng a + b + c = 2p, ab + bc + ca = p2 + r2 + 4rR, abc = 4Rrp thay vào (1.58) được (b + c)(c + a)(a + b) 2p(p2 + r2 + 4rR) − 4Rrp = = (a + b + c)3 8p3 2p2 + 2pr2 + 4Rrp 1 2r2 + 4Rr 1 = = + > . 8p3 4 8p2 4 Bài toán 1.31. (IMO Shorlist, 1996) Cho ABC là tam giác đều và P là một điểm trong nó. Các đường thẳng AP, BP, CP cắt các cạnh BC, CA, AB tại các điểm A1 , B1 , C1 , tương ứng. Chứng minh rằng A1 B1 .B1 C1 .C1 A1 ≥ A1 B.B1 C.C1 A. Giải. Áp dụng định lí hàm số cosin cho ∆CA1 B1 , ta được A1 B1 = A1 C 2 + B1 C 2 − A1 C.B1 C ≥ A1 C.B1 C. 2 2 Tương tự, B1 C1 ≥ B1 A.C1 A, C1 A1 ≥ C1 B.A1 B. Nhân 3 bất đẳng thức này, ta được A1 B1 .B1 C1 .C1 A2 ≥ A1 C.B1 C.B1 A.C1 A.C1 B.A1 B. 2 2 1 (1.59) Bây giờ, các đường thẳng AA1 , BB1 , CC1 đồng quy, vì vậy áp dụng định lí Ceva ta có A1 B.B1 C.C1 A = AB1 .BC1 .CA1 , thay vào (1.59) ta thu được bất đẳng thức cần chứng minh. Đẳng thức xảy ra khi và chỉ khi CA1 = CB1 , BA1 = BC1 và AB1 = AC1 . Điều này xảy ra khi và chỉ khi P là tâm của đường tròn ngoại tiếp tam giác ABC. www.MATHVN.com - HOANG NGOC QUANG, Yen Bai
  • 40. 38 Bài toán 1.32. (IMO Shorlist, 1999) Cho tam giác ABC và M là một điểm nằm trong nó. Chứng minh rằng min {M A, M B, M C} + M A + M B + M C < AB + BC + CA. Trước hết, ta chứng minh bổ đề sau: Bổ đề 1.2. Nếu M là một điểm nằm trong tứ giác lồi ABCD thì M A + M B < AD + DC + CB. Chứng minh. Hình 1.30 Hình 1.31 Gọi N là giao điểm của AM và CD (Hình 1.30). Khi đó M A + M B < M A + M N + N B ≤ AN + N C + CB ≤ AD + DN + N C + CB = AD + DC + CB. Giải. Lấy D, E, F theo thứ tự là trung điểm của BC, CA, AB (Hình 1.31). Xét điểm M nằm trong tam giác, M sẽ thuộc ít nhất hai trong ba hình thang ABDE, BCEF, CAF D. Không mất tính tổng quát, giả sử M ∈ ABDE, BCEF . Áp dụng bổ đề trên ta có M A + M B < AE + ED + DB, M B + M C < BF + F E + EC. Cộng theo vế hai bất đẳng thức trên, ta được M B + (M A + M B + M C) < AB + BC + CA. Vậy min {M A, M B, M C} + M A + M B + M C < AB + BC + CA. www.MATHVN.com - HOANG NGOC QUANG, Yen Bai
  • 41. 39 Bài toán 1.33. (IMO Shorlist, 2002) Cho tam giác ABC và F là một điểm trong nó thỏa mãn AF B = BF C = CF A . Các đường thẳng BF và CF cắt các cạnh AC và AB tại D và E, tương ứng. Chứng minh rằng AB + AC ≥ 4DE. Trước hết, ta chứng minh bổ đề sau: Bổ đề 1.3. Cho tam giác ABC, các điểm P và Q nằm trên các tia FD, FE tương ứng, sao cho P F ≥ λDF, QF ≥ λEF , trong đó λ > 0. Nếu P F Q ≥ 900 thì P Q ≥ λDE. Chứng minh. Đặt P F Q = θ. Vì θ ≥ 900 , ta có cos θ ≤ 0. Bây giờ, áp dụng định lý hàm số cosin, ta có P Q2 = P F 2 + QF 2 − 2P F.QF. cos θ ≥ (λDF )2 + (λEF )2 − 2 (λDF ) . (λEF ) . cos θ = (λDE)2 . Do đó P Q ≥ λDE. Giải. Lưu ý rằng AF E = BF E = CF D = AF D = 600 . Gọi P, Q là giao điểm của các đường thẳng BF, EF với đường tròn ngoại tiếp tam giác CF A, AF B tương ứng. Khi đó, dễ dàng thấy cả hai tam giác CP A, AQB là đều. Gọi P1 là chân đường vuông góc hạ từ F xuống cạnh AC và giả sử đường trung trực của AC cắt đường tròn ngoại tiếp tam giác CF A tại P và P2 . Gọi M là trung điểm PD của AC. Khi đó DF = PM PM F P1 ≥ M P2 = 3 vì vậy P F ≥ 4DF . Tương tự, ta có QF ≥ 4EF . Áp dụng hệ quả trên với λ = 4 và θ = DEF = 1200 ta được P Q ≥ 4DE. Cuối cùng, áp dụng bất đẳng thức Hình 1.32 tam giác, AB + AC = AQ + AP ≥ P Q ≥ 4DE. Bài toán 1.34. (IMO, 2006) Cho tam giác ABC với I là tâm đường tròn nội tiếp. Một điểm P nằm trong tam giác thỏa mãn P BA+ P CA = www.MATHVN.com - HOANG NGOC QUANG, Yen Bai
  • 42. 40 P BC + P CB. Chứng minh rằng AP > AI và đẳng thức xảy ra khi và chỉ khi P ≡ I. Giải. Đặt A = α, B = β, C = γ. Vì P BA + P CA + P BC + P CB = β+γ β + γ nên từ điều kiện bài toán ta có P BC + P CB = . Suy ra α 2 BP C = 900 + . 2 β+γ Mặt khác, BIC = 1800 − = α 2 900 + . Do đó BP C = BIC, và vì 2 P và I nằm cùng phía với BC nên các điểm B, I, P, C cùng nằm trên một đường tròn. Nói cách khác, P nằm trên đường tròn ω ngoại tiếp tam giác BCI. Gọi Ω là đường tròn ngoại tiếp tam giác ABC. Rõ ràng tâm của ω là trung điểm M của cung BC của Ω. Đây cũng là giao điểm thứ hai của phân giác AI và ω. Từ tam giác AP M Hình 1.33 ta có AP + P M ≥ AM = AI + IM = AI + P M . Do đó AP ≥ AI. Đẳng thức xảy ra khi và chỉ khi P nằm trên đoạn AM , điều này xảy ra khi và chỉ khi P ≡ I. 1.7. Các bất đẳng thức trong tứ giác Kí hiệu ABCD là tứ giác lồi với các đỉnh là A, B, C, D được vẽ theo một chiều nhất định nào đó (cùng chiều kim đồng hồ hay ngược chiều kim đồng hồ). Để đơn giản, độ lớn của góc ứng với các đỉnh A, B, C, D cũng được kí hiệu là A, B, C, D. Độ dài các cạnh của tam giác: AB = a, BC = b, CD = c, DA = d. a+b+c+d Nủa chu vi của tứ giác: p = . 2 Độ dài các đường chéo: AC = m, BD = n. Diện tích của tứ giác: S = SABCD hay [ABCD] www.MATHVN.com - HOANG NGOC QUANG, Yen Bai
  • 43. 41 1.7.1. Các bất đẳng thức cơ bản trong tứ giác Bài toán 1.35. Cho tứ giác ABCD, có AB + BD ≤ AC + DC. Chứng minh AB < AC. Giải. Gọi O là giao điểm của AC và BD . Xét các tam giác OAB và ODC ta có AB < OA + OB, DC < OC + OD. Do đó AB + CD < (OA + OC) + (OB + OD) =AC + BD. (1.60) Mặt khác, theo giả thiết ta có AB + BD ≤ AC + DC. (1.61) Hình 1.34 Cộng theo vế (1.60) và (1.61) ta được 2AB + DC + BD < 2AC + BD + DC. Suy ra AB < AC. Bài toán 1.36. Tổng hai đường chéo của một tứ giác lồi ABCD nhỏ hơn chu vi của tứ giác và lớn hơn nửa chu vi của nó. Giải. Gọi O là giao điểm của AC và BD, ta có AC + BD = (OA + OB) + (OC + OD) > AB + CD. (1.62) AC + BD = (OA + OD) + (OB + OC) > AD + BC. (1.63) Cộng theo vế các bất đẳng thức (1.62) và (1.63) ta thu được AB + BC + CD + DA AC + BD > . 2 Mặt khác, AC < AB + BC và AC < DA + CD. Cộng theo vế hai bất đẳng thức này, ta có AB + BC + CD + DA AC < . (1.64) 2 Tương tự, AB + BC + CD + DA BD < . (1.65) 2 Cộng theo vế hai bất đẳng thức (1.64) và (1.65) , ta thu được AC + BD < AB + BC + CD + DA. www.MATHVN.com - HOANG NGOC QUANG, Yen Bai
  • 44. 42 Bài toán 1.37. Cho tứ giác ABCD, gọi M, N, P, Q lần lượt là trung điểm của các cạnh AB, BC, CD, DA. Chứng minh rằng AD + BC a)M P ≤ . (1.66) 2 AB + BC + CD + DA b)M P + N Q ≤ . (1.67) 2 Giải. a) Gọi I là trung điểm của BD, M I là đường trung bình của ∆ABD. Suy ra M I = AD . IP là đường trung bình của 2 ∆DBC, suy ra IP = BC . Xét 3 điểm 2 I, M, P ta có M P ≤ M I + IP , suy ra M P ≤ AD+BC . 2 b) Áp dụng ý a) ta có N Q ≤ AB+DC . 2 AD+BC AB+DC Do đó M P + N Q ≤ 2 + 2 = AB+BC+CD+DA Hình 1.35 2 . Bài toán 1.38. Cho hình vuông ABCD cạnh a. M, N là hai điểm ở √ trong hình vuông đã cho. Chứng minh rằng M N ≤ a 2. Giải. Vì ABCD là hình vuông cạnh √ √ a nên AC = AB 2 = a 2. Vẽ đường tròn (O) ngoại tiếp hình vuông ABCD. Ta có đường √ kính của (O) là a 2, M và N là hai điểm nằm trong O. Gọi M N là dây cung đi qua M và N . Ta có M N ≤ M N mà √ M N ≤ a 2 (đường kinh là dây cung lớn nhất trong đường tròn). √ Do đó M N ≤ a 2. Hình 1.36 Bài toán 1.39. Cho hình thang ABCD có đáy nhỏ là AB và C + D ≤ 900 . Gọi M và N lần lượt là trung điểm của các cạnh AB và CD. Chứng minh rằng M N ≤ CD−AB . 2 www.MATHVN.com - HOANG NGOC QUANG, Yen Bai
  • 45. 43 Giải. Qua M vẽ đường thẳng song song với AD cắtDC tại E và qua M vẽ đường thẳng song song với BC cắt DC tại F . Suy ra D = E1 , C = F1 . Suy ra E1 + F1 = D + C ≤ 900 ⇒ EM F ≥ 900 . Theo bài toán 1.3 tam giác M EF có EM F ≥ 900 và M N là Hình 1.37 trung tuyến nên M N ≤ EF . 2 Mặt khác, có AB//DC và AD//M E nên ADEM là hình bình hành. Suy ra DE = AM = 1 AB, tương tự F C = M B = 1 AB. Do đó 2 2 EF = CD − AB. Vậy M N ≤ CD−AB . 2 Bài toán 1.40. Cho tứ giác ABCD, M là một điểm thuộc cạnh CD (M khác C, D). Chứng minh rằng M A + M B < max {CA + CB; DA + DB} . (1.68) Giải. Gọi A là điểm đối xứng của A qua CD. A B cắt CD ở P . Vì M thuộc đoạn CD nên M thuộc ∆A BC hoặc ∆A BD. Theo định lí 1.20 ta có M A + M B < CA + CB M A + M B < DA + DB M A + M B < CA + CB Hình 1.38 ⇒ M A + M B < DA + DB Do đó M A + M B < max {CA + CB; DA + DB}. Chú ý 1.3. Từ bài toán 1.40 ta có các kết quả sau: 1) Cho tứ giác ABCD, M là một điểm thuộc cạnh CD (M có thể trùng với C hoặc D). Ta có bất đẳng thức M A + M B ≤ max {CA + CB; DA + DB}. www.MATHVN.com - HOANG NGOC QUANG, Yen Bai
  • 46. 44 2) Cho ABCD là hình chữ nhật và điểm M nằm trên cạnh CD. Ta có bất đẳng thức M A + M B ≤ CA + CB. 3) Cho ABCD là hình vuông cạnh a và điểm M nằm trên cạnh CD. Ta √ có bất đẳng thức M A + M B ≤ (1 + 2)a. Đẳng thức trong các bất đẳng thức trên xảy ra khi và chỉ khi M ≡ C hoặc M ≡ D. Bài toán 1.41. (Đề thi vào lớp 10 chuyên toán-tin, ĐHSP, ĐHQG Hà Nội 1998-1999) Cho hình chữ nhật ABCD và điểm M nằm trong hình chữ nhật và có thể nằm trên các cạnh của ABCD. Chứng minh rằng M A + M B + M C + M D ≤ AB + AC + AD. Giải. Qua M vẽ đường thẳng song song với AD cắt AB, DE lần lượt tại E, F . Áp dụng chú ý 1.3 của bài toán 1.40 vào các hình chữ nhật AEF D, EBCF và ABCD ta có M A+M D ≤ EA+ED, M B+M C ≤ EB + EC, ED + EC ≤ AD + AC. Do đó M A + M B + M C + M D ≤ (EA + EB) + Hình 1.39 (ED + EC) ≤ AB + AC + AD. Ta có bài toán tổng quát hơn sau đây Bài toán 1.42. (Tuyển tập 5 năm tạp chí THTT) Cho tứ giác ABCD, M là một điểm trong tứ giác. Đặt dA = AB + AC + AD, dB = BC + BD + BA, dC = CD + CA + CB, dD = DA + DB + DC. Chứng minh rằng M A + M B + M C + M D < max {dA ; dB ; dC ; dD } . Giải. Kéo dài AM một đoạn M B bằng M B. Qua M kẻ đường trung trực của BB . Đường này theo thứ tự cắt hai cạnh tứ giác tại I, J. Có thể xảy ra một trong ba trường hợp hình (A), (B), (C). Vì trong các hình (B), (C) bài toán được chứng minh tương tự nhưng đơn giản hơn trong trường hợp (A) nên ở đây ta chỉ chứng minh trong trường hợp (A). Không mất tính tổng quát giả sử rằng IC + ID = max {IC + ID, JC + JD} . www.MATHVN.com - HOANG NGOC QUANG, Yen Bai
  • 47. 45 Hình 1.40 Áp dụng bài toán 1.40 cho tứ giác CIJD ta có M C +M D < IC +ID. Lại có M A + M B = M A + M B = AB < IA + IB = IA + IB. Do đó M A + M B + M C + M D < IA + IB + IC + ID = IA + ID + BC. Áp dụng bài toán 1.40 cho tứ giác ABCD ta có IA + ID < max {CA + CD; BA + BD} . Vậy M A + M B + M C + M D < max {CA + CD; BA + BD} + BC = = max {BC + BD + BA; CD + CA + CB} = max {dA ; dC } ≤ max {dA ; dB ; dC ; dD } . Chú ý 1.4. Từ bài toán 1.42 ta có các kết quả sau: 1) Cho hình chữ nhật ABCD có độ dài các cạnh là a, b và độ dài đường chéo là c. M là một điểm nằm bên trong hình chữ nhật đó. Ta có bất đẳng thức M A + M B + M C + M D < a + b + c. 2) Cho hình vuông ABCD cạnh và M là một điểm nằm bên trong hình √ vuông đó. Ta có bất đẳng thức M A + M B + M C + M D < (2 + 2)a. 1.7.2. Các bất đẳng thức khác trong tứ giác Bài toán 1.43. (IMO shorlist) Diện tích của một tứ giác với các cạnh a, b, c và d là S. Chứng minh rằng a+c b+d S≤ . . 2 2 Giải. Trước tiên, giả sử tứ giác ABCD không lồi. Khi đó một trong các đường chéo của nó, chẳng hạn BD sẽ không có điểm chung với phần trong của tứ giác. www.MATHVN.com - HOANG NGOC QUANG, Yen Bai